X



トップページ数学
1002コメント379KB
大学学部レベル質問スレ 18単位目
レス数が1000を超えています。これ以上書き込みはできません。
0001132人目の素数さん
垢版 |
2022/04/28(木) 21:13:06.70ID:oq75KvzG
大学で習う数学に関する質問を扱うスレ

・質問する前に教科書や参考書を読むなりググるなりして
・ただの計算は
http://wolframalpha.com
・数式の表記法は
http://mathmathmath.dote ra.net
・質問のマルチポストは非推奨
・煽り、荒らしはスルー

関連スレ
分からない問題はここに書いてね478
http://rio2016.5ch.net/test/read.cgi/math/1511604229/

※前スレ
大学学部レベル質問スレ 17単位目
https://rio2016.5ch.net/test/read.cgi/math/1637449244/
0002132人目の素数さん
垢版 |
2022/04/28(木) 21:19:23.90ID:oq75KvzG
分からない問題はここに書いてねスレは現行落ちてるようですが、
消すのもなんだったので、前スレ>>1に書いてたのをとりあえずそのまま書いてます
0003132人目の素数さん
垢版 |
2022/04/28(木) 21:53:16.62ID:37/SqDmQ
>>991
>排中律と矛盾律の区別すらつかんのかおまえら
排中律
P∨¬P
(君の言うところの)矛盾律
¬(P∧¬P)
古典論理ではどちらも真
特にドモルガンの法則で同値変形できる
直観主義論理では前者は真ではなく後者は真
開集合とその補集合の内包の合併集合は一般に全体集合でないけれども
開集合とその補集合の内包の共通部分の補集合の内包は必ず全体集合になるからね
0004132人目の素数さん
垢版 |
2022/04/28(木) 21:56:25.03ID:37/SqDmQ
同様に
排中律の否定¬(P∨¬P)は古典論理ではもちろん直観主義論理でも偽
開集合とその補集合の内包の合併集合の補集合の内包は空集合だからね
0005132人目の素数さん
垢版 |
2022/04/28(木) 23:37:51.33ID:XL8AiXBD
三角形の五心(重心、垂心等々)のどれについても、3本の直線が1点で交わるのが不思議でなりません。
なにか深い理由があるのでしょうか?(個々の場合がそうであるのはもちろん分かるのですが)
0006132人目の素数さん
垢版 |
2022/04/29(金) 06:44:36.78ID:mJMIOEKH
((A→B)∧(¬A→B))→Bは直観主義論理でも成り立つのかどうか、考えたけどわからなかった
0007132人目の素数さん
垢版 |
2022/04/29(金) 07:26:43.82ID:vvTdXbVu
((A→B)∧(¬A→B))→B
=¬((¬A∨B)∧(¬¬A∨B))∨B
=¬((¬A∧¬¬A)∨B)∨B
=¬(¬(A∨¬A)∨B)∨B
=¬B∨B
成立するとは言えません
0008132人目の素数さん
垢版 |
2022/04/29(金) 09:41:42.56ID:vvTdXbVu
>>5
現実を受け入れましょう
0011132人目の素数さん
垢版 |
2022/04/29(金) 14:02:48.47ID:J8ImCyMJ
>>9
先の1か所では全く回答が得られなかったので

>>8
当然既知の問題だろうと思うのですがどこにも答えがみつかりません。
もしわかれば教えてください。
0012132人目の素数さん
垢版 |
2022/04/29(金) 14:04:41.33ID:+oeK6kIJ
>>11
>当然既知の問題だろうと思うのですが
問題にしている人を見たことありません
0013132人目の素数さん
垢版 |
2022/04/29(金) 14:14:06.46ID:J8ImCyMJ
>>12
答えは自明だということでしょうか?
疑問が問題として成立していないということでしょうか?
それとも超難問ということでしょうか?(それはないと思いますが)

どこかで話題にされていることだと思うのですが。
0014132人目の素数さん
垢版 |
2022/04/29(金) 14:16:21.62ID:+oeK6kIJ
>>13
1点で交わることが証明できるので証明されたわけです
その「意味」を考えるのは「意味」があるのかどうか
0016132人目の素数さん
垢版 |
2022/04/29(金) 14:19:31.75ID:+oeK6kIJ
もちろん「意味」を考えようとするのは「無意味」ではないかもしれません
面白い解釈・説明を思いついたら教えてください
0017132人目の素数さん
垢版 |
2022/04/29(金) 14:20:23.28ID:+oeK6kIJ
でもこのスレでなくて
より適切なスレがあるかも
0018132人目の素数さん
垢版 |
2022/04/29(金) 21:02:05.07ID:J8ImCyMJ
>>16
「意味」というより、むしろ「形式」に関する不思議さという感じです。

>>17
そうですね。たしかに迷ったのですが。
数学基礎論?あるいは?
でも、そんなに珍しい問いとも思えないのですが
0019132人目の素数さん
垢版 |
2022/04/29(金) 21:40:22.33ID:vvTdXbVu
>>18
>「意味」というより、むしろ「形式」に関する不思議さという感じです。
形式とは?
0020132人目の素数さん
垢版 |
2022/04/29(金) 21:41:04.78ID:vvTdXbVu
>>18
>でも、そんなに珍しい問いとも思えないのですが
1+1=2になるのは不思議ですと言っているように見えますよ
0021132人目の素数さん
垢版 |
2022/04/30(土) 00:19:13.00ID:q4MfboGi
>>15で終わってる話だとは思うが
>>5はEncyclopedia of Triangle Centersでも眺めとけば

分野的には総合幾何学 (synthetic geometry) だろうが現代的にはドマイナーだろ
>>18は一体どっから基礎論出てきた?
0022132人目の素数さん
垢版 |
2022/04/30(土) 12:00:08.12ID:JrpS5AMt
△ABCで点の名前を取り替えても引かれる直線は同じだから
3直線の交点は一般には3点あるけど
点の名前を入れ替えても変わらないから実は1点
こういうことを言わせたかったのか
0023132人目の素数さん
垢版 |
2022/04/30(土) 12:11:23.46ID:/9DIEapR
999 返信:132人目の素数さん[sage] 投稿日:2022/04/28(木) 21:40:27.83 ID:+gaZyQqp [1/2]
>>996
だからAさんが計算したらJordanの標準形がXになりました
Bさんが計算したらYになりました
そんな事が起こるのかでしょ?
もちろん答えは起こらない、なぜか、で紹介されてる話が
XとYが同じ行列Aと相似ならXとYも相似にならざるをえず、その場合任意の整数kに対してrank(X^k)とrank(Y^k)は一致しないといけないでしょ?

ありがとうございます。

齋藤正彦著『線型代数演習』の一意性の説明を見て、分かりました。
0024132人目の素数さん
垢版 |
2022/04/30(土) 12:12:24.67ID:JrpS5AMt
たぶんそういう攻め方では示せないと思うけれど
期待しているのはそういう「原理」?
0025132人目の素数さん
垢版 |
2022/04/30(土) 13:17:45.14ID:YWfy5BEL
>>24
そうそう
0026132人目の素数さん
垢版 |
2022/04/30(土) 15:34:21.37ID:IP1jGD2r
>>25
簡単な証明が欲しいと泣いても仕方ありませんよ
0027132人目の素数さん
垢版 |
2022/04/30(土) 20:47:34.44ID:/9DIEapR
齋藤正彦著『齋藤正彦 線型代数学』

以下の命題が当然成り立つと書かれています。
証明してください。

-----------------------------------------------

T を有限次元ベクトル空間 V 上の線形変換とする。
α を T の固有値とする。
m を α の重複度とする。
W(α) を α に属する広義固有空間とする。

dim W(α) ≦ m

が成り立つ。
0028132人目の素数さん
垢版 |
2022/04/30(土) 20:55:06.53ID:/9DIEapR
>>27

あ、分かりました。
0029132人目の素数さん
垢版 |
2022/04/30(土) 20:58:58.78ID:/9DIEapR
>>27

あ、やっぱり分かりません。

証明してください。
0030132人目の素数さん
垢版 |
2022/04/30(土) 21:12:55.96ID:vz0MgWS9
jordan分解したら当たり前
つまりそもそも質問する時にjordan分解使っていいのかを明示しとかんと質問にならん
0032132人目の素数さん
垢版 |
2022/04/30(土) 21:32:34.78ID:/9DIEapR
>>27

齋藤正彦さんが、この命題が成り立つのは「当然」と書いているのは、有限次元のベクトル空間が広義固有空間の直和に分解できるという定理の証明の中でです。

T を有限次元ベクトル空間 V 上の線形変換とする。 T の異なる固有値の全部を β_1, …, β_p, それらの重複度を m_1, …, m_p とする。

V = W(β_1) (+) W(β_2) (+) … (+) W(β_p)

が成り立つ。

齋藤さんは、 dim W(β_i) ≧ m_i のほうは証明しています。

そして、 W(β_1) + W(β_2) + … + W(β_p) が直和であることも証明しています。


ちょっと思ったのですが、有限次元のベクトル空間が広義固有空間の直和に分解できるという定理を証明するためには、

dim W(β_i) = m_i であることを証明する必要はなく、 dim W(β_i) ≧ m_i が証明できさえすれば充分ですよね?

dim [W(β_1) (+) W(β_2) (+) … (+) W(β_p)] = dim W(β_1) + dim W(β_2) + … + dim W(β_p) ≧ m_1 + m_2 + … + m_p = n

から、 dim [W(β_1) (+) W(β_2) (+) … (+) W(β_p)] = n が分かりますし、結果的に、 dim W(β_i) = m_i となることも分かります。
0033132人目の素数さん
垢版 |
2022/04/30(土) 21:46:41.65ID:vz0MgWS9
>>32

> 齋藤さんは、 dim W(β_i) ≧ m_i のほうは証明しています。
>
> そして、 W(β_1) + W(β_2) + … + W(β_p) が直和であることも証明しています。

この2つ証明しててまだ分からないならその本にで出せるレベルにないよ
0034132人目の素数さん
垢版 |
2022/04/30(土) 22:03:23.86ID:/9DIEapR
dim W(β_i) = m_i

が成り立つことは

>>33

に書いた通り、分かります。

分からないのは、齋藤正彦さんが dim W(β_i) ≦ m_i が成り立つのは「当然」と書いたことです。
0035132人目の素数さん
垢版 |
2022/04/30(土) 22:04:45.64ID:/9DIEapR
訂正します:

dim W(β_i) = m_i

が成り立つことは

>>32

に書いた通り、分かります。

分からないのは、齋藤正彦さんが dim W(β_i) ≦ m_i が成り立つのは「当然」と書いたことです。
0037132人目の素数さん
垢版 |
2022/04/30(土) 22:08:37.29ID:/9DIEapR
齋藤正彦さんの本には、このような意味不明な記述が沢山あります。

完成度が佐武一郎さんの本に比べてずっと低いと思います。
0038132人目の素数さん
垢版 |
2022/04/30(土) 22:14:30.38ID:/9DIEapR
>>36

齋藤正彦さんの有限次元のベクトル空間が広義固有空間の直和に分解できるという定理の証明の流れを書きます。
それを読めば、齋藤正彦さんが「当然」と書いたことが奇妙であることが分かると思います。

(1) V_p = W(β_1) + W(β_2) + … + W(β_p) が直和であることを証明している。
(2) V = V_p であることを示せば良いと書き、そのためには dim W(β_i) = m_i を示せばよいと書いている。
(3) dim W(β_i) ≧ m_i を証明し、逆の不等式は当然だから、 dim W(β_i) = m_i が成り立つと書いている。

もし、この証明が上のように書かれていなくて、以下のようだったなら、何も奇妙なところはなかったことになります:

(1) V_p = W(β_1) + W(β_2) + … + W(β_p) が直和であることを証明する。
(2) V = V_p であることを示せば良い。そのためには dim W(β_i) ≧ m_i を示せばよい。
(3) dim W(β_i) ≧ m_i を証明する。
0039132人目の素数さん
垢版 |
2022/04/30(土) 22:15:52.39ID:/9DIEapR
>>38

そして、結果的に

dim W(β_i) = m_i

が成り立つことも分かるということになります。
0040132人目の素数さん
垢版 |
2022/04/30(土) 22:17:24.09ID:vz0MgWS9
だからそんな事書かなくてもそのレベルの本が読めるレベルの人間なら当たり前だって言ってるんだよバーカ
0041132人目の素数さん
垢版 |
2022/04/30(土) 22:19:58.91ID:/9DIEapR
>>40

では証明してください。
以下の証明以外の証明をお願いします。


dim [W(β_1) (+) W(β_2) (+) … (+) W(β_p)] = dim W(β_1) + dim W(β_2) + … + dim W(β_p) ≧ m_1 + m_2 + … + m_p = n
∴ dim [W(β_1) (+) W(β_2) (+) … (+) W(β_p)] = n
∴ dim W(β_i) = m_i
0042132人目の素数さん
垢版 |
2022/04/30(土) 22:22:59.73ID:/9DIEapR
結論としては、

(1) V_p = W(β_1) + W(β_2) + … + W(β_p) が直和であることを証明する。
(2) V = V_p であることを示せば良い。そのためには dim W(β_i) ≧ m_i を示せばよい。
(3) dim W(β_i) ≧ m_i を証明する。

と証明を書くべきところを、齋藤正彦さんはきちんと書けなかったということになります。
0044132人目の素数さん
垢版 |
2022/04/30(土) 22:41:33.57ID:vz0MgWS9
V = ⊕W(βᵢ)であるからn = ΣdimW(βᵢ)
miが多重度の全体だから n = Σmi
dimW(βᵢ)≧mi
∴ dimW(βᵢ)=mi (∀i)
バーカ
0045132人目の素数さん
垢版 |
2022/04/30(土) 23:39:04.63ID:JrpS5AMt
>>38
>(1) V_p = W(β_1) + W(β_2) + … + W(β_p) が直和であることを証明している。
>(2) V = V_p であることを示せば良いと書き、そのためには dim W(β_i) = m_i を示せばよいと書いている。
>(3) dim W(β_i) ≧ m_i を証明し、逆の不等式は当然だから、 dim W(β_i) = m_i が成り立つと書いている。
この流れで自然ですよ
>(1) V_p = W(β_1) + W(β_2) + … + W(β_p) が直和であることを証明する。
>(2) V = V_p であることを示せば良い。そのためには dim W(β_i) ≧ m_i を示せばよい。
>(3) dim W(β_i) ≧ m_i を証明する。
こちらでも結構ですよ
0046132人目の素数さん
垢版 |
2022/04/30(土) 23:41:20.47ID:JrpS5AMt
>>43
>明らかで済んだら教科書は要らない
まあここは明かで十分ですかね
0047132人目の素数さん
垢版 |
2022/05/02(月) 14:00:23.88ID:MlfB1dB5
統失です。
球面は、正方形の角の軌跡の集合なのでしょうか?
いろんな角度で正方形を回転させる。
0048132人目の素数さん
垢版 |
2022/05/02(月) 14:05:01.31ID:zy1+Ye+n
正四面体の角の中面の長さの軌跡と考える方が一般的です
0049132人目の素数さん
垢版 |
2022/05/02(月) 14:53:02.73ID:2kUu3ZDv
『数学セミナー2022年03月号』の「圏論入門の足掛かり」に、

C を圏としたとき、 C における2つの射 f, g : X → Y とそのイコライザーとその普遍性を表した図式が可換だと書かれています。

f, g : X → Y という図式を考えると、一般に f ≠ g なので、この部分で可換であるという条件が満たされないように思うのですが、どうなんでしょうか?

可換の定義が厳密に書いてないため、判断できません。
0050132人目の素数さん
垢版 |
2022/05/02(月) 15:48:44.48ID:pGRJEqRO
e:Z→Xがf,gのequalizer
:⇔
(1) 図式
Z→X
↓ ↓
X→Y
が可換(ただしZ→Xはどちらもe、X→Yは片方fでもう一方がg
(2) 略
の図式の話やろ?
f,gが等しくなくても可換になることなんて山のようにあるでしょ?
0051132人目の素数さん
垢版 |
2022/05/02(月) 16:02:03.06ID:oGW0u+cE
イコライザーの図式で可換と言っているのは、普遍性を満たす3つの対象の図式であって、
全体の図式(X→→Yを含む)は可換ではない
確かに初学者には、はっきり書いたほうが分かりやすいかもしれない
0053132人目の素数さん
垢版 |
2022/05/02(月) 16:39:17.73ID:bafKYVlf
>>51
なに言ってんのか分からない
0055132人目の素数さん
垢版 |
2022/05/02(月) 18:37:31.78ID:bafKYVlf
>>54
ん?X→Yの2つの射のところのこと?
この図全体の図式が可換というのは
図のどの部分を抜き出しても可換図式であるという意図?
ID:2kUu3ZDv もそのように解釈してるのかな?
イコライザーに関しては
>>50の(1)が可換なときウィキペの三角が可換となるuが存在するわけで
可換というとこの2ヶ所つまり>>50の説明が普通だと思う>>49
ウィキペのような書き方も良くあるけど
Y←X

X
のプルバックがイコライザという書き方にした方がいい
>>51で言っているのはそれ?
0056132人目の素数さん
垢版 |
2022/05/02(月) 19:15:43.04ID:2kUu3ZDv
>>55

そうです。
図式を、対象を点、射を辺とする有向グラフと考えたときに、有向パスで結ばれた2点を任意にとるとその2点を結ぶ任意のパスに対応する射の合成がすべて
等しいとき可換図式というのかと思っていました。
X → Y には2つの射 f, g があるので、可換図式であるためには、 f = g でなくてはならないのかと思っていました。
0057132人目の素数さん
垢版 |
2022/05/02(月) 20:10:39.93ID:exYQqzLc
まぁでもwikiに載ってる図式が可換と書いてあったらf=gになってしまう気はするけどな
wikiのレベルなんてそんなもん
0058132人目の素数さん
垢版 |
2022/05/02(月) 20:14:51.62ID:Bm4ClGqt
>>56
グラフ理論の定義が上手くいくか自分は分からないが、考え方はそれで合ってる
0059132人目の素数さん
垢版 |
2022/05/02(月) 20:27:54.96ID:2kUu3ZDv
『数学セミナー2022年03月号』の「ガウスの数論から現代数学へ(II)」(栗原将人)

「また、種の理論が相互法則だけで組み尽くせない力を持っていることも、わかっていただけると思う。高瀬正仁氏は[3]155ページで「ガウスの目には、
ガウス以前の素数の形状問題は特別な形で表現された平方剰余の理論のように映じたでしょう」と述べているが、これらの表はそうではないことを
語っていると思われる。」

脚注には、以下のように書かれています。

「[2]212ページには「ガウスの目には、素数の形状問題は平方剰余の理論の一区域のように見えたのではないかと思います」と同じ主張が述べられている。」


以下の文献を上の文章を書くためだけに引用しています。

[2] 高瀬正仁『ガウスの数論、わたしのガウス』筑摩書房(2011年)
[3] 高瀬正仁『ガウスに学ぶ初等整数論』東京図書(2017年)
0060132人目の素数さん
垢版 |
2022/05/02(月) 21:45:35.88ID:kqmCXTIx
>>56
>射の合成がすべて
>等しいとき可換図式というのかと思っていました。
可換な三角や四角は中に○矢印書くのが可換図式
0061132人目の素数さん
垢版 |
2022/05/02(月) 21:48:56.88ID:kqmCXTIx
>>57
イコライザの説明では良くある図ではある
ソースとターゲット同じ射f,gが(2点間の有向グラフとして)可換てのは
その通りf=gだよね
そういう無意味なことまで杓子定規に考えるのはよろしくなかろう
0062132人目の素数さん
垢版 |
2022/05/02(月) 21:57:28.83ID:Bm4ClGqt
自分も>>51で終わったものを何故そこまでややこしくしたのか不思議でならなかったな
0063132人目の素数さん
垢版 |
2022/05/02(月) 22:02:05.18ID:Bm4ClGqt
ちなみにイコライザーのwikipediaは一応きちんと「可換にする(eq○u=mを満たす)」と書いてある
読んでないが、引用を見る限り数学セミナーの書き方が混乱を招くものだったのかもしれない
0064132人目の素数さん
垢版 |
2022/05/02(月) 22:46:34.74ID:Sh/7oPin
正確に数式も併記しとけばいいんだよ
てかそもそも数式の方がメインで図式は参考図というのが基本
ましてや“辞書”として使える文章なら感覚的な捉えやすさと正確さの天秤は基本正確さの方を優先すべき
読者の感覚的理解も磨く事を目的ともする教科書とはそこが違う
0065132人目の素数さん
垢版 |
2022/05/03(火) 11:38:44.38ID:+4d/bR9f
正規形でないODEで重要なものってありますか?
0067132人目の素数さん
垢版 |
2022/05/04(水) 10:02:20.27ID:kBPIAUHG
>>66
連立一階常微分方程式です。
0068132人目の素数さん
垢版 |
2022/05/04(水) 10:19:09.98ID:+hrWGuHu
>>65
「重要」の定義も
0069132人目の素数さん
垢版 |
2022/05/04(水) 11:19:39.59ID:BHpsp0O5
>>68
ばぁ〜か🤪
うんこ食って寝ろ
0070132人目の素数さん
垢版 |
2022/05/04(水) 16:02:12.27ID:wllA7GIa
T を T(x_1, x_2, x_3, …, x_n) = (x_1, 2*x_2, 3*x_3, …, n*x_n) で定義される C^n 上の線形変換とする。

T の不変部分空間をすべて決定せよ。
0071132人目の素数さん
垢版 |
2022/05/04(水) 16:45:51.01ID:s1jxb931
>>69
おまODE知らんのだろ
重要性って人によって全然違うんだが?
0075132人目の素数さん
垢版 |
2022/05/04(水) 21:23:45.23ID:+hrWGuHu
>>74
やだやだゆうやつがやだ
0076132人目の素数さん
垢版 |
2022/05/04(水) 21:51:37.71ID:8ikhgo9W
( ´∀`) < 矢田屋だ
   ↑
  矢田
0077132人目の素数さん
垢版 |
2022/05/06(金) 10:01:12.48ID:6xrbKvNl
2×2行列A、Bがいずれも逆行列を持たず、A+Bは逆行列をもち、さらに、AB=BAならば、AB=0であることを証明せよ。


この問題への以下の回答がよくわからないので
解説して下さい、よろしくお願いします


Tacosan
A は 2つの独立な固有ベクトル x, y をもち, これらはどちらも B の固有ベクトルでもある. で条件から Ax = By
= 0 としてよいので AB = O.

https://oshiete.goo.ne.jp/qa/12932948.html
0078132人目の素数さん
垢版 |
2022/05/06(金) 11:56:11.70ID:6B9UxDz1
AもBも固有ベクトル分解を持たないならいずれのJordan cellは[[0,1],[0,0]]と相似で特に冪零になる
すると
(A+B)(A-B)=A²-B²=O
からA=Bになってしまうので矛盾
よってAに関する固有ベクトル分解V = Fx⊕Fyを持つとしてよい
A,Bが可換だからこれはBに関する固有ベクトル分解にもなっている
実際x,yに対する固有値をl,mとして
ABx = BAx = lBx
∴BxはAの固有値lに対する固有ベクトル
∴Bx = ux (∃u)
同様にBy = vy
さらに仮定からkとl、uとvのいずれかが0だがk=u=0なら(A+B)x=0となって仮定に反する
∴k=0, v=0 or l=0, u=0
前者のとき
ABx = BAx = 0、ABy= 0
でAB=0
後者も同様
0080132人目の素数さん
垢版 |
2022/05/06(金) 14:00:40.73ID:J2V25/dU
>>76
評価
0081132人目の素数さん
垢版 |
2022/05/06(金) 22:58:27.32ID:E32NkoOD
>>77
A,Bがいずれも逆行列を持たないのでいずれも0を固有値に持つ
つまりAx=0,By=0となるx≠0、y≠0がある
これらが両方の固有ベクトルであるかどうかはどうでもよくて
もしx,yが一次従属(つまり平行)ならば(A+B)x=0となってA+Bが逆行列を持つことと矛盾するからx,yは一次独立
なのでABx=BAx=0とABy=0よりAB=O
0082132人目の素数さん
垢版 |
2022/05/07(土) 10:12:42.10ID:AmqezNRt
>>77が回答として適当かといわれると微妙なところだな
あまり分かってない人が書いてそう
0083132人目の素数さん
垢版 |
2022/05/07(土) 10:19:24.81ID:Tocm4SRv
一行目からなんかすごいこと言ってるし
0084132人目の素数さん
垢版 |
2022/05/08(日) 00:08:03.49ID:xPQo1NTZ
微分形式がわかりません
わかりやすく教えて下さい
よろしくおねがいします!
0085132人目の素数さん
垢版 |
2022/05/08(日) 16:08:43.59ID:Os1Q4ACv
証明について質問です
↓で『定理5.6.56』と、これの群論による証明があります
 https://pisan-dub.jp/doc/2011/20110114001/5_6.html
これを群論によらない形で証明できないでしょうか? (背理法とか?)

-----
定理は以下の式についてのものです。
 X^n≡a (mod p) (p:素数)

@この式が解を持つとき、解の個数はd個。
  d=(n, p-1) (d:GCD(最大公約数))

Aこの式が解を持つ必要十分条件は、以下を満たすこと。
  a^((p-1)/d)≡1 (mod p)

-----
特に上記の@について証明を知りたいです
(元の記述から@Aの順番を入れ替えています)
よろしくお願いします
0086132人目の素数さん
垢版 |
2022/05/08(日) 17:17:56.93ID:SPrfFCn+
>>85
なぜ群論を避けたいの?
もちろんガウスは群論無しで証明したけれどかなりまわりくどいよ
0087132人目の素数さん
垢版 |
2022/05/08(日) 17:48:00.71ID:SPrfFCn+
z/pzが体であることを認めれば書けないことはないくらいの分量になるが、群論が嫌なのに体論を使うのもおかしいしな。
0088132人目の素数さん
垢版 |
2022/05/08(日) 18:03:23.43ID:Or033kxl
有理数より無理数のほうが圧倒的に多いようですが
対角線論法では、有理数の数え上げでは取りこぼすような数が少なくとも1つは存在する
という結論となっており、有理数より無理数のほうが圧倒的に多いという考えに至りません。
どうすれば有理数より無理数のほうが圧倒的に多いということを実感できるでしょうか?
0089132人目の素数さん
垢版 |
2022/05/08(日) 18:10:11.70ID:EXHQXD/I
偶数と自然数の濃度は同じです
同様に、10000000000000000の倍数と自然数の濃度も同じ
有理数と自然数の濃度も同じですね

これらの数はどんなに頑張っても1個余ることすらもないのに、無理数だと1個余ってしまうのです

めちゃくちゃ多いような気がしませんか?
0090132人目の素数さん
垢版 |
2022/05/08(日) 18:13:18.51ID:SPrfFCn+
測度論を勉強する
0091132人目の素数さん
垢版 |
2022/05/08(日) 18:18:36.18ID:6JZ1zf26
Qは可算なのにR\Qは非可算
Q~Nは例えばN×Nでも可算なので有理数を有理数個集めても無理数には全然足りない
0092132人目の素数さん
垢版 |
2022/05/08(日) 18:26:22.39ID:SPrfFCn+
それを言ったら、2つの素数で書ける自然数はN×N個だが自然数全体の中での密度はごく僅かだ。
実数濃度と可算濃度の違いは非構成的な概念を持ち出さないと実感できないと思う。
0093132人目の素数さん
垢版 |
2022/05/08(日) 18:51:26.04ID:WmfJGbb+
V を有限次元複素ベクトル空間とし、 T を V 上の線形変換とする。
T が対角化可能であるための必要十分条件は

V = Ker (T - λ*I) (+) Im (T - λ*I)

が任意の λ ∈ C に対して成り立つことであることを示せ。
0094132人目の素数さん
垢版 |
2022/05/08(日) 18:54:06.77ID:Os1Q4ACv
>>86
>なぜ群論を避けたいの
すいません。
これは私が群論を理解出来ていないためです。

群論の初心者向けテキストなどを尋ねて、証明は参照しているサイトを見るべきでしたかね?
0095132人目の素数さん
垢版 |
2022/05/08(日) 18:58:12.62ID:SPrfFCn+
>>94
有限体は原始根を持つことさえ認めれば、>>85の質問は道具無しで証明できる
0096132人目の素数さん
垢版 |
2022/05/08(日) 19:03:37.88ID:Os1Q4ACv
>>95
群論の考え方と原始根について理解できれば、もうそれだけで終わると言うことですかね。
ご対応ありがとうございました。
勉強します。
0097132人目の素数さん
垢版 |
2022/05/11(水) 11:48:11.15ID:96G/XLjv
U を C 上のベクトル空間とする。

U ∋ x → ||x|| ∈ [0, +∞) をノルムとする。

||x|| = √(<x, x>) を満たすような U 上の内積が存在するための必要十分条件は、

||u + v||^2 + ||u - v||^2 = 2 * (||u||^2 + ||v||^2) が任意の u, v ∈ U に対して成り立つ

ことであることを示せ。
0100132人目の素数さん
垢版 |
2022/05/12(木) 21:06:40.73ID:hcVULNQE
I(n)=d^n/dx^n, [A,B]=AB-BAと定義して
[I(n), x]=n・I(n-1)
となるらしいんですが証明ってできますか?
0102132人目の素数さん
垢版 |
2022/05/12(木) 21:52:14.08ID:I1e7nnfK
>>100
ライプニッツルール
[ln,x]f=ln(xf)-xlnf=(nl(n-1)f+xlnf)-xlnf=nl(n-1)f
0104132人目の素数さん
垢版 |
2022/05/12(木) 22:32:09.24ID:hcVULNQE
なるほどxの2階以上の微分は0だから項が消えるんですね
ありがとうございました
0105132人目の素数さん
垢版 |
2022/05/13(金) 19:15:14.65ID:DXZMOfN+
任意のベクトル空間に基底が存在するという定理の証明が載っている本を松坂和夫著『集合・位相入門』以外で教えてください。
0106132人目の素数さん
垢版 |
2022/05/13(金) 19:28:07.29ID:DXZMOfN+
任意のベクトル空間に基底が存在するという定理の証明が載っている本を松坂和夫著『集合・位相入門』、『解析入門』以外で教えてください。
0107132人目の素数さん
垢版 |
2022/05/13(金) 19:47:00.25ID:DXZMOfN+
Zornの補題の証明も同時に載っている本でお願いします。
0109132人目の素数さん
垢版 |
2022/05/13(金) 21:48:23.03ID:DXZMOfN+
>>108

Zornの補題の証明
http://www.math.pku.edu.cn/teachers/anjp/Zorn.pdf

の最初の箇所について質問です。

以下はあっていますか?

A を X の全順序部分集合とする。
仮定により、 A には上界 b が存在する。
b ∈ A ならば、 b は X の極大元ではないから、 b < c となるような c ∈ X が存在する。
任意の a ∈ A に対して、 a ≦ b < c だから、 a < c が成り立つから c は A の上界である。
c ∈ A ならば、 c ≦ b < c となり矛盾するから、 c ∈ X - A である。

ゆえに、 {c ∈ X - A | c は A の上界である} は空集合ではない。

選出公理により、任意の X の空でない全順序部分集合 A に対して、 A の上界 g(A) ∈ X - A を選出できる。
0110132人目の素数さん
垢版 |
2022/05/13(金) 21:50:16.88ID:DXZMOfN+
訂正します:

>>108

Zornの補題の証明
http://www.math.pku.edu.cn/teachers/anjp/Zorn.pdf

の最初の箇所について質問です。

以下はあっていますか?

A を X の全順序部分集合とする。
仮定により、 A には上界 b が存在する。
b ∈ A ならば、 b は X の極大元ではないから、 b < c となるような c ∈ X が存在する。
任意の a ∈ A に対して、 a ≦ b < c だから、 a < c が成り立つから c は A の上界である。
c ∈ A ならば、 c ≦ b < c となり矛盾するから、 c ∈ X - A である。

ゆえに、 {c ∈ X - A | c は A の上界である} は空集合ではない。

選出公理により、任意の X 全順序部分集合 A に対して、 A の上界 g(A) ∈ X - A を選出できる。
0111132人目の素数さん
垢版 |
2022/05/13(金) 21:59:27.28ID:vvMs26DA
>>109
教えてもらって礼も言わないんだな。
例の頭のおかしい人か。
0112132人目の素数さん
垢版 |
2022/05/14(土) 00:53:01.33ID:svau5T56
微分形式がわかりません
高校生の僕にもわかるように教えて下さい
0113132人目の素数さん
垢版 |
2022/05/14(土) 01:11:45.23ID:5q/uGXBy
座標(x,y)と任意定数cを含む方程式
F(x,y,c)=0…(4)
両辺を微分して
∂F/∂x+∂F/∂y・y'=0…(5)
(4)と(5)から文字cを消去すればx,y,y'を含む方程式
f(x,y,y')=0
が得られる
とあったのですがなぜcを消去できるのですか?
0114132人目の素数さん
垢版 |
2022/05/14(土) 07:15:59.80ID:BsolXhbA
>>113
F(x,y,c)=0を変形して(解いて)f(x,y)=cにしたら両辺微分してf_x+f_yy'=0になるよ
0116132人目の素数さん
垢版 |
2022/05/14(土) 12:25:31.77ID:XI6WBYfZ
陰関数定理の条件満たしていなければもちろんできないですけど、実用的には代替の場合は解けますよね
ある点だけでは解けなくても、その他大勢の場合は解けるという場合も多々あるかと思います
0117132人目の素数さん
垢版 |
2022/05/14(土) 12:33:19.79ID:BsolXhbA
>>115
できる場合はってことだけど?
0118132人目の素数さん
垢版 |
2022/05/14(土) 12:53:39.22ID:BsolXhbA
あるいはF(x,y,c)=0は(x,y,c)∈R^3において(一般には)2次元の曲面を意味していてcを(x,y)の陰関数と考えたのがf(x,y)=cと
具体的にf(x,y)が求まるかどうかはF(x,y,c)に依存する話だけど
陰関数を考えるのは自由だしね
0119132人目の素数さん
垢版 |
2022/05/14(土) 14:53:12.98ID:Q13+IRsT
あるいはF(x,y,c)=0からF_xdx+F_ydy+F_cdc=0でcが(x,y)に関して定数dc=0としたらF_xdx+F_ydy=0からF_x(x,y,c)+F_y(x,y,c)y'=0
これは(x,y,y',c)∈R^4の中でF(x,y,c)=0かつF_x(x,y,c)+F_y(x,y,c)y'=0の(一般には)2次元の曲面Sを表していて
cを固定する毎に(x,y,y')∈R^3の中の(一般には)1次元の曲線なのでc∈Rの全体で(x,y,y')∈R^3の中の(一般には)2次元の曲面
つまりSをR^4→R^3:(x,y,y',c)→(x,y,y')によって射影したものとなるけど
0120132人目の素数さん
垢版 |
2022/05/14(土) 19:20:27.01ID:5q/uGXBy
んー、知識がないのでよく分かりませんでしたすみません😣💦
もう少し勉強します
答えてくださりありがとうございました
0121132人目の素数さん
垢版 |
2022/05/14(土) 22:10:12.29ID:BsolXhbA
>>120
f(x,y)=0がxy平面上の(一般には)曲線を表すこと
曲線上の点の座標x,yの間にはこの条件式で表せる一定の関係があること
関数f(x)=yもやはりx,yの間にfで表せる一定の関係があることを意味するという意味では同じであること
違いはxに対して
yの値をf(x)によって直接表せるか
f(x,y)=0を満たすyと間接的に決まるか
前者を陽関数後者を陰関数
f(x,y)=0はx,yの関係なのでxに対してyと考える代わりにyに対してxと考えても良い
同様に
f(x)=yもx,yの関係としてyに対してxと考えて逆関数と呼ぶ
少し拡張して
F(x,y,z)=0はxyz空間内の(一般には)曲面を表し
この曲面上の点のx,y,z座標にはこの条件で表せる一定の関係がある
f(x,y)=zもx,y,zの間にこの等式で表せる一定の関係があることは同じということ
0122132人目の素数さん
垢版 |
2022/05/15(日) 11:15:36.55ID:ypbJpKxb
>>120
(中略)
SをR^4→R^3で射影した(一般には)2次元の曲面も同じSで表せば
(x,y,z)∈Sであることをf(x,y,z)=0という関係が成立すると捉えることが出来ると
一般に具体的なfが得られるとは限らないけれど
たとえば(x,y,z)∈R^3とSと距離d(x,y,z)を想定して(x,y,z)∈Sをd(x,y,z)=0と
0123132人目の素数さん
垢版 |
2022/05/15(日) 21:31:15.88ID:QNo78cdY
aは実定数、εは正定数、Tは対称作用素とし、
(a-ε,a+ε)⊂ρ(T)のとき、||(a-T)^(-1)||<1/ε となることを示せ、
という問題は示せますか?
0124132人目の素数さん
垢版 |
2022/05/16(月) 15:11:51.54ID:eBPtGkdT
複素変数zの関数w=f(z)が領域Dで正則であるとき、Dの任意の点zで、独立変数zの増分Δzにともなうwの増分をΔwとすれば
Δw=f'(z)Δz+ρ(z,Δz)Δzが成り立つ(ρ(z,Δz)→0 (Δz→0))
このΔw=の式がなぜ成り立つか教えてください
0126132人目の素数さん
垢版 |
2022/05/16(月) 15:38:39.85ID:eBPtGkdT
>>125
Δz→0にすれば微分の定義式になるのは分かるのですが
Δw=f'(z)Δz+ρ(z,Δz)Δz
がΔz→0以外でどうして成り立つかが知りたいです
ちんぷんかんぷんなこと言ってたらすみません!
0127132人目の素数さん
垢版 |
2022/05/16(月) 15:55:56.43ID:LfrQnfV3
>>124
丁寧にかくなら

Δw = f(z+Δz) - f(z)
p(z,Δz) = (f(z+Δz) - f(z))/Δz - f'(z)

と定めれはこのとき

> Δw=f'(z)Δz+ρ(z,Δz)Δz

が成り立っていて、そして

(ρ(z,Δz)→0 (Δz→0))

にもなるって言ってるんだよ
ここまで丁寧に書いてくれる教科書はもう出てこないよ
この程度の行間は自分で埋めんとダメ
0128132人目の素数さん
垢版 |
2022/05/16(月) 16:07:05.19ID:eBPtGkdT
>>127
はー
p(z,Δz) = (f(z+Δz) - f(z))/Δz - f'(z)
ですかなるほど
先が思いやられます…
答えくださった方々ありがとうございます
0129132人目の素数さん
垢版 |
2022/05/16(月) 20:28:46.69ID:G2jAktwR
ビブンケイシキガーさんはこのようなことすら理解できないのでしょうね
かわいそうな方達です
0130132人目の素数さん
垢版 |
2022/05/16(月) 21:02:06.23ID:Hy3YN9ps
どんだけ劣等感強いんだこいつwwwww
0131132人目の素数さん
垢版 |
2022/05/18(水) 21:09:29.73ID:OB38FyAa
ルベーグ積分の定義において、被積分関数が可測という条件はどこで使われているのですか?
つまり、以下の(☆)がなくてもルベーグ積分は定義されると思うのですが、なぜ必要なのでしょうか?

以下、ルベーグ積分の定義


予備定義:

(X, M, μ)測度空間
sを可測単関数とする、すなわち

s = ∑[A∈I] α_A χ_A

ここでIはMの有限部分集合で、Iに属するどの2つの可測集合も共通部分を持たないとする。
α_Aは非負整数、χ_AはAの特性関数(ふなわち、x∈Aなら1、さもなくば0)
このとき、E∈Mに対して、sのE上のルベーグ積分を

∫_E s dμ := Σ[A∈I] α_A μ(A∩E)

で定義する。


定義:

(X, M, μ)を測度空間
「f: X → Rを可測関数」 ………… (☆) とする。
E∈Mに対して、fのE上のルベーグ積分を

∫_E f dμ := sup[s] ∫_E s dμ

で定義する。
ただし、sはEのすべての点xでs(x)≦f(x)となる、可測単関数全体を動く。
0132132人目の素数さん
垢版 |
2022/05/18(水) 21:10:44.61ID:OB38FyAa
ああ、関数は全部非負です
0133132人目の素数さん
垢版 |
2022/05/18(水) 21:14:00.12ID:OB38FyAa
α_Aは非負整数じゃなくて、非負実数です
0134132人目の素数さん
垢版 |
2022/05/18(水) 22:24:55.69ID:W1DwPwl7
>>131
どの文献にあった定義ですか?
0136132人目の素数さん
垢版 |
2022/05/19(木) 04:04:30.37ID:oxyGuqNU
単調収束定理を示すのに必要

(a, ∞]のfによる逆像が可測集合という議論が証明中に出てくる
0137132人目の素数さん
垢版 |
2022/05/19(木) 04:05:21.96ID:oxyGuqNU
可測性の仮定を入れないと、重要な定理のほとんどは示せないと思う
0138132人目の素数さん
垢版 |
2022/05/19(木) 04:07:42.00ID:MePhHF7c
直接数学に関係ないし完全に数学素人の質問なんだけど適切な板とスレが見つからなかったからここで質問させてほしい。

戸田山和久の『科学哲学の冒険』のなかで、あらゆる科学的説明の共通点(条件?)として「反事実的依存関係」が挙げられてるんだけど、その意味するところはある科学上の仮説「PならばQ」の科学的な正しさは「PのときにQ」ということだけでなく「PでないときにはQでない」も成り立ってないといけないらしい。(反事実条件法)
他方で論理学の条件文の説明では、命題「PならばQ」の真偽に関して、前提Pが偽で結論Qが真の場合でも命題「PならばQ」は真であると説明されてるんだよね。
これって戸田山の言ってることと論理学の理屈が矛盾してない?
論理学の方はごく一般的な命題論理の説明だけど。

正直おれが論理学と科学(哲学)の関係をよく理解してないだけなんだろうけど、どうしても気になってしまう。
0139132人目の素数さん
垢版 |
2022/05/19(木) 06:29:31.43ID:bzDA9ULu
Gを群とする
Gの位数とは、Gの濃度のことである

別にこの定義をするのに、Gが群である必要ないが、それと同じようなものだろう
0140132人目の素数さん
垢版 |
2022/05/19(木) 06:43:44.46ID:irjf/OKk
>>138
>科学上の仮説「PならばQ」の科学的な正しさは「PのときにQ」ということだけでなく「PでないときにはQでない」も成り立ってないといけないらしい
「ならば」をどう解釈するか
「科学上の仮説」とかいう概念における
その人の定義が普通と違うってことでは?
勝手にやらせておけばイイと思うよ
0141132人目の素数さん
垢版 |
2022/05/19(木) 06:47:56.79ID:irjf/OKk
なんなら
「科学上の仮説」の場合は「PなばらQ」とでも呼べばいい
そうすれば「なばら」と「ならば」は字面の上でも違うから
意味が違って矛盾してるって悩まなくて良くなる
0142132人目の素数さん
垢版 |
2022/05/19(木) 07:15:03.70ID:Am6ABFt6
>>131
fが或る非可測集合の定義関数である場合、その定義はその集合の内測度を与える。
もちろん加法性はない。
0143132人目の素数さん
垢版 |
2022/05/19(木) 12:39:15.27ID:UcZIJYO0
>>138
日常生活での言葉遣いと論理学の言葉遣いが違うってのと同じような話
昼飯はラーメンまたはカレーを食うって言って両方食ったら変人だと思われるように
科学だとなにか予言できないような仮説は無意味だから
結論が予言できるような形の主張しか取り扱わないってだけ
0144132人目の素数さん
垢版 |
2022/05/19(木) 22:56:45.35ID:MePhHF7c
>>143
本当に基礎的なとこから整理したんだけどこういう理解でいいのかな?
論理学の世界では↓
ニワトリならば卵を産む。ニワトリである、だから卵を産む。(妥当な演繹)
ニワトリならば卵を産む。ニワトリでない、しかし卵を産む。(裏は必ずしも真ならず)
このとき論理学では最初の命題「ニワトリならば卵を産む」は“形式上”偽にならない。

一方、科学的説明(現実の世界)の特徴は↓
「ニワトリならば卵を産む」という仮説の正しさは論理的形式が担保するものじゃないため、ニワトリでないのに卵を産む生物が実際に見つかってしまった場合その仮説は否定されてしまう。

これ、書いてて虚しくなるくらい当たり前なことなんだけど、単純にこれだけの話なのかな。
そうすると形式論理って一体どういう意味があるのか素人の俺にはよくわからない…
0145132人目の素数さん
垢版 |
2022/05/19(木) 23:02:40.39ID:wuA22kz/
過不足なく説明されている必要があるかどうかって話じゃないの?
0146132人目の素数さん
垢版 |
2022/05/19(木) 23:35:43.67ID:irjf/OKk
>>144
>このとき
このときって?
上に書かれてる2つの事柄がどちらも真であるとき?
>“形式上”偽にならない
形式上とは?
偽にならないってそもそも真であるときを考えてたんじゃないの?>「ニワトリならば卵を産む」という仮説の正しさは論理的形式が担保するものじゃないため、ニワトリでないのに卵を産む生物が実際に見つかってしまった場合その仮説は否定されてしまう。
なんで否定しなくちゃいけないの?
ニワトリでなくて卵を産むモノを排除しているようには読めないんだけど

まあいずれにせよどうでもいい話だな
勝手に考えて悩んでたらいいのではないかな
0147132人目の素数さん
垢版 |
2022/05/19(木) 23:43:55.89ID:irjf/OKk
けっきょくその「ならば」は「iff」の意味で使っているってだけではないかな
それならば明確にそう書くべきではないかならば
通常の意味合いと異なる用語の使用ならば不親切だと思うね
0148132人目の素数さん
垢版 |
2022/05/20(金) 00:00:19.77ID:ybkx/Z2m
>>146
このときってのは、「ニワトリでない(前件が偽)、しかし卵を産む(後件が真)」を指してる。形式上って書いたのは一般的な論理学で採用されてる二値原理(すべての命題は真か偽のいずれか)をもとにした条件法上、前件が偽で後件が真のとき元の命題は偽にならない(真になる)っていう原則のことを指してる。

否定のところだけど、まさにその部分と論理学の関係がわからなくて質問したわけよ。
戸田山は「科学的説明すべてに共通してるのは反事実的依存関係(反事実条件法)」だと明確に書いてる。
『科学哲学の冒険』p244
0149132人目の素数さん
垢版 |
2022/05/20(金) 05:20:22.01ID:GqaaZfZr
>>148
>このときってのは、「ニワトリでない(前件が偽)、しかし卵を産む(後件が真)」を指してる
「ニワトリでない」は偽と確定しないし「卵を産む」も真とは鍵らない
「ニワトリではないが卵を産む」とは
「「ニワトリではない」が真であるとき「卵を産む」が真である」という命題のことで
「「ニワトリではない」が偽であるかまたは「卵を産む」が真である」と同値
>否定のところだけど、まさにその部分と論理学の関係がわからなくて質問したわけよ。
このときというのが「ニワトリではないが卵を産む」が真であるという意味だとして
君は「ニワトリは卵を産む」もそもそも真であるとしているのでしょ?
それともこちらは真かどうか分からないと考えていて
「ニワトリではないが卵を産む」が真であることが「ニワトリは卵を産む」が偽であることを導くのではないかと疑問に思っているということ?
いずれにせよ
「ニワトリではないが卵を産む」が真であっても
「ニワトリは卵を産む」の真義には関係ない
なぜなら
後者が真であるとは
「ニワトリである」が偽であるかまたは「卵を産む」が真であるということ
「卵を産む」が真であれば両者とも真だし
「卵を産む」が偽であれば前者が真のとき後者は偽なのでね
>戸田山は「科学的説明すべてに共通してるのは反事実的依存関係(反事実条件法)」だと明確に書いてる。
勝手に言わせておけば?
「ならば」で悩むなら「科学的説明」の場合は「ならば」を「iff」(念のためだけど「if and only if」のことね)の意味で使うとその人は主張していると考えたら?
いずれにせよその主張は正しいと思えないけど別にどうでもいい
0150132人目の素数さん
垢版 |
2022/05/20(金) 05:41:47.12ID:GqaaZfZr
ついでに書くと
日本語では主語を省略することが多いし
量化子も明確に書かないのが普通(こちらは他の言語でもそう)
「ニワトリは卵を産む」はそれらの意味で曖昧
生物xがニワトリであることをP(x)
生物xが卵を産むことをQ(x)
として
「ニワトリは卵を産む」とは「P(x)→Q(x)がすべての生物について真である」
つまり
「∀x:生物(¬P(x)∨Q(x))」が真だってこと
0151132人目の素数さん
垢版 |
2022/05/21(土) 00:54:54.61ID:UeYbwFhj
>>149
ニワトリみたいな具体的な例を出したら逆に伝わりにくくなった気がする。
もう一度整理して疑問を書くわ。

論理学的には、ある命題「PならばQ」が真のとき、当然「裏」が真とは限らない。
しかし戸田山は科学的説明の特徴として、ある仮説(命題)「PならばQ」が真であったとしてもそれだけでは不十分で、「PでないならばQでない」も真である必要(特徴)があるという趣旨のことを主張してる。
しかし、Pが偽でQが真のとき(つまり¬Pが真で¬Qが偽のとき)PならばQは真になる一方で、¬Pならば¬Qは偽になる。
つまりこの場合はPならばQと¬Pならば¬Qは両立せず矛盾し合うから戸田山の言ってることはなんかおかしい気がするってことが言いたかった。

けどそのあと真理表を見て考えたら、Pが真でQが真のときはPならばQも真だし、¬Pならば ¬Qも真なんだよね。戸田山はこのことを指してるのかな。
つまり単に前件も後件もどちらも真でないといけないと言ってるにすぎないってこと?

そうするとおれの疑問は、PならばQが真で、¬Pならば¬Qも真になるときと、PならばQが真で、¬Pならば¬Qが偽のときとの違いがよくわからないってことになるわ。
自然言語で例えて説明してくれたら助かる。
0152132人目の素数さん
垢版 |
2022/05/21(土) 01:29:27.65ID:UeYbwFhj
素人だからレベルの低い質問してることは重々承知なんだけどどうしても気になってしまう
0153132人目の素数さん
垢版 |
2022/05/21(土) 04:58:03.07ID:DyqXYBYq
もう著者に直接ききなよ
0154132人目の素数さん
垢版 |
2022/05/21(土) 05:54:31.64ID:Pr1/8Sjv
>>151
あのね
その人が勝手に言っていることを解釈する手助けを求めているって自分でも分かってるだろうけど
意味ないことを他人に考えさせようとしてるってことだよ
手助けとしては随分前から書いているように
・その人の考えはどうでもいい(おかしくても正しくても別にどうでもイイってこと)
・その人の「ならば」は「iff」の意味に解釈したら?
これだけ
>そうするとおれの疑問は、PならばQが真で、¬Pならば¬Qも真になるときと、PならばQが真で、¬Pならば¬Qが偽のときとの違いがよくわからないってことになるわ。
PQ=真真・真偽・偽真・偽偽
P→Q=¬P∨Q=真・偽・真・真
¬P→¬Q=P∨¬Q=真・真・偽・真
P→Qが真で¬P→¬Qが真なのはPQの真偽が一致しているとき(これがiffね)
P→Qが真で¬P→¬Qが偽なのはPQの真偽が逆であるとき
>自然言語で例えて説明してくれたら助かる。
めんどくさい
0155132人目の素数さん
垢版 |
2022/05/21(土) 05:58:13.96ID:Pr1/8Sjv
>>154
>P→Qが真で¬P→¬Qが偽なのはPQの真偽が逆であるとき
違った
Pが偽でQが真の時だけ
0156132人目の素数さん
垢版 |
2022/05/21(土) 08:14:19.11ID:4/dL5yKq
「階段行列」って言葉は元になった英語の専門用語はあるんですか?
0158132人目の素数さん
垢版 |
2022/05/21(土) 18:25:34.43ID:UeYbwFhj
>>154
同値として解釈するということね
スレチなのにしつこく質問して申し訳なかった
回答ありがとう
0159132人目の素数さん
垢版 |
2022/05/22(日) 16:12:56.23ID:i2b+uo6e
科学的説明に限らず日常の条件のほとんどは裏も含んでると思うよ

60点以上なら合格
には
60点未満なら不合格
の意味も含まれている
0160132人目の素数さん
垢版 |
2022/05/22(日) 17:42:20.04ID:068Pg5W/
>>159
プロスポーツの「明日勝てば優勝」は「明日勝てなかったら優勝できない」を意味しない
0162132人目の素数さん
垢版 |
2022/05/22(日) 19:24:19.29ID:wDKW2spO
>>159
それは建前・・・でね
0163132人目の素数さん
垢版 |
2022/05/22(日) 19:46:56.84ID:068Pg5W/
>>161
「明日雨が降ったら花見は中止」は「明日晴れたら必ず花見をする」を意味しない
0164132人目の素数さん
垢版 |
2022/05/22(日) 21:56:44.74ID:I3pHT75R
>>163
それは意味してる
意味してなきゃ中止って言葉使わない
0165132人目の素数さん
垢版 |
2022/05/22(日) 22:00:16.35ID:068Pg5W/
>>164
それは頭悪いぞ
大地震があっても中止だろ
0166132人目の素数さん
垢版 |
2022/05/22(日) 23:04:02.35ID:fyEUnEGM
天候の話で地震を持ち出すのは頭が悪いからなの?
そんなん確実なことが何一つ言えなくなるわ
0168132人目の素数さん
垢版 |
2022/05/23(月) 05:22:08.30ID:IGwAoT6e
>>166
日常の話だよ?天気に限定する必要ある?
0169132人目の素数さん
垢版 |
2022/05/23(月) 06:00:32.96ID:PcnuWTCp
なるほどこれが学部レベルの質問ですか
0170132人目の素数さん
垢版 |
2022/05/23(月) 06:12:04.33ID:IGwAoT6e
>>164>>166は何がおかしいかというと
何かになるための条件をすべて枚挙できることなどごく限られた事象でしかないのに、「裏も言える」なんて簡単に考えている点
0171132人目の素数さん
垢版 |
2022/05/23(月) 06:26:41.52ID:3SPmHOq2
>>170
それ言い出したら表も言えるかどうか分からんようになるが?
0172132人目の素数さん
垢版 |
2022/05/23(月) 06:56:39.03ID:IGwAoT6e
>>171
戸田山はそこまで主張しているように思える。
私は戸田山の主張はおかしいと思う。
0173132人目の素数さん
垢版 |
2022/05/24(火) 07:43:53.81ID:kSKnLqqI
行けたら行く
0174132人目の素数さん
垢版 |
2022/05/24(火) 12:36:45.25ID:kikZAvtQ
>>161
スポーツは科学ではないという命題には
スポーツでないものは科学であるということも含んでいる!
0175132人目の素数さん
垢版 |
2022/05/24(火) 17:24:49.15ID:smOjadws
ヒルベルト流ならぬ戸田山流
0176132人目の素数さん
垢版 |
2022/05/24(火) 20:31:40.66ID:CuQTUJu7
>>138
1+1=0ならば明日は日が昇る は仮説として正しくないから排除って話では?
0179132人目の素数さん
垢版 |
2022/05/25(水) 18:20:44.35ID:prrGxe18
ポアソン分布が参考書読んでもよく分かりません。
簡単に説明してもらえないでしょうか?
0180132人目の素数さん
垢版 |
2022/05/25(水) 19:03:02.20ID:UeyUaLnB
二項分布で成功確率pと試行回数nとの積が一定の下でpを0に近づけたもの
0181132人目の素数さん
垢版 |
2022/05/25(水) 19:51:52.04ID:YTcx5TUW
>>180
>成功確率pと試行回数nとの積が一定の下で
npは平均
すなわちその事象が平均何回起こるかを固定して
試行回数すなわちその事象が起こりえる機会を無限に増やした極限
0182132人目の素数さん
垢版 |
2022/05/25(水) 19:55:14.42ID:YTcx5TUW
p→0はイメージしにくいからn→∞の方が良くない?
0183132人目の素数さん
垢版 |
2022/05/26(木) 14:17:15.78ID:41PCNX0p
(1+x^2)^aをweightに持つ直交多項式を探してるんですが、ご存知の方いますか?

(1+x)^aならJacobiでいいんですけど。
0184132人目の素数さん
垢版 |
2022/05/26(木) 14:23:13.59ID:6bzT+Nka
>>920
3点取られても同点で後攻、4点取られたら逆転される
ヒットアンドロールでもフリーズしたりうまく隠れたら1点か2点に抑える可能性はあった
0185132人目の素数さん
垢版 |
2022/05/26(木) 14:23:31.36ID:6bzT+Nka
>>184
すれ違いでした
0187132人目の素数さん
垢版 |
2022/05/26(木) 15:47:30.04ID:41PCNX0p
>>186
失礼しました
[-∞, ∞]です。
0188132人目の素数さん
垢版 |
2022/05/26(木) 15:48:41.85ID:41PCNX0p
訂正
(-∞, ∞)です。
0189132人目の素数さん
垢版 |
2022/05/26(木) 16:05:21.31ID:PV2MRVLr
>>183
その区間だと多項式ではめっちゃ小さい空間しかはれないのでは?
∫[-∞,∞] Pm(x)Pn(x)/(1+x^2)^a dx
が内積とするとn次式がL^2に入るのはn^2<2a+1のときでヒルベルト空間のほんのちょっとにしかならない
0190132人目の素数さん
垢版 |
2022/05/26(木) 16:26:29.65ID:41PCNX0p
>>189
なるほど。たしかにそうですね。
問題を勘違いしてるかもしれないです。

指摘ありがとうございます
0191132人目の素数さん
垢版 |
2022/05/27(金) 13:07:28.59ID:LFUyaxJt
>>181
なるほど

ありがとうございます
0192132人目の素数さん
垢版 |
2022/05/27(金) 21:10:36.27ID:sj0+OOtS
たまに使用するt検定ですが、その式の証明を高校数学レベルから理解したいのですが、
なかなか、高校数学レベルで理解できるような証明が載っているページが見つかりません、
あれば教えてください、本でもよいです。
0193132人目の素数さん
垢版 |
2022/05/28(土) 01:22:49.69ID:FfsR3PI5
>>192
>その式の証明を
その式とは?
0194132人目の素数さん
垢版 |
2022/05/29(日) 02:15:38.92ID:XfHRDDXZ
可換環Rのイデアル I1,・・・,Ir(r≧2)について、
環準同型φ:R→R/I1×・・・×R/Ir が全射であることを仮定すると、
i ≠jのとき、Ii+Ij=Rが成立することを示せ.
という問題で次のように考えたのですが合っていますかね?
φが全射なので
x+Ii=1+Ii、x+Ij=1+1+Ij
となるx∈Rが存在
x=1+xi、x=1+1+xj (xi∈Ii、xj∈Ij)
1+xi=1+1+xj
xi-xj=1
よって1∈Ii+IjでIi+Ij=R
0195132人目の素数さん
垢版 |
2022/05/29(日) 02:37:42.73ID:PhcQXkS0
あってるけどやや無駄

x ≡ 1 ( mod Ii)
x ≡ 0 ( mod Ij ) ( ←≡2はあまり意味ない)

でいい
1-x∈Ii、x∈Ijなので
1 = (1 - x) + x ∈ Ii + Ij
0197132人目の素数さん
垢版 |
2022/05/29(日) 18:28:55.22ID:wJLTXUFU
V, W を体 K 上のベクトル空間とする。
体 K 上のベクトル空間 U_0 と V ? W から U への双線形写像 f の組で以下の性質を満たすものが存在する。

U を K 上のベクトル空間とし、 g を V ? W から U への双線形写像とすると、
U_0 から U への線形写像 G で、 G ・ f = g を満たすものが一意的に存在する。  


ここまではいいのですが、

↑の U_0 と f の組 (U_0, f) が本質的に一つしか存在しないということも書いてあります。
T_0 と h の組 (T_0, h) も (U_0, f) と同じ性質を持つとします。
なぜ、これらを同一視できるのでしょうか?
0198132人目の素数さん
垢版 |
2022/05/29(日) 18:29:38.59ID:wJLTXUFU
V, W を体 K 上のベクトル空間とする。
体 K 上のベクトル空間 U_0 と V ? W から U への双線形写像 f の組で以下の性質を満たすものが存在する。

U を K 上のベクトル空間とし、 g を V ? W から U への双線形写像とすると、
U_0 から U への線形写像 G で、 G ・ f = g を満たすものが一意的に存在する。  


ここまではいいのですが、

↑の U_0 と f の組 (U_0, f) が本質的に一つしか存在しないということも書いてあります。
T_0 と h の組 (T_0, h) も (U_0, f) と同じ性質を持つとします。
なぜ、これらを同一視できるのでしょうか?
0199132人目の素数さん
垢版 |
2022/05/29(日) 18:30:44.64ID:wJLTXUFU
V, W を体 K 上のベクトル空間とする。
体 K 上のベクトル空間 U_0 と V × W から U への双線形写像 f の組で以下の性質を満たすものが存在する。

U を K 上のベクトル空間とし、 g を V × W から U への双線形写像とすると、
U_0 から U への線形写像 G で、 G ・ f = g を満たすものが一意的に存在する。  


ここまではいいのですが、

↑の U_0 と f の組 (U_0, f) が本質的に一つしか存在しないということも書いてあります。
T_0 と h の組 (T_0, h) も (U_0, f) と同じ性質を持つとします。
なぜ、これらを同一視できるのでしょうか?
0200132人目の素数さん
垢版 |
2022/05/29(日) 18:31:23.51ID:wJLTXUFU
訂正します:

V, W を体 K 上のベクトル空間とする。
体 K 上のベクトル空間 U_0 と V × W から U_0 への双線形写像 f の組で以下の性質を満たすものが存在する。

U を K 上のベクトル空間とし、 g を V × W から U への双線形写像とすると、
U_0 から U への線形写像 G で、 G ・ f = g を満たすものが一意的に存在する。  


ここまではいいのですが、

↑の U_0 と f の組 (U_0, f) が本質的に一つしか存在しないということも書いてあります。
T_0 と h の組 (T_0, h) も (U_0, f) と同じ性質を持つとします。
なぜ、これらを同一視できるのでしょうか?
0201132人目の素数さん
垢版 |
2022/05/29(日) 19:14:06.97ID:wJLTXUFU
あ、分かりました。
0204132人目の素数さん
垢版 |
2022/05/31(火) 18:46:18.77ID:9pZbMVxh
Bruce C. Berndt "Ramanujan's Notebooks Part II" p.261
0206132人目の素数さん
垢版 |
2022/06/01(水) 12:15:11.57ID:LaJpzWDH
複素平面上のzを原点を中心に反時計回りに120度回転した点をz1 , 240度回転した点をz2とする.
任意のzに対してf(z)+f(z1)+f(z2)=0を満たす正則関数の非自明な例を挙げてください
0207132人目の素数さん
垢版 |
2022/06/01(水) 17:01:55.92ID:bE5ip8Y1
齋藤正彦著『線型代数入門』

p.234 「実係数の多項式は、実数の範囲内で、一次式および二次式の積に分解される。」

この命題の証明に不備がありますね。

α を f(x) の虚根としたとき、

(x - α)^m * (x - conj(α))^n (m ≠ n) となる可能性がありますが、はじめからその可能性を考えていません。
そういうことが起こらないことも証明しなければならないのに。
0208132人目の素数さん
垢版 |
2022/06/01(水) 17:04:40.24ID:bE5ip8Y1
訂正します:

齋藤正彦著『線型代数入門』

p.234 「実係数の多項式は、実数の範囲内で、一次式および二次式の積に分解される。」

この命題の証明に不備がありますね。

α を f(x) の虚根としたとき、

(x - α)^m | f(x) が成り立ち、 (x - α)^(m+1) | f(x) が成り立たない。
(x - conj(α))^n | f(x) が成り立ち、 (x - conj(α))^(n+1) | f(x) が成り立たない。
m ≠ n である。

このような可能性がありますが、はじめからその可能性を考えていません。
そういうことが起こらないことも証明しなければならないのに。
0209132人目の素数さん
垢版 |
2022/06/01(水) 17:20:52.69ID:bE5ip8Y1
h(x) = (x - α) * (x - conj(α)) とおいて、

f(x) を h(x) で割ります。

f(x) = h(x) * g(x) + a * x + b

a, b は実数です。

a * α + b =0
a * conj(α) + b = 0

もしも、 a ≠ 0 ならば、 α = conj(α) となって矛盾。

よって、 a = b = 0
0212132人目の素数さん
垢版 |
2022/06/01(水) 22:28:49.76ID:gCfFlVL8

自明だからね
0213132人目の素数さん
垢版 |
2022/06/01(水) 22:54:21.53ID:bE5ip8Y1
因数定理から、

f(x) = (x - α) * (x - conj(α)) * g_1(x)

を満たす複素多項式 g_1(x) が存在する。

g_1(x) は実は実多項式であるが、実多項式であることは本当に自明なことであろうか?
0214132人目の素数さん
垢版 |
2022/06/01(水) 22:55:17.85ID:bE5ip8Y1
実多項式 × 実でない複素多項式 = 実多項式

が成り立たないことは明らかといえば明らかかもしれません↓

実でない複素多項式の実でない係数のうち最低次(= k とする)の係数を a とする。

実多項式 × 実でない複素多項式 の k 次の係数は明らかに実でない複素数。
0215132人目の素数さん
垢版 |
2022/06/01(水) 22:59:52.16ID:bE5ip8Y1
h(x) = (x - α) * (x - conj(α)) とおいて、

f(x) を h(x) を実数の範囲内で割ります。

f(x) = h(x) * g(x) + a * x + b

h(x), g(x) は実多項式です。
a, b は実数です。

a * α + b =0
a * conj(α) + b = 0

もしも、 a ≠ 0 ならば、 α = conj(α) となって矛盾。

よって、 a = b = 0

よって、 f(x) = h(x) * g(x) が実数の範囲内で成り立つ。

α が g(x) の根ならば、上と同様にして、

g(x) = h(x) * g_2(x) と実数の範囲内で書ける。

これを繰り返せば、

f(x) = h(x)^n * g_k(x), (α は g_k(x) の根ではない)

と書ける。

もちろん、 conj(α) も g_k(x) の根ではない。

以上から、以下の可能性を排除できた。

(x - α)^m | f(x) が成り立ち、 (x - α)^(m+1) | f(x) が成り立たない。
(x - conj(α))^n | f(x) が成り立ち、 (x - conj(α))^(n+1) | f(x) が成り立たない。
m ≠ n である。
0216132人目の素数さん
垢版 |
2022/06/01(水) 23:04:08.34ID:bE5ip8Y1
齋藤正彦さんは、 f(x) の根 α が n 重根ならば、 conj(α) も n 重根であることを証明することなしに認めたということですね。
0217132人目の素数さん
垢版 |
2022/06/01(水) 23:13:20.39ID:A2jjx+Ao
>>208
f(x)は実係数多項式なので
(x - α)^n | f(x) ⇔(x - conj(α))^n | f(x)
となり自明
0218132人目の素数さん
垢版 |
2022/06/01(水) 23:14:50.87ID:bE5ip8Y1
>>271

f(x)は実係数多項式なので
(x - α)^n | f(x) ⇔(x - conj(α))^n | f(x)

これを証明してください。
0219132人目の素数さん
垢版 |
2022/06/01(水) 23:52:28.78ID:bE5ip8Y1
>>217

あ、分かりました。
0220132人目の素数さん
垢版 |
2022/06/01(水) 23:56:55.31ID:A2jjx+Ao
斎藤正彦にごめんなさいは?
0221132人目の素数さん
垢版 |
2022/06/02(木) 00:02:09.38ID:LAeU8LUx
f(x) = f(conj(conj(x))) = conj(f(conj(x)))

から

f(x)は実係数多項式なので
(x - α)^n | f(x) ⇔(x - conj(α))^n | f(x)

は明らかですね。
0222132人目の素数さん
垢版 |
2022/06/02(木) 00:04:27.93ID:LAeU8LUx
>>220

いや、齋藤さんは

>>221

のような考察なしに、ただ成り立つに違いないと思いこんでいただけだと推測します。
0223132人目の素数さん
垢版 |
2022/06/02(木) 00:14:09.47ID:/twmlc/F
>>222
推測で謝らないとか馬鹿か?
0224132人目の素数さん
垢版 |
2022/06/02(木) 00:22:33.34ID:GnbQTlle
>>221が「考察」か……正に馬鹿の考え休むに似たりという言葉がぴったり
0225132人目の素数さん
垢版 |
2022/06/02(木) 03:01:58.47ID:3gKNBofh
てか
実係数の方程式P(x)=0がx=αを解に持てばx=α̅も解である
は高校の教科書ですら定理として紹介されとるやろ
0226132人目の素数さん
垢版 |
2022/06/02(木) 07:36:23.34ID:LAeU8LUx
>>225

そして問題になるのが、それらの重複度が一致するか否かということです。
0228132人目の素数さん
垢版 |
2022/06/02(木) 07:44:23.90ID:LAeU8LUx
佐武一郎さんの本も見てみました。

f(x) = (x-α) * (x - conj(α)) * f_1(x) と分解できる。
f_1(x) に対して同様の操作をくり返せばよい。

という説明です。

f(x) = (x-α) * (x - conj(α)) * f_1(x) と複素多項式の範囲内で分解できる。
f_1(x) に対して f(x) に対するのと同様の操作を適用するには、 f_1(x) が実多項式であることを示す必要があります。
佐武さんはこのことを自明と考えたのでしょうか?

実多項式 = 実多項式 × 複素多項式

と分解されることがないのはそんなに自明なことでしょうか?
0229132人目の素数さん
垢版 |
2022/06/02(木) 07:46:14.72ID:4aq5yLCN
p(x)が実係数で(x-α)(x-α̅)で割った商も実係数になる事がもう高校の教科書に載ってる、というより高校の教科書では実係数の多項式しか扱うことすらないわバーカ
0230132人目の素数さん
垢版 |
2022/06/02(木) 07:49:42.77ID:LAeU8LUx
実多項式 = 実多項式 × 複素多項式

これはいかにも成り立たないように見える等式です。
無反省に成り立たないと思う人が多いはずです。
ですが、証明が必要なのではないでしょうか?
もしかしたら、うまく右辺の複素係数の項がすべて消えてしまうかもしれません。

証明は以下のようにできます:

実でない複素多項式 a_0 + a_1*x + … + a_n * x^n の実でない係数のうちインデックスが最小のものを a_k とします。

実多項式 × 実でない複素多項式 の k 次の係数は実でない複素数なります。

これは右辺が実多項式であることに矛盾します。
0231132人目の素数さん
垢版 |
2022/06/02(木) 07:51:24.35ID:LAeU8LUx
訂正します:

実多項式 = 実多項式 × 複素多項式

これはいかにも成り立たないように見える等式です。
無反省に成り立たないと思う人が多いはずです。
ですが、証明が必要なのではないでしょうか?
もしかしたら、うまく右辺の複素係数の項がすべて消えてしまうかもしれません。

証明は以下のようにできます:

実でない複素多項式 a_0 + a_1*x + … + a_n * x^n の実でない係数のうちインデックスが最小のものを a_k とします。

実多項式 × 実でない複素多項式 の k 次の係数は実でない複素数になります。

これは左辺が実多項式であることに矛盾します。
0232132人目の素数さん
垢版 |
2022/06/02(木) 07:53:32.52ID:LAeU8LUx
>>229

高校の教科書ではどのように証明しているのでしょうか?
0233132人目の素数さん
垢版 |
2022/06/02(木) 07:53:40.26ID:4aq5yLCN
>>230
だから本来そんなもん高校数学の時点で
「なんでA(x) = B(x)Q(x) + R(x)のQ(x)とかR(x)となる実係数の整式がとれるんだろう?」
と思ってなかったアンポンタンだからだよ能無し君
0234132人目の素数さん
垢版 |
2022/06/02(木) 07:54:56.21ID:/twmlc/F
>>226
だからそれが>>217
0235132人目の素数さん
垢版 |
2022/06/02(木) 08:04:35.14ID:/twmlc/F
>>230
多項式g(x)に対し、各項の係数をその複素共役にしたものをconj(g)(x)と書くと
conj(g)(conjx)=conj(g(x))なのと
実係数多項式f(x)に対してconj(f)(x)=f(x)であることからわかる
0236132人目の素数さん
垢版 |
2022/06/02(木) 08:16:46.29ID:LAeU8LUx
>>235

高校の教科書でもそのような証明をしているのでしょうか?
0237132人目の素数さん
垢版 |
2022/06/02(木) 08:22:58.66ID:/twmlc/F
それは知らない
本題ではない
0238132人目の素数さん
垢版 |
2022/06/02(木) 11:25:04.59ID:/twmlc/F
>>235
私もちゃんとわかってなかった。重複度についてはこれでは駄目だな。
conj(fg)=conj(f)conj(g)がわかればよくて、これは自明。
0241132人目の素数さん
垢版 |
2022/06/02(木) 21:14:29.89ID:PfjoXJSN
https://oshiete.goo.ne.jp/qa/12973611.html
この質問に対する回答者の主張が全体的によく分からないんですけど、
AなのはBだけである
という予想については、Bの要素でAとなるものが存在することが必須である、というのが大学数学では一般的なのでしょうか?

具体的な問題への質問ではないのですが、教えて下さい
よろしくお願いします
0242132人目の素数さん
垢版 |
2022/06/02(木) 21:51:44.36ID:/twmlc/F
ma-kun....love....という質問者がおかしな人みたいだな
0243132人目の素数さん
垢版 |
2022/06/02(木) 21:53:29.28ID:/twmlc/F
「回答へのお礼」を見たら完全に狂人だった
0244132人目の素数さん
垢版 |
2022/06/02(木) 22:02:15.06ID:uYvG3wTi
BlockedOldMan という回答者の方がおかしいな
0245132人目の素数さん
垢版 |
2022/06/02(木) 22:08:46.03ID:PfjoXJSN
すみません、理由もお願いします
0246132人目の素数さん
垢版 |
2022/06/02(木) 22:57:36.53ID:PdD/i1lw
>>244
>BlockedOldMan という回答者の方がおかしいな
そっかな
別におかしく思わなかったが
質問者の人は質問したいことをこねくりすぎて無用な文章付けすぎかなとは思った
0247132人目の素数さん
垢版 |
2022/06/02(木) 23:00:32.58ID:PdD/i1lw
>>245
回答者の主張を忖度無しに知りたいなら直接本人に聞いたらどうかな
その質問と回答とは面倒くさくてどうでも良いけど
>>241
>AなのはBだけである
>という予想については、Bの要素でAとなるものが存在することが必須である、というのが大学数学では一般的なのでしょうか?
君がこれだけ質問したのなら答えたくなる人も多かったはず
0249132人目の素数さん
垢版 |
2022/06/02(木) 23:10:38.78ID:y5qBYrnK
ふと気になったことがあるのですが教えて下さい

例えば
AなのはBだけである
という予想があったとします
このときこの予想には、Bの要素でAとなるものが存在することが必須である、というのが大学数学では一般的なのでしょうか?

具体的な問題への質問ではないのですが、よろしくお願いします
0250132人目の素数さん
垢版 |
2022/06/02(木) 23:19:47.18ID:/twmlc/F
そもそもAとなる事があるかないか解っていないこともあるが、その場合はまず「Aとなる事があり得る」という予想が立てられる。
0251132人目の素数さん
垢版 |
2022/06/02(木) 23:26:17.23ID:U4L/+60k

ハア?ww

何言ってんだコイツwwwww
0252132人目の素数さん
垢版 |
2022/06/02(木) 23:32:07.96ID:fKOqvty6
>リーマンゼータなら、実軸 と 実部=1/2 上に
>実際に零点を持つため、どちらの「だけ」でも
>文の意味は違わないのですが。


回答者の理屈が謎すぎる。
0253132人目の素数さん
垢版 |
2022/06/02(木) 23:35:26.94ID:/twmlc/F
何を言ってるかと言うと、Aとなることがなければ「AなのはBだけである」という命題は真だけどBの要素でAとなるものはない、ということ。
0254132人目の素数さん
垢版 |
2022/06/02(木) 23:53:17.77ID:/twmlc/F
2^n+1が素数なのはnが2の巾乗の時だけである(これは正しい)
したがって「nが33以上のとき2^n+1が素数なのはnが2の巾乗の時だけである」も正しい
しかしnが6以上のとき2^(2^n)+1の形の素数が存在するかどうかは未解決である
0255132人目の素数さん
垢版 |
2022/06/03(金) 01:12:24.25ID:4jOZ8hqh
そもそも非自明零点無限にあるんだからRHが正しければre=1/2上に無限に零点あるわな
0256132人目の素数さん
垢版 |
2022/06/03(金) 05:21:10.64ID:eh04e4rm
>>254続き
予想として提出するならば
「nが33以上のとき2^n+1が素数なのはnが2の巾乗の時だけである」という形ではなく
「nが33以上のとき2^n+1の形の素数が存在する」
または(こちらが普通)「nが6以上のとき2^(2^n)+1の形の素数が存在する」という形になる
0257132人目の素数さん
垢版 |
2022/06/03(金) 07:27:41.45ID:C4qtS3Tw
質問者が言ってるのは
2^(2^n)+1の形の素数が存在するのはn<10^100だけである
みたいな予想じゃない?

そして回答者が言ってる
n<10^100に2^(2^n)+1の形の素数が存在する
とか、それによって だけ の意味が変わるとかは
私にも理解出来ない
0258132人目の素数さん
垢版 |
2022/06/03(金) 18:25:29.45ID:IU/y62w/
直観主義論理では帰納法が成立しないと書いてる人がいたのですが、帰納法は成立しますよね?
0259132人目の素数さん
垢版 |
2022/06/03(金) 21:23:01.07ID:IrQnsvNK
((P → R) ∧ (Q → R)) → (﹁R →﹁ P ∧﹁ Q)


この論理式の証明図を誰か書いてくれませんか?
0260132人目の素数さん
垢版 |
2022/06/03(金) 21:47:42.44ID:TVg9Gppq
>>258
なんで?
0261132人目の素数さん
垢版 |
2022/06/03(金) 21:57:17.64ID:TVg9Gppq
>>259
最後の一つ前が
¬R→¬P∧¬Q
てことはその一つ前が
¬P∧¬Q
てことはその一つ前が
¬Pと¬Q
てことはその一つ前がどちらも人
その仮定はそれぞれがPとQ
Pから人を出すには
P→RのMPと¬Rを使えば良い
Qからも同様
もうわかりましたね?
0262132人目の素数さん
垢版 |
2022/06/03(金) 22:16:16.81ID:k5M3T03B
1,((P → R) ∧ (Q → R)) ・・・・仮定
2,﹁R・・・・・・・・・・・・・仮定
3,P・・・・・・・・・・・・・・仮定
4,P → R・・・・・・・・・・・・1より∧除去
5,R・・・・・・・・・・・・・・3,4より→除去
6、矛盾・・・・・・・・・・・・・2と5より
7,¬P・・・・・・・・・・・・・3と6より¬導入
8,Q・・・・・・・・・・・・・・仮定
9,Q → R・・・・・・・・・・・・1より∧除去
10,R・・・・・・・・・・・・・・8と9より→除去
11,矛盾・・・・・・・・・・・・10と2より
12,¬Q・・・・・・・・・・・・・8と11より¬導入
13,﹁ P ∧﹁ Q・・・・・・・・・・7と12より∧導入
14,﹁R→(﹁ P ∧﹁ Q)・・・・・2と13より→導入
15,((P → R) ∧ (Q → R))→(﹁R→(﹁ P ∧﹁ Q))・・・1と14より→導入
0263132人目の素数さん
垢版 |
2022/06/03(金) 23:07:56.98ID:lGR4JKZh
6の矛盾も11の矛盾もnot Rのやつですか?
0264132人目の素数さん
垢版 |
2022/06/03(金) 23:11:02.45ID:lGR4JKZh
ごめんなさい自分でも何言ってるか分からなかった
0265132人目の素数さん
垢版 |
2022/06/03(金) 23:19:39.13ID:d5fRiSJT
>>262
()がつく仮定の番号が分からないのですが何でしょうか
0266132人目の素数さん
垢版 |
2022/06/03(金) 23:27:00.79ID:iEs4o4/V
((P → R) ∧ (Q → R))が仮定1

﹁ Rが仮定2

Pが仮定3

Qが仮定4

で合ってますか?
0267132人目の素数さん
垢版 |
2022/06/03(金) 23:33:34.80ID:IU/y62w/
>>260
なんで?というのはつまり必ずしも成立しないという意味ですか?
成立するという言葉がまずかったのかな
直観主義論理は排中律を受け入れないだけで、帰納法を受け入れないわけではないと理解してるのですが、これは間違ってますか?
0268132人目の素数さん
垢版 |
2022/06/04(土) 03:51:33.06ID:RI91PHM7
>>267
俺の知ってる帰納法の証明だと排中律ていうか背理法使ってるけどな
帰納法
P(1)∧∀n∈N(P(n)→P(n+1))→∀n∈N(P(n))
0269132人目の素数さん
垢版 |
2022/06/04(土) 04:02:17.59ID:ULi2R9Rk
変数 x1, x2,x3, ... すべて0以上の整数とする。

今、 zの目標値を
・ z_target = z_t = 1240
とする。
また、 zの実効値 z_effective = z_eff を以下のように変数x の組であらわす。
・ z_eff = A*x1 + B*x2 + C*x3.......

・z_eff は z_t よりも大きくなければならない。
・また、そのうちでもっとも小さいzの値をzの実効上の最適値とする。

Q.1 z_eff = 120*x1 + 180*x2
この時、実効上の最適値 z_eff の値と その時の (x1,x2) を求めよ。

Q.2 z_eff = 120*x1 + 180*x2 + 211*x3
この時、実効上の最適値 z_eff の値と その時の (x1,x2,x3) を求めよ。

補足: Q.1 は高校レベル? の1変数の操作にして解けると思うけど
Q2 は多分、高校レベルじゃ無理ぽ!?
0270132人目の素数さん
垢版 |
2022/06/04(土) 04:20:25.24ID:RI91PHM7
証明
P(1)∧∀n∈N(P(n)→P(n+1))∧∃n∈N(¬P(n))→人
0271132人目の素数さん
垢版 |
2022/06/04(土) 07:16:10.75ID:j+0F4bbH
>>270
言われてみるとたしかに…
自然数全体の集合Nに関する命題P(n)について、帰納法は全てP(n)か¬P(n)かを言ってることになりますね…

Curry-Howard同型対応の文献で、直観主義一階述語論理を帰納法が成立するよう拡張する部分の説明を勘違いしました
つまり帰納法が成立するのはあくまで拡張であって、本来の直観主義論理ではないということですか
0273132人目の素数さん
垢版 |
2022/06/04(土) 08:12:01.17ID:GOI5W0c/
完全に構成的なら限定算術なんて分野は存在しなくなるよな
0274132人目の素数さん
垢版 |
2022/06/04(土) 10:20:30.97ID:RI91PHM7
>>272
対偶法
(¬Q→¬P)→(P→Q)
証明
P∧¬Q∧(¬Q→¬P)→P∧¬P→人
P∧(¬Q→¬P)→¬¬Q→Q
(¬Q→¬P)→(P→Q)
0275132人目の素数さん
垢版 |
2022/06/04(土) 10:28:25.80ID:RI91PHM7
>>271
P(1)∧∀n∈N(P(n)→P(n+1))∧∃n∈N(¬P(n))→人
P(1)∧∀n∈N(P(n)→P(n+1))→¬∃n∈N(¬P(n))→∀n∈N(¬¬P(n))
(DNE)→∀n∈N(P(n))
と思ったのだが
無限の証明を許す無限論理だと
P(1)∧(P(1)→P(2))∧(P(2)→P(3))∧…∧(P(n)→P(n+1))∧…→P(1)∧P(2)∧P(3)∧…∧P(n)∧…
でいいのかも?
0276132人目の素数さん
垢版 |
2022/06/04(土) 10:29:35.08ID:RI91PHM7
>>271
>本来の直観主義論理ではない
ブラウワーはおそるらく無限に長い証明は駄目って宣うんじゃなかろかなあ
0277132人目の素数さん
垢版 |
2022/06/04(土) 10:32:41.47ID:RI91PHM7
あと
>>275
>¬∃n∈N(¬P(n))→∀n∈N(¬¬P(n))
のところ
¬(P∨Q)→¬P∧¬Q
からきてるけど
もしかしたらたしかこれも排中律必要だったとかも?
0278132人目の素数さん
垢版 |
2022/06/04(土) 10:37:21.63ID:RI91PHM7
>>277
>¬(P∨Q)→¬P∧¬Q
P∧¬(P∨Q)→(P∨Q)∧¬(P∨Q)→人
¬(P∨Q)→¬P
Q∧¬(P∨Q)→(P∨Q)∧¬(P∨Q)→人
¬(P∨Q)→¬Q
¬(P∨Q)→¬P∧¬Q
ここはいらんかったわ
0280132人目の素数さん
垢版 |
2022/06/04(土) 10:59:30.01ID:RI91PHM7
>>279
>>275
分からなかったらイイヤ
0282132人目の素数さん
垢版 |
2022/06/04(土) 11:30:24.96ID:RI91PHM7
>>281
ZFで先にNを作っておいてそれが公理を満たすって流れなのでは?
0283132人目の素数さん
垢版 |
2022/06/04(土) 12:05:14.89ID:zUPP7e2C
リーマンの再配列定理(条件収束数列は和交換で任意の値に収束させられる)がありますが
リーマンゼータにでてくる、収束しない級数も任意の値に収束させられるとおもうのですが違いますか? 解析接続でつかう関数を別にすれば
リーマンゼータは物理でも使われますが、
リーマンゼータがたまたま良く研究されてたので、物理で使われたのか、リーマンゼータで計算した値が正しいからこれなんですか?


1+2+3+4+… - Wikipedia
一見するとこの級数が意味のある値を持つことは全くないように思われるが、これに数学的に意味のある値を結びつける方法があり、そうして得られた値は複素解析や、物理学における場の量子論、特に弦理論などの分野において応用がある。
様々な総和法を用いることで、上記のごとき発散級数にさえ有限な数値を割り当てることができ、特にゼータ関数正規化やラマヌジャン総和法では-1/12 を値として割り当てる。
0284132人目の素数さん
垢版 |
2022/06/04(土) 12:48:44.44ID:kfulOg92
>>282
ZFの中で自然数作ってそれがペアノ算術の公理満たすって話はならそりゃそうだけど、それならそれで>>270は証明になってないのでは?
0285132人目の素数さん
垢版 |
2022/06/04(土) 13:01:50.58ID:RI91PHM7
>>284
NがP(1)∧∀n∈N(P(n)→P(n+1))→∀n∈N(P(n))を満たすことを示すには
必然的に
P(1)∧∀n∈N(P(n)→P(n+1))∧∃n∈N(¬P(n))→人
を示すことになるんじゃないの?
でそこではDNE使うと思うので直観主義論理じゃだめじゃないかってことだけど
0286132人目の素数さん
垢版 |
2022/06/04(土) 13:11:00.23ID:RI91PHM7
ZFで作ったN={0,1,2,…}だと
部分集合(B)に最小値(∩B)が存在することと
0以外のnに前者(n=m∪{m})が存在することが証明できて
それが
P(1)∧∀n∈N(P(n)→P(n+1))∧∃n∈N(¬P(n))→人
を示すことになるて流れで
0287132人目の素数さん
垢版 |
2022/06/04(土) 13:12:14.25ID:kfulOg92
>>285
イヤ、公理的集合論の中でのωの定義はa≦bを
a≦b :⇔ a=b or a∈b
で定義して
aが順序数:⇔(a,≦)がwell ordered set
と定義してωの定義は最小の無限順序数として定義する
このωぎペアノの公理を満たすのはφが
φ(0)∧φ(x)→φ(x+1)
を満たす述語のとき

w' = { x∈ω | φ(x) }
とすればω'は順序数かつ無限集合であるからωの最小性よりω=ω'

なんじゃないの?
0288132人目の素数さん
垢版 |
2022/06/04(土) 13:13:15.34ID:RI91PHM7
ああこちらのNはN={1,2,3,…}
ZFのNと0が入る入らないが違う
0289132人目の素数さん
垢版 |
2022/06/04(土) 14:44:14.47ID:j+0F4bbH
>>287
>w' = { x∈ω | φ(x) }
>とすればω'は順序数かつ無限集合であるからωの最小性よりω=ω'

調べてみるとこれは、
ω'がωより小さければωの最小性に矛盾する、よってω=ω'

という最小性を使った背理法が帰納法に紛れ込んでいると
言われているようです
0290132人目の素数さん
垢版 |
2022/06/04(土) 14:52:48.21ID:lN5GxstC
>>289
イヤ
ωの定義が

0∈ω、x∈ω→x+1∈ω、∀ω'(0∈ω'、x∈ω'→x+1∈ω')→ ω⊂ω')

でここ背理法関係ありません
0291132人目の素数さん
垢版 |
2022/06/04(土) 15:19:39.46ID:j+0F4bbH
>>290
すみません、>>289は注意しないと背理法が紛れ込むという文脈のようでした
基礎知識足りてないのでもう黙っときます…
0292132人目の素数さん
垢版 |
2022/06/04(土) 18:32:07.46ID:36zoZw+O
>>270
P(1)∧∀n∈N(P(n)→P(n+1))∧∃n∈N(¬P(n))→人
0293132人目の素数さん
垢版 |
2022/06/04(土) 18:38:28.86ID:36zoZw+O
>>270
P(1)∧∃n∈N(¬P(n))→∃n∈N(P(n)∧¬P(n+1))
が論理だけで証明できますか?
だったら帰納法は公理として必要ないですね
0294132人目の素数さん
垢版 |
2022/06/04(土) 18:44:36.61ID:36zoZw+O
>>267
古典論理でも数学的帰納法は証明できません
一方直観主義論理で数学的帰納法を公理として
追加しても排中律は証明できません
0295132人目の素数さん
垢版 |
2022/06/04(土) 18:57:42.02ID:GOI5W0c/
>>294
最初の方は当たり前
論理学で自然数論が展開できたらおかしい
0296132人目の素数さん
垢版 |
2022/06/04(土) 19:20:28.90ID:Pmn+9eta
背理法って、設定した仮定から帰結された結論が誤っていれば、
設定した仮定が誤っている、という論法のようだけど
誤った仮定から正しい結論を導くことも可能ではないでしょうか?
0297132人目の素数さん
垢版 |
2022/06/04(土) 19:37:28.18ID:RI91PHM7
>>294,295
>>287で良いんじゃ無いの?
0298132人目の素数さん
垢版 |
2022/06/04(土) 19:38:45.37ID:RI91PHM7
>>296
論理式で書いて
背理法を書くと
(¬P→人)→P
0299132人目の素数さん
垢版 |
2022/06/04(土) 19:46:42.71ID:GOI5W0c/
>>297
それは集合論からペアノ数論が展開できるという話
0300132人目の素数さん
垢版 |
2022/06/04(土) 20:00:17.16ID:RI91PHM7
>>299
それでいいんじゃ無いのってことだけど
論理学だけで帰納法証明できるわけでないってことでしょ?
素朴なイメージのNで背理法で帰納法証明するのも
Nがどういうものかイメージがあってのこと
厳密にするなら>>287で良かろうという話で
P(0)∧∀n∈ω(P(n)→P(n+1))→∀n∈ω(P(n))
が証明されたってことでは?
0301132人目の素数さん
垢版 |
2022/06/04(土) 22:21:10.01ID:JKh2dhS3
R^nの部分集合Sがデカルト積であることを示すにはどうすればいいですか?
0302132人目の素数さん
垢版 |
2022/06/04(土) 22:45:33.64ID:IuMYTp6H
示すことはできません
0303132人目の素数さん
垢版 |
2022/06/05(日) 02:19:07.04ID:72b1xwJa
>>296
>背理法って、設定した仮定から帰結された結論が誤っていれば、
>設定した仮定が誤っている、という論法のようだけど
それは違くて
(P→人)→¬P
は背理法でなくてただの否定の定義(¬P≡P→人)
背理法は
(¬P→人)→P
で二重否定の除去と同値
>誤った仮定から正しい結論を導くことも可能ではないでしょうか?
言いたいことが分からないので
論理式で書いて欲しい
0304132人目の素数さん
垢版 |
2022/06/05(日) 02:21:56.49ID:72b1xwJa
その上で
背理法とどう関係する疑問なのかも説明して欲しい
0305132人目の素数さん
垢版 |
2022/06/05(日) 04:50:22.09ID:72b1xwJa
>>287
>aが順序数:⇔(a,≦)がwell ordered set
これだと{0,2}も順序数?そう定義すると最小はどう定義する?
{0,2,4,6,…}の方がある意味小さいけど
普通はx∈y∈a→x∈aかつ∈(<)についてwell orderedだと思うけど
>φ(0)∧φ(x)→φ(x+1)
>を満たす述語のとき
>w' = { x∈ω | φ(x) }
>とすればω'は順序数かつ無限集合
ω'が順序数であることの証明は
well orderedだけならωの部分集合だからいいけれど
x∈y∈ω'→x∈ω'も必要なら
y∈ω'からφ(y)は真だけどx∈y(x<y)のxについてφ(x)が真であることに背理法は使わないかな?yが有限回(y回)で0から+1で届く(y∈ω'⊂ωだから)から大丈夫かな?
こうしたらどうかな
φを
∀y∈ω(∀x∈y(φ(x))→φ(y))
を満たす述語として
ω''={y∈ω|∀x∈y(φ(x))}
とするとx∈y∈ω''→x∈ω''も言えるから順序数かつ無限公理を満たしてω''=ωになるし
∀y∈ω(∀x∈y(φ(x))→φ(y))→∀x∈ω(φ(x))
のタイプの帰納法を成立させることになるし
0306132人目の素数さん
垢版 |
2022/06/05(日) 04:59:32.87ID:72b1xwJa
>>305
>ω''={y∈ω|∀x∈y(φ(x))}
>とするとx∈y∈ω''→x∈ω''も言えるから順序数かつ無限公理を満たしてω''=ωになるし
>∀y∈ω(∀x∈y(φ(x))→φ(y))→∀x∈ω(φ(x))
φの条件から
ω''={y∈ω|∀x∈y(φ(x))}⊂ω'={y∈ω|φ(y)}
なのでωの最小性より
ω⊂ω''⊂ω'⊂ω

ω=ω''=ω'
なので
∀y∈ω(∀x∈y(φ(x))→φ(y))→∀x∈∀y∈ω(φ(x))→∀x∈ω(φ(x))
0308132人目の素数さん
垢版 |
2022/06/05(日) 09:05:01.91ID:7zPiOD96
>>287
直観主義集合論使ってもええけど
選択入れたら排中律、証明できてまうで
ま、今では古典論理でもCurry=Howard成立するって
分かったから直観主義論理に固執せんけどな
0309132人目の素数さん
垢版 |
2022/06/05(日) 11:14:23.35ID:72b1xwJa
>>305
>∀y∈ω(∀x∈y(φ(x))→φ(y))→∀x∈ω(φ(x))
>のタイプの帰納法を成立させることになるし
∀y∈ω(∀x∈y(φ(x))→φ(y))
が成立しているときy=0={}については∀x∈{}(φ(x))=∀x(x∈{}→φ(x))は真なので∀x∈0(φ(x))→φ(0)が真とはすなわちφ(0)が成立することであるし
n∈ωについて∀x∈n+1(φ(x))→φ(n)は真なので
φが
∀n∈ω(φ(n)→φ(n+1))
を満たしていれば
∀n∈ω(∀x∈n+1(φ(x))→φ(n+1))
も満たすので
φが
φ(0)∧∀n∈ω(φ(n)→φ(n+1))
を満たしていれば
(∀x∈0(φ(x))→φ(0))∧∀n∈ω(∀x∈n+1(φ(x))→φ(n+1))
すなわち
∀n∈ω(∀x∈n(φ(x))→φ(n))
も満たすから
∀y∈ω(∀x∈y(φ(x))→φ(y))→∀x∈ω(φ(x))
のタイプの帰納法が成立するとき
φ(0)∧∀n∈ω(φ(n)→φ(n+1))→∀n∈ω(φ(n))
のタイプの帰納法も成立する
0310132人目の素数さん
垢版 |
2022/06/05(日) 11:21:54.59ID:72b1xwJa
φ(0)∧∀n∈ω(φ(n)→φ(n+1))→∀n∈ω(φ(n))
のタイプの帰納法も成立するとき
∀y∈ω(∀x∈y(φ(x))→φ(y))→∀x∈ω(φ(x))
のタイプの帰納法も成立すると思うけど
どう示せば良いのかよく分からないや
∀y∈ω(∀x∈y(φ(x))→φ(y))
が成立しているとき
∀n∈ω(φ(n)→φ(n+1))
も成立することがいえればいいんだろうけど
(φ(0)≡∀x∈0(φ(x))→φ(0)なのでφ(0)は成立する)
0311132人目の素数さん
垢版 |
2022/06/05(日) 12:25:04.87ID:72b1xwJa
なんか
n∈ωによって異なる長さの証明ってのが無限論理みたくてちょっといや
ω無矛盾とかもこの違いを言っているわけだし
0312132人目の素数さん
垢版 |
2022/06/05(日) 16:46:26.61ID:p6+O/n5w
双対空間がよくわかりません
これって単なる座標変換のことかと思ってるんですがあってます?
0313132人目の素数さん
垢版 |
2022/06/05(日) 18:05:52.38ID:lwOYlRry
空間が座標変換とはどういう意味?
汎関数の空間でしょ
0315132人目の素数さん
垢版 |
2022/06/06(月) 08:44:31.62ID:4PlQ6agd
>>314
はい共変ベクトルの空間です

そしてごめん関数空間を念頭において汎関数とかいっちゃったけどただの係数体への線形写像の空間やね
0316132人目の素数さん
垢版 |
2022/06/06(月) 09:27:35.90ID:djra2yDV
>>258
要するに直観論理は数学的議論に使うには貧弱過ぎるということね。
0319132人目の素数さん
垢版 |
2022/06/06(月) 23:11:57.71ID:wIH0wz2s
>>316
埋め込み定理があるから
古典論理の立場からすれば
それほどの違いは無いかと
0321132人目の素数さん
垢版 |
2022/06/07(火) 21:20:29.96ID:WhGXTET+
カリーハワード同型って
結局のところ
P→Q
が写像(プログラム言語での関数)と見なせて
MPはその適用るってことしか言ってないよな
0327132人目の素数さん
垢版 |
2022/06/08(水) 15:59:14.04ID:cP2hrzsC
>>322
失礼しました、これだけでは上手くいかないですね…
f(x)は[a,b]上連続です、よろしくお願いします
0328132人目の素数さん
垢版 |
2022/06/08(水) 16:15:51.65ID:cP2hrzsC
>>324
ありがとうございます
色々調べてみましたがデルタ関数がよく分からなくて理解できませんでした
学部1年生なので色々至らずすみません
0329132人目の素数さん
垢版 |
2022/06/08(水) 17:16:58.53ID:Mv3yPLyj
δ関数がわからないということがわかったのであれば、調べれば良いのでは。
0330132人目の素数さん
垢版 |
2022/06/08(水) 21:41:51.86ID:iBKQRksn
間違えるので式変形を手でやりたくないです
式変形チェッカーとか式変形してくれるソフトないでしょうか
できれば無料で
0331132人目の素数さん
垢版 |
2022/06/08(水) 21:44:14.09ID:uhdD1weu
をるふらむあるふぁ?
0333132人目の素数さん
垢版 |
2022/06/09(木) 10:43:06.81ID:4R55Bhqd
この問題分かる?

(問題)太郎は三郎の父である。三郎は花子の父である。
    それゆえ、花子は太郎の孫である。
 この演繹の論理定項を抜き出せ。
0334132人目の素数さん
垢版 |
2022/06/10(金) 07:31:39.70ID:oBqML+gz
>>333
意味分からん
0335132人目の素数さん
垢版 |
2022/06/10(金) 10:25:43.55ID:hKDKVRiL
連続関数は原始関数存在しますが不連続関数は全て原始関数は存在しないと言う事でいいですかね?
0336132人目の素数さん
垢版 |
2022/06/10(金) 10:56:08.46ID:xtZHle0v
f(0) := 0
f(x) := 2*x*sin(1/x) - cos(1/x) for x ∈ [-1, 0) ∪ (0, 1]

と f を定義すると

F(0) := 0
F(x) = x^2 * sin(1/x) for x ∈ [-1, 0) ∪ (0, 1]

は f の原始関数になるが、 f は x = 0 で不連続。
0338132人目の素数さん
垢版 |
2022/06/10(金) 21:14:23.76ID:w/5IiG0f
A, Bはn×nの正方行列
Sは集合{1,...,n}のべき集合
s⊂Sに対してA_sとB_sはsに含まれる行と列を取り出した小行列 (たとえばs={1,2}なら、A_sはAの1行目2行目、1列目2列目からなる2×2の行列)
s'はSの補集合
とした場合に、
Σ_{s⊂S} det(A_s) det(B_{s'})
ってnの多項式時間で計算できますか?
または、別のシンプルな形に書き換えられたりしますか?
0340132人目の素数さん
垢版 |
2022/06/10(金) 22:18:08.28ID:Ix7t4aeS
係数環によるやろ
係数環が体に埋め込まれるケースとかならいけるけどそうでないなら無理やろ
0341132人目の素数さん
垢版 |
2022/06/10(金) 22:34:12.20ID:w/5IiG0f
>>340
実数または複素数で考えてるのですが、もし計算できるなら計算方法教えていただけると嬉しいです
0342132人目の素数さん
垢版 |
2022/06/10(金) 22:43:22.11ID:KEkGsbeO
ハルモスのの言葉を思い出した。応用数学者から質問を受けた経験から得た教訓。
Whatever you do, don't solve the problem you're asked about.

Don't answer the client's question, but, instead, help him to formulate it.
0344132人目の素数さん
垢版 |
2022/06/11(土) 00:11:54.78ID:0ORewrJk
>>343
0345132人目の素数さん
垢版 |
2022/06/11(土) 00:15:32.49ID:S/8vfUle
行列A
wx
yz

行列B
WX
YZ
>>338の定義で考えると
(wz-xy) +wZ+Wz + (WZ-YZ)
だけど
det(A+B) =(w+W)(z+Z) - (x+X)(y+Y)
とは違う
0346132人目の素数さん
垢版 |
2022/06/11(土) 00:21:28.60ID:0ORewrJk
(a11+b11)(a22+b22)-(a12+b12)(a21+b21)
≠(a11a22-a12a21)1+(a11b22)+(a22b11)+1(b11b22-b12b21)
0348132人目の素数さん
垢版 |
2022/06/11(土) 01:02:17.90ID:3uNYTIr/
まぁしかしとりあえず列変形で値が不変なのは当たり前だからそれでなんとかなりそうではある
0349132人目の素数さん
垢版 |
2022/06/11(土) 01:07:26.68ID:3uNYTIr/
あれ?行変形でも不変かな?
だったらこの手の変形の標準形が見つかってるからそれでいけるはずだけど
0350132人目の素数さん
垢版 |
2022/06/11(土) 06:31:48.46ID:9nEwS8vl
どういう流れから生じた問題なのか知りたい。
大体の場合知りたいことと問題の設定がズレている。
0352132人目の素数さん
垢版 |
2022/06/11(土) 15:57:25.27ID:MKUwp4cU
実数xおよび自然数nが与えられたとき、
Σ(k=1,n-1)[x+k/n]=[nx]
が成り立つ事を証明せよ。

解答で
x=m+a/n(0≦a<n,[a]=u)と置いて解いてるのですが実数xに対してこう置いても問題ないのはなぜですか?
0353132人目の素数さん
垢版 |
2022/06/11(土) 16:56:00.17ID:eXbdoopC
順にmを決めてaをn(x-m)とすればいい。
あと問題がおかしい。
0354132人目の素数さん
垢版 |
2022/06/11(土) 17:30:08.03ID:JMskhT+z
距離空間の完備化で、もとの空間が完備化した空間の中で稠密になります。この性質をなぜ完備化の条件に課すのですか?
単にもとの空間を完備にするだけではだめなんですか?
0355132人目の素数さん
垢版 |
2022/06/11(土) 17:34:57.14ID:S8pZhzAs
x = q + r/n + ρ(ρ ∈ [0,1) )、x₀= q + r/nとすれば
x→x₀としても左辺も右辺も値は変わらない
0356132人目の素数さん
垢版 |
2022/06/11(土) 18:40:37.11ID:JMskhT+z
距離空間Sの完備化S*の定義:
(1) S*は完備空間である。
(2) S*⊃Sである。
(3) S*の真部分集合は完備ではない。

この定義では駄目ですか?
0357132人目の素数さん
垢版 |
2022/06/11(土) 18:50:14.15ID:9nEwS8vl
>>356
「Sを含む最小の完備空間」っていう定義が既にあって、それと同値。
0359132人目の素数さん
垢版 |
2022/06/11(土) 20:01:29.75ID:tIuKg9t5
>>356
>(3) S*の真部分集合は完備ではない。

一点集合も駄目となるとどうしようもねえなこれ
0360132人目の素数さん
垢版 |
2022/06/11(土) 20:09:12.89ID:JMskhT+z
距離空間Sの完備化S*の定義:
(1) S*は完備空間である。
(2) S*⊃Sである。
(3) S*の真部分集合でSを含むものがあればそれは完備ではない。

この定義では駄目ですか?
0361132人目の素数さん
垢版 |
2022/06/11(土) 20:09:49.07ID:JMskhT+z
>>359
なるほど。
>>360
ではどうですか?
0362132人目の素数さん
垢版 |
2022/06/11(土) 20:15:45.71ID:9nEwS8vl
ID:JMskhT+zは何がしたいのだろう?
何に拘っているんだろう?
0363132人目の素数さん
垢版 |
2022/06/11(土) 20:24:48.96ID:JMskhT+z
Sを含む最小の完備空間S*をSの完備化と定義したいのですが、それだと駄目なので、
>>360
ではどうかという質問です。
0364132人目の素数さん
垢版 |
2022/06/11(土) 20:36:24.47ID:9nEwS8vl
駄目じゃないよ
Sが一点集合ならSの完備化はS自身で、その真部分集合はSを含まない
0365132人目の素数さん
垢版 |
2022/06/11(土) 20:50:07.36ID:tIuKg9t5
「Sの完備化=Sを含む最小の完備空間」は正しい
>>356の駄目なところは「Sに含まれる完備空間が存在しない」という条件を与える(3)

というか元々の>>354見るに完備性と稠密性の定義がわかってないようにしか見えん
0366132人目の素数さん
垢版 |
2022/06/11(土) 21:12:49.26ID:JMskhT+z
「Sの完備化=Sを含む最小の完備空間」は正しい

S=[0,1)とする。
S1*=[0,1]はSの完備化
S2*=[0,1)∪{2}はx∈[0,1)に対してd(x,2)=|x-1|, d(2,2)=0と定義すれば、Sの完備化
S1*⊂S2*でもS2*⊂S1*でもないので、最小の完備化は存在しない。
0368132人目の素数さん
垢版 |
2022/06/11(土) 21:26:29.58ID:9nEwS8vl
距離を変えたらその完備化が変わるのは当たり前だろ。
馬鹿を相手にしてたのか、あ〜あ。
0369132人目の素数さん
垢版 |
2022/06/11(土) 21:28:30.99ID:tIuKg9t5
>>368
これが松坂くんクオリティだ
どんまい
0370132人目の素数さん
垢版 |
2022/06/11(土) 22:38:29.58ID:6j4r5+Y4
可分な距離空間の位相同型類の種類は連続濃度です
可分な位相空間の位相同型類の種類は自明な位相空間だけ考えるだけでもずっと大きい事が分かります
可分なハウスドルフ空間やT0空間、T1空間の位相同型類の種類の濃度はどうなるのでしょうか
0371132人目の素数さん
垢版 |
2022/06/11(土) 22:50:47.16ID:KxIQFZgd
>>370

> 可分な距離空間の位相同型類の種類は連続濃度です

え?
そんなのどうやって証明するんですか?
0372132人目の素数さん
垢版 |
2022/06/11(土) 23:06:23.03ID:AFXBCBw8
可分だから可算個の稠密集合とってその間の距離関数作ってでなんとなく連続濃度^可算になりそうではあるけど
例えばこの構成で出てるやつは全部非同型とは限らないし
しかもコレで完全にパラメトライズされてる?
0373132人目の素数さん
垢版 |
2022/06/12(日) 02:13:44.42ID:oCkDVVCq
>>370
やっぱり嘘やろコレ
(m,n)∈ℕ²に対してℝ²の部分集合C(m,n)を中心(m+1/2,n+1/2)、半径1/2の円とする
0,1の無限列𝕒=aᵢに対して空間x(𝕒)を
x(𝕒) = ℝ×{0} ∪ (∪[i∈ℕ、a=1ᵢ1≦j≦pᵢ] C(2u,j)
と定める(ただしpᵢはi番目の素数)
例えば𝕒=1010101...
だったらX(𝕒)は

│◯◯ (𝕒₁=1 なのでp₁=2個の◯が繋がってる部分を持つ)

│◯◯◯◯◯ (𝕒₃=1 なのでp₃=5個の◯が繋がってる部分を持つ)

│◯◯◯◯◯◯◯◯◯◯◯


みたいな空間とする
違う列は違う空間の同型類を与えてコレらは全部非同形てとりあえずこの時点で連続体無限個ある
さらに𝕒に対して実数r(𝕒)を
r(𝕒)の小数第2i桁=1 iff 𝕒ᵢ=1
r(𝕒)の小数第奇数桁=0
で定めてℝ³の部分集合S(𝕒)を
S(𝕒) = { ( r(𝕒), y,z ) | (y,z) ∈ X(𝕒) }
と定める
次に01列の集合𝔸に対して空間X(𝔸)⊂ℝ³を
X(𝔸) = ∪[𝕒∈𝔸]S(𝕒)
で定める
X(𝔸)の連結成分に含まれてるS(𝕒)にどんなものが入っているかで元の𝔸が完全に復元される、すなわち対応𝔸→X(𝔸)は01列の集合の冪集合からℝ³の部分空間の同型類への単射を与える
ここでℝ³は可分距離空間だからその部分集合も可分距離空間(ソースwiki)
すなわち可分距離空間の同型類の全体は少なくとも2^連続体濃度より大きい
0374132人目の素数さん
垢版 |
2022/06/12(日) 17:27:02.52ID:42RKVyuF
>>333
>>334
野矢「まったくゼロからの論理学」の中の問題?
哲学的論理学?
0375132人目の素数さん
垢版 |
2022/06/12(日) 22:14:01.97ID:27UMT7fG
X := {1, 2, …, n}
T := {S ∈ 2^(2^X) | S に属する集合の要素の数は奇数。S に属する任意の2つの集合の共通部分の要素の数は偶数。}
T に属する集合の中でその要素数が最大であるものの要素数を求めよ。
0376132人目の素数さん
垢版 |
2022/06/13(月) 00:04:40.18ID:EXC3bC0j
>>375

T := {S ∈ 2^(2^X) | S に属する集合の要素の数は奇数。S に属する任意の2つの集合の“相異なる”共通部分の要素の数は偶数。}

だとしてmax{ ♯S | S∈T } = 2^n
(証) Sが条件を満たすには任意のA∈Sに対して
A\∪[B∈S,B≠A]B ≠ Φ
が必要
よってSの各元Aに対してA\∪[B∈S,B≠A]B の元を選択する関数をとることができて明らかに単射
∴ ♯S≦2^n
0377132人目の素数さん
垢版 |
2022/06/13(月) 02:49:20.89ID:Vmhjqq7F
>>376
Sが条件を満たすには任意のA∈Sに対して
A\∪[B∈S,B≠A]B ≠ Φ
になるのはなぜですか?
0378132人目の素数さん
垢版 |
2022/06/13(月) 02:56:56.21ID:Vmhjqq7F
S ∈ 2^(2^X)

すなわち、

S ⊂ 2^X

であり

#(2^x) = 2^n

なので、

#S ≦ 2^n

は明らかです。
0379132人目の素数さん
垢版 |
2022/06/13(月) 03:00:58.24ID:Vmhjqq7F
>>375
解答がない場合には、解答を書き込みます。
0380132人目の素数さん
垢版 |
2022/06/13(月) 03:08:38.82ID:Vmhjqq7F
>>375

問題を訂正します:

X := {1, 2, …, n}
T := {S ∈ 2^(2^X) | S に属する集合の要素の数は奇数。S に属する任意の異なる2つの集合の共通部分の要素の数は偶数。}
T に属する集合の中でその要素数が最大であるものの要素数を求めよ。
0381132人目の素数さん
垢版 |
2022/06/13(月) 03:10:42.10ID:Vmhjqq7F
S := {{1}, {2}, …, {n}}

とすると、

S ∈ T

です。

よって、

答えは n 以上になります。
0382132人目の素数さん
垢版 |
2022/06/13(月) 03:27:35.27ID:UeeXzCMK
>>379
ここは質問スレであり、出題スレではない。
0383132人目の素数さん
垢版 |
2022/06/13(月) 07:38:32.42ID:Vmhjqq7F
正解である解答がない場合には、今日の夜に解答を書く予定です。
0384132人目の素数さん
垢版 |
2022/06/13(月) 07:47:20.28ID:GHa65NWZ
完備化で恥かいたから問題出して優越感に浸ろうとしてるのね
小学生かな?
0385132人目の素数さん
垢版 |
2022/06/13(月) 09:49:47.74ID:VnbOXQxy
しかも答え出てるのにな
人の証明はガダガタ言うくせに自分のつける証明は穴だらけでしかも今回に至っては結論そのもの間違ってるし
0386132人目の素数さん
垢版 |
2022/06/13(月) 10:08:37.85ID:7LdbCjEq
横からですまんが、こういうことを求めてるんじゃないか:

(Z/2Z)^n に内積入れてノルム1の互いに直交する元の数を数える。
答えは n
0387132人目の素数さん
垢版 |
2022/06/13(月) 10:57:18.43ID:1/0JqUXT
>>386
流石に𝔽₂係数の内積は退化してしまってるのでそこまで簡単には行かないけどその方針が楽やな
条件は
a⃗ᵢa⃗ᵢ=1、a⃗ᵢa⃗
0388132人目の素数さん
垢版 |
2022/06/13(月) 11:08:51.56ID:1/0JqUXT
途中で書いてしまった
a⃗≠0̅であったとしても(a⃗,a⃗)=0かもしれないのでn次元空間の互いに“直交”するベクトルの最大個数がnはそこまで自明ではない
けどその方針が見やすいな
0389132人目の素数さん
垢版 |
2022/06/13(月) 11:41:28.66ID:z0e6ctv1
内積というか行列のランクの問題のつもりじゃない?
rankA^TA=rankA
0390132人目の素数さん
垢版 |
2022/06/13(月) 11:54:54.71ID:JCdVpk9z
>>389
せやな
(𝕒ₖ)を
𝕒ₖ𝕒ₖ=1、𝕒ₖ𝕒ₗ=0
を満たす𝔽₂係数のn次元の列ベクトルがm組あったとして横に並べてn行m列の行列Aを作ればいいんやな
0391132人目の素数さん
垢版 |
2022/06/13(月) 13:24:07.96ID:QvHwF/Ow
Rみたいに、四則演算だけじゃなくてべき乗も定義された体ってなんか名前ついてる?
0392132人目の素数さん
垢版 |
2022/06/13(月) 13:37:01.33ID:yAXjnnZF
正解です。
Babai and FranklのLinear Algebra Methods in Combinatoricsのウォーミングアップ問題です。
0393132人目の素数さん
垢版 |
2022/06/13(月) 14:47:04.62ID:NeWUTx1A
>>392
正解どうこう言う以前にお前はそのクソみたいな性格なおせや
人の証明は細かいとこ重箱の隅つつくような話ばっかりして自分は平気で雑っい理論で「こうなんやろな」ですます議論重ねて挙げ句の果ての>>370
恥ずかしいないんか?
0394132人目の素数さん
垢版 |
2022/06/15(水) 08:09:07.27ID:26yVsFvx
A を n 次正方行列とする。
C := {A ∈ M_n(R) | det(A) = 0} は閉集合です。
C の他の幾何学的性質は何ですか?
0395132人目の素数さん
垢版 |
2022/06/15(水) 08:11:31.12ID:26yVsFvx
n ≧ 2 のとき C は4次元以上の空間です。

C の顕著な性質は何ですか?
0396132人目の素数さん
垢版 |
2022/06/15(水) 08:12:07.31ID:26yVsFvx
訂正します:

n ≧ 2 のとき C は4次元以上の空間の中の図形です。

C の顕著な性質は何ですか?
0397132人目の素数さん
垢版 |
2022/06/15(水) 08:29:01.74ID:26yVsFvx
R^3 の中の曲面が閉じているというのはどう定義するのでしょうか?
0398132人目の素数さん
垢版 |
2022/06/15(水) 08:36:23.42ID:14JE7iBS
>>394
ここは質問スレだ。出題スレではない。
0399132人目の素数さん
垢版 |
2022/06/15(水) 09:24:35.50ID:Jc374dX3
まぁこの恥知らずなとこがコイツの数学力初めとした無能さの根源なわけなんだが
0400132人目の素数さん
垢版 |
2022/06/15(水) 09:39:39.52ID:26yVsFvx
{(x_1, …, x_n) ∈ R^n | f_1(x_1, …, x_n) = 0, …, f_m(x_1, …, x_n) = 0}

が連結かそうじゃないかってどう調べるんですか?
0401132人目の素数さん
垢版 |
2022/06/15(水) 09:44:20.10ID:QG64CxpQ
>>400
一般的な方法はないんじゃない?
0402132人目の素数さん
垢版 |
2022/06/15(水) 09:46:19.93ID:26yVsFvx
S := {(x_1, …, x_n) ∈ R^n | f(x_1, …, x_n) = 0}

という図形の性質を調べたいとします。

f(x_1, …, x_n) = 0 の解たちがすべて分かったとして、その解たち(数字の組の集合)からどうやって S の幾何学的な性質を知ることができるのでしょうか?
0403132人目の素数さん
垢版 |
2022/06/15(水) 09:52:12.79ID:QG64CxpQ
>>397
境界のない有界閉曲面?
0404132人目の素数さん
垢版 |
2022/06/15(水) 09:53:41.87ID:QG64CxpQ
>>402
個別にどうやるかを考えるんじゃないかな
0405132人目の素数さん
垢版 |
2022/06/15(水) 09:55:36.99ID:26yVsFvx
数学的に面白い理論を知りたいから数学を勉強するというのではなく、例えば、実数の定義や面積の定義や曲面の定義を
知りたいから数学を勉強するという人っていますかね?
0406132人目の素数さん
垢版 |
2022/06/15(水) 09:56:34.70ID:26yVsFvx
>>401,403-404

ありがとうございます。
0407132人目の素数さん
垢版 |
2022/06/15(水) 18:50:09.46ID:rgakG9iD
秋山仁って日本国内で評価されてるんですか?
0408132人目の素数さん
垢版 |
2022/06/15(水) 19:05:36.79ID:QG64CxpQ
>>407
当たり前だろ
第一発見者ではないにせよ
正多面体の分割の定理は
大いにビックリした
0410132人目の素数さん
垢版 |
2022/06/15(水) 20:33:50.02ID:izx+NJih
>第一発見者ではないにせよ

第一発見者でないなら、学術的には評価に値しないじゃん
0411132人目の素数さん
垢版 |
2022/06/15(水) 21:13:32.58ID:QG64CxpQ
>>410
証明付けたのは彼
0414132人目の素数さん
垢版 |
2022/06/16(木) 22:26:35.73ID:t8sJQAHA
この式の証明図を書いていただきたいです

(P→Q∨R)→((Q→S)∧(R→S)→(P→S))
0415132人目の素数さん
垢版 |
2022/06/17(金) 00:15:04.66ID:1vAmlj5h
>>407
されてない
秋山仁と同列に扱われるとイラっとする数学者は多い
1年生がやりがち
0416132人目の素数さん
垢版 |
2022/06/17(金) 01:59:04.19ID:Un+Ku3Rt
1,P→Q∨R ・・・・・・・仮定
2,(Q→S)∧(R→S) ・・・仮定
3,p ・・・・・・・・・・仮定
4,Q∨R ・・・・・・・・1と3より→除去
5,Q ・・・・・・・・・仮定
6,Q→S ・・・・・・・・2より∧除去
7、S ・・・・・・・・・・5と6より→除去
8,R ・・・・・・・・・・仮定
9,R→S ・・・・・・・・2より∧除去
10,S ・・・・・・・・・8と9より→除去
11,S ・・・・・・・・・4と7と10より∨除去
12,p→S ・・・・・・・3と11より→導入
13,(Q→S)∧(R→S)→(p→S) ・・・2,12より→導入
14,(P→Q∨R)→((Q→S)∧(R→S)→(p→S)) ・・・1,13より→導入
0417132人目の素数さん
垢版 |
2022/06/17(金) 15:17:43.87ID:6aqe6cMH
>>415
でも実際世界が大して振り向かない定理、概念を扱ってるような数学者も日本には多いよね
それどころかむしろ証明できてないものを出来たとしてしまった人達もいる
秋山仁とは違うなんて言える存命中の日本の数学者なんて指折りで数えられるくらいしかいない
0418132人目の素数さん
垢版 |
2022/06/17(金) 19:13:15.95ID:P27hnq19
松坂和夫著『集合・位相入門』

(S, O) を位相空間とする。
x ∈ S とする。
V(x) を x の近傍全体の集合とする。

pp.161-162

定理10
(Vi) すべての V ∈ V(x) に対して、 x ∈ V。
(Vii) V ∈ V(x) で V ⊂ V' (V' ∈ 2^S)ならば、 V' ∈ V(x)。
(Viii) V1 ∈ V(x)、 V2 ∈ V(x) ならば、 V1 ∩ V2 ∈ V(x)。
(Viv) 任意の V ∈ V(x) に対して、次の条件を満たす W ∈ V(x) が存在する:
W の任意の点 y に対して V ∈ V(y)。

定理11
(Vi)〜(Viv) を満たす S から 2^S - {空集合} への関数 x → V(x) に対して、
V(x) が位相空間 (S, O) における x の近傍系となるような位相空間 (S, O) が一意的に存在する。

定理10って以下のように書いたほうがいいですよね?

(V0) すべての x ∈ S に対して、 V(x) は空集合ではない。
(Vi) すべての V ∈ V(x) に対して、 x ∈ V。
(Vii) V ∈ V(x) で V ⊂ V' (V' ∈ 2^S)ならば、 V' ∈ V(x)。
(Viii) V1 ∈ V(x)、 V2 ∈ V(x) ならば、 V1 ∩ V2 ∈ V(x)。
(Viv) 任意の V ∈ V(x) に対して、次の条件を満たす W ∈ V(x) が存在する:
W の任意の点 y に対して V ∈ V(y)。

定理11
(V0)〜(Viv) を満たす S から 2^S への関数 x → V(x) に対して、
V(x) が位相空間 (S, O) における x の近傍系となるような位相空間 (S, O) が一意的に存在する。
0419132人目の素数さん
垢版 |
2022/06/17(金) 19:19:45.46ID:P27hnq19
S = {0, 1, 2, …, n-1}

に対して、(Vi)〜(Viv)を満たすような関数 x → V(x) をすべて求めるPythonプログラムを作って、
S に入る位相の数を数えたのですが、最初答えが合いませんでした。

原因は、(Vi)〜(Viv) を満たす S から 2^S への関数 x → V(x) をすべて求めていたため、
x に空集合を対応させるような関数も答えに含めていたためでした。

わざわざ分かりにくく書いた松坂和夫さんの意図は何でしょうか?
0420132人目の素数さん
垢版 |
2022/06/17(金) 20:45:11.09ID:pVq0H3YU
自分のケアレスミスを他人に責任転嫁するクズ
0422132人目の素数さん
垢版 |
2022/06/17(金) 23:13:05.51ID:NOvhGjyV
>>416
仮定の番号もそれぞれ教えていただきたい
0423132人目の素数さん
垢版 |
2022/06/17(金) 23:15:46.29ID:Xj+NER5+
>>421
は?
(V0)はV(x)が空集合ではないということなのだが?
0424132人目の素数さん
垢版 |
2022/06/17(金) 23:16:41.10ID:Xj+NER5+
>>422
書いてるが?
0425132人目の素数さん
垢版 |
2022/06/17(金) 23:20:42.65ID:CrgqlzS+
>>423
なんやその口の聞き方?
口の聞き方以外見るとこなかったけどそれも終わりかクズ
0426132人目の素数さん
垢版 |
2022/06/17(金) 23:21:40.67ID:9eENwgzP
>>424
私は演繹"図"で書きたいので
0427132人目の素数さん
垢版 |
2022/06/17(金) 23:28:16.87ID:62YgmL6J
例えば異なる形の仮定でも同じ推論図(∨E)で閉じることがあるでしょう?
その場合は同じ番号になるじゃないですか
QとRが同じとかそういうの知りたいです
0428132人目の素数さん
垢版 |
2022/06/17(金) 23:33:20.87ID:ek9T52yp
多変量ガウス分布を
P({x_i})=C*exp(-1/2*Σ_i,j A_i,j*x_i*x_j)
としたときの二乗平均<x_i x_j>の計算がわかりません。
今、x_iは平均からのずれとして簡単化していて、行列Aは対称行列かつ固有値は全て正です。
ヒントとして直交変換をしてAを対角化すれば計算できるというのですがやり方が分かりません。
ちなみに答えは<x_i x_j>=(A^-1)_i,jです。
どなたか途中まででも大丈夫ですのでご教授くださいm(__)m
0429132人目の素数さん
垢版 |
2022/06/17(金) 23:38:09.81ID:U0hglOqw
>>426
無能で偉そうなクズ
0430132人目の素数さん
垢版 |
2022/06/17(金) 23:39:43.97ID:Xj+NER5+
>>425
はぁ
ホントに理解してないみたいだな
0431132人目の素数さん
垢版 |
2022/06/17(金) 23:41:05.19ID:Xj+NER5+
>>427
>例えば異なる形の仮定でも同じ推論図(∨E)で閉じることがあるでしょう?
>その場合は同じ番号になるじゃないですか
具体例書いて
0432132人目の素数さん
垢版 |
2022/06/17(金) 23:44:10.55ID:c0QhThwt
>>426
突然横槍入れてきてお前は誰なんだよこの無能クズ
0433132人目の素数さん
垢版 |
2022/06/17(金) 23:44:37.94ID:c0QhThwt
間違えた
>>426じゃなくて>>429に対して言った
0434132人目の素数さん
垢版 |
2022/06/17(金) 23:51:16.35ID:wnJ720ut
>>431
ごめんなさい
あなたに聞いてるんじゃねンだわ
0435132人目の素数さん
垢版 |
2022/06/17(金) 23:53:12.21ID:f9emcFc/
>>428
そのヒントが正しいなら直交行列Uを
UᵗAU = diag(λ₁,λ₂,...λₙ)と選べば𝕩を変数の行ベクトルとして
U<𝕩ᵗ,𝕩>Uᵗ = diag(λ₁⁻¹,λ₂⁻¹,...λₙ⁻¹)になるんじゃない?

<𝕩ᵗ,𝕩> = Uᵗ diag(λ₁⁻¹,λ₂⁻¹,...λₙ⁻¹)U = A⁻¹となるのではないかと
0436132人目の素数さん
垢版 |
2022/06/17(金) 23:54:52.28ID:f9emcFc/
>>430
なんやカス
お前がこの板でそんな口叩ける力ないのはもうみんな知っとるわ
そもそも>>418からの書き込みも能無し度満載やろがカス
0437132人目の素数さん
垢版 |
2022/06/18(土) 00:00:52.45ID:Td60LNBN
>>431と松坂くんが同一だと思っちゃうのは半年ROMってろとしか言えんわ
0438132人目の素数さん
垢版 |
2022/06/18(土) 00:01:54.82ID:Td60LNBN
>>430だったけど>>431も同じ人だからいいか
0439132人目の素数さん
垢版 |
2022/06/18(土) 00:08:26.02ID:WugKaVjL
>>435
レスありがとうございます。
行列苦手で難しいです。最初の式が対角化なのはわかりました。次の
U<𝕩ᵗ,𝕩>Uᵗ = diag(λ₁⁻¹,λ₂⁻¹,...λₙ⁻¹)
になるのはどうしてでしょうか?
普通の二乗平均を計算する時に
<x^2>=∫dx x^2P(x)
のようにやると思いますが(ベクトルだと違うのですかね?)U,U^-1で挟んだときの式変形がわからないです。
0440132人目の素数さん
垢版 |
2022/06/18(土) 00:21:30.45ID:xFPBUxAo
>>434
じゃ
>>431は誰に聞いてるの?
何も指定が無いんだけど
それと指定が有っても
別にそれが遵守されるわけではないがな
0441132人目の素数さん
垢版 |
2022/06/18(土) 00:23:44.26ID:xFPBUxAo
>>436
結局この恥ずかしい
>>421
>(vi)も満たしてるのになんで空集合混じるんだよ
>アホか
がまるで見当違いの罵倒だとは理解できないのだね
0443132人目の素数さん
垢版 |
2022/06/18(土) 00:45:36.20ID:khMb2Wmv
>>439
とりあえず2行2列の場合なら

U<𝕩ᵗ,𝕩>Uᵗ
=U ∫(xy)ᵗ (x y) exp( -(xy)A(xy)ᵗ )dxdy Uᵗ
=∫U(xy)ᵗ (x y) U exp(- (xy)A(xy)ᵗ )dxdy
=∫( s t )ᵗ (s t) exp( -(s t )UᵗAU(s t )ᵗ )dsdt
=∫( s t )ᵗ (s t) exp(- (s t )diag(λ₁,λ₂)(s t )ᵗ )dsdt
=∫( s t )ᵗ (s t) exp( -λ₁s²-λ₂t²) dsdt
= diag(-λ₁⁻¹、λ₂⁻¹)

でコレnにするだけかな
0445132人目の素数さん
垢版 |
2022/06/18(土) 01:13:19.99ID:WugKaVjL
>>443
n=2で確かめて同様にnのときもこう書けるってことですね、ありがとうございます!
0446132人目の素数さん
垢版 |
2022/06/18(土) 12:24:02.55ID:88/h3s+i
定理11の証明って結構難しくないですか?
斎藤毅さんが「閉包作用素や近傍系などによる位相の定義、およびそれらの、開集合系による定義との同値性」について
あえて扱わなかったと「参考書」のところに書いていますね。
難しいわりにどうでもいい話ということですよね。

『集合・位相入門』の難所はこのあたりと「位相の比較、位相の生成」の
ところあたりだと思いますが合っていますか?

定理10
(Vi) すべての V ∈ V(x) に対して、 x ∈ V。
(Vii) V ∈ V(x) で V ⊂ V' (V' ∈ 2^S)ならば、 V' ∈ V(x)。
(Viii) V1 ∈ V(x)、 V2 ∈ V(x) ならば、 V1 ∩ V2 ∈ V(x)。
(Viv) 任意の V ∈ V(x) に対して、次の条件を満たす W ∈ V(x) が存在する:
W の任意の点 y に対して V ∈ V(y)。

定理11
(Vi)〜(Viv) を満たす S から 2^S - {空集合} への関数 x → V(x) に対して、
V(x) が位相空間 (S, O) における x の近傍系となるような位相空間 (S, O) が一意的に存在する。
0447132人目の素数さん
垢版 |
2022/06/18(土) 12:35:01.91ID:88/h3s+i
トポロジーの基礎 上下 単行本 – 2022/6/17
河澄 響矢 (著)

これって買ったほうがいいですかね?
0448132人目の素数さん
垢版 |
2022/06/18(土) 18:02:05.19ID:fKNkkiC8
>>446
与えられた定義から開核作用子 i を定義し、それが実際に開核作用子になっていることを証明し、
そして i から作られる位相 θ_i が求める位相になっていることを示すのが定石。
定理11の場合は、A⊂X に対して

i(A):= { x∈X|A∈V(x) }

と定義する。これが開核作用子になっていることを証明するのだが、ごく普通に証明できる。
次に、i から作られる位相 θ_i について、x∈X の θ_i に関する近傍系を n_{θ_i}(x) と置くとき、
n_{θ_i}(x)=V(x) が成り立つことが、これまた普通に証明できる。
従って、この位相 θ_i は求める性質を満たす(存在性の証明がこれで終わる)。
一意性については、別の位相 θ が

∀x∈X s.t. n_θ(x)=V(x)

を満たすとすると、n_{θ_i}(x)=V(x)にも注意して

∀x∈X s.t. n_θ(x)=n_{θ_i}(x)

となるので、θ=θ_i となることがごく普通に証明できる。
0449132人目の素数さん
垢版 |
2022/06/19(日) 12:16:29.88ID:wD10mGCI
>>448

ありがとうございました。
0450132人目の素数さん
垢版 |
2022/06/19(日) 12:17:15.02ID:wD10mGCI
トポロジーの基礎 上下 単行本 – 2022/6/17
河澄 響矢 (著)

↑この本ですが、中身を確認せずに注文してしまいました。
0451132人目の素数さん
垢版 |
2022/06/19(日) 12:51:09.02ID:z9M0e1Rz
河澄響矢さんは大丈夫な人なのでしょうか?
0452132人目の素数さん
垢版 |
2022/06/19(日) 13:05:21.98ID:y+uuhDOR
幾何学用語の英語のrayは日本語の数学用語で何と呼ばれていますか?
端点が片方だけにあって矢印で表される直線のことですが。光線?
0453132人目の素数さん
垢版 |
2022/06/19(日) 13:10:02.56ID:y+uuhDOR
ーーーーーーーーー・・・・→∞
左に端点、右が無限に伸びる直線。
0455132人目の素数さん
垢版 |
2022/06/19(日) 13:14:05.22ID:fBILhsD6
半直線の像ね
0456132人目の素数さん
垢版 |
2022/06/19(日) 13:15:52.15ID:fBILhsD6
輻(や)はどうかしら
0458132人目の素数さん
垢版 |
2022/06/19(日) 17:48:23.51ID:+MqTtwGz
>>455
それはアホなツッコミだと思う
0459132人目の素数さん
垢版 |
2022/06/19(日) 18:02:36.55ID:wD10mGCI
Sheldon Axler著『Measure, Integration & Real Analysis』

ハードカバーのものがSpringerで送料込3600円くらいで買えるので、注文しました。
PDFファイルは無料公開されていますが、やはり紙の本のほうがいいです。
0460132人目の素数さん
垢版 |
2022/06/20(月) 00:25:06.83ID:RSNQPYxo
(zz*-iz-iz*-1)/(z^2+1)→-1 (z→i)
となるらしいんですけどなんでですかね?
z*はzの共役複素数です
0462132人目の素数さん
垢版 |
2022/06/20(月) 01:11:44.87ID:MvQk22Tg
なるほど、(zz*-iz-iz*-1)/(z^2+1)=(z-i)(z*-i)/(z-i)(z+i)
=(z*-i)/(z+i)なんですね。
0463132人目の素数さん
垢版 |
2022/06/20(月) 06:54:13.93ID:HbQxMA18
・関数fが点aで微分可能ならば点aで連続
・関数fが点aで連続でも点aにおいて微分可能とは限らない
・関数fが点aで右側微分可能なら点a右側連続

以上がいずれも正しいことは分かるのですが、以下も正しいのでしょうか?

・関数fが点aで右側連続でも点aで右側微分可能とは限らない

もし正しい場合には具体例を教えて頂ければ助かります。それとも、右側微分に関しては以下が成り立つのでしょうか。

・関数fが点aで右側連続ならば点aで右側微分可能
0464132人目の素数さん
垢版 |
2022/06/20(月) 07:12:41.37ID:Sgrahl7/
>>463
f(x)=(x-a)sin(1/(x-a)) (x>a)
f(a)=0
とか
0465463
垢版 |
2022/06/20(月) 07:21:17.72ID:HbQxMA18
>>464
すごいですね!ありがとうございます。勉強になりました。
0466132人目の素数さん
垢版 |
2022/06/20(月) 10:59:47.38ID:RSNQPYxo
>>461
分母のz^2+1を(z+i)(z-i)に分解するのが思い付きませんでした
ありがとうございました!
0467132人目の素数さん
垢版 |
2022/06/20(月) 11:18:13.38ID:BN7e+diH
複素数体ℂの乗法群ℂ*:=ℂ-{0}の真部分群Gでℂ*と同型(G≅ℂ*)になる例はありますでしょうか?
0468132人目の素数さん
垢版 |
2022/06/20(月) 11:46:54.48ID:HsQvmwK3
>>467
ℂ*の乗法群の直和因子でねじれ部分(何乗かして1になる元)を含まないものGを任意にとる( ex. G = { exp( q ) | q ∈ ℚ } )
ℂ* = G⊕HとしてH⊂ℂ*
一般にAがアーベル群でBがその部分群に対してB̅={ g∈G | ng ∈ B (∃n∈ℤ) }とすればB̅はAの直和因子
0469132人目の素数さん
垢版 |
2022/06/20(月) 12:13:08.68ID:RSNQPYxo
複素数z,wに対して
|z-w|=2sin|(Arg z-Arg w)/2|

となっているのですがなぜこうなるのでしょうか……?
その他の条件として、
|z|=|w|=1、z,w≠-1、(Im z)(Im w)≧0
も提示されてます
0470132人目の素数さん
垢版 |
2022/06/20(月) 12:38:16.52ID:4SEHQk75
お絵描きすればわかるんじゃないの
0472132人目の素数さん
垢版 |
2022/06/20(月) 15:40:00.08ID:VR6JOPwA
f(x)=
{e^(-1/x) (x>0)
0 (x≦0)}
が2階微分可能であることを示すときってどうすればいいかな?
微分の定義を使おうにも0/0の不定形になって極限値が出てこない
0473132人目の素数さん
垢版 |
2022/06/20(月) 16:19:08.06ID:CgzmwIvi
>>472
n階導関数が全てある多項式P(x)をとって

P(1/x)e^(1/x) (x>0)
0 (x≦0)

の形になる事を帰納法で示す
0474132人目の素数さん
垢版 |
2022/06/20(月) 17:16:33.40ID:HUegpZIO
>>473
帰納法かなるほど
それなら微分可能だと言えるのか
微分の定義を使って示すならどうなるかな
0476132人目の素数さん
垢版 |
2022/06/20(月) 18:59:12.32ID:K0ykCOif
松坂和夫著『集合・位相入門』

『現代数学概説II』は難しい本なのかと思っていましたが、今パラパラ見てみたら、
『集合・位相入門』よりもやさしい本ですね。

『集合・位相入門』って結構抽象的ですよね。

松坂さんがどの本を参考にしたのか知りませんが、その本がそうだからだと思います。
0477132人目の素数さん
垢版 |
2022/06/21(火) 11:18:50.95ID:SRLpBDC3
tan(π/4+i)の値を求めようとしても(1+isinh2)/cosh2にならない
何回やっても1+isinh2になるんだけど1/cosh2はどこから出てきたんだ……
0478132人目の素数さん
垢版 |
2022/06/21(火) 14:35:32.80ID:70lQIs+s
>>477
分母を(cosh1)^2-(sinh1)^2にしてるんじゃないの
マイナスじゃなくてプラスだぞ
マイナスにすると1になって分母がなくなる
0479132人目の素数さん
垢版 |
2022/06/21(火) 16:29:48.91ID:dTXXI6b2
tan(π/4+i)
=(tan(π/4) + tani )/(1+tan(π/4)tani)
=( cosi + sin i)/( cosi - sini )
= (1 + 2 sini cosi )/( cos²i - sin²i )
= (1 + sin2i)/ cos2i)
= ( 1 + i sinh 2)/ cosh 2)
0480132人目の素数さん
垢版 |
2022/06/21(火) 21:02:04.87ID:H+1sk9JM
トポロジーの基礎 上下 単行本 2022/6/17
河澄 響矢 (著)

なんか難しいといっている人がいますね。
本屋で見てから注文すれば良かったかもしれません。
明日、届く予定です。
0481132人目の素数さん
垢版 |
2022/06/21(火) 23:50:57.97ID:V+iS2Znh
有理関数の部分分数分解して積分の話ですが、そもそも分母が因数分解できなければ話にならないですよね?
5次以上の代数方程式は解の公式がない訳ですから
0482132人目の素数さん
垢版 |
2022/06/22(水) 06:54:13.43ID:KFyhpcRD
実係数多項式は1次と2次の実係数多項式の積で表されるってガウスせんせーが言ってた
0483132人目の素数さん
垢版 |
2022/06/22(水) 10:07:09.86ID:1RlaUqai
N,Zを自然数、整数の集合として
f:Z→{0,1}が
∀M∈N ∃C_M∈N ∀x,y∈Z ∃z∈Z  |y-z|<C_M and (f(x),f(x+1), ... ,f(x+M))=(f(z),f(z+1), ... ,f(z+M))
を満たすとき
{Σ[|x|<N] f(x)}/N はN→∞で収束しますか?
0485132人目の素数さん
垢版 |
2022/06/22(水) 10:31:28.03ID:HJ9uZz2a
>>484
どんな遠く(y)にもってことでしょ
0486132人目の素数さん
垢版 |
2022/06/22(水) 10:40:08.52ID:HJ9uZz2a
状況分かりにくいけど
Zで番号付けられたパターンの
どの有限巾部分も
ある程度の間隔で繰り返している?
0488132人目の素数さん
垢版 |
2022/06/22(水) 10:48:22.89ID:h9wDabWL
>>483
何を考えているときに出てきた問題ですか?
0489132人目の素数さん
垢版 |
2022/06/22(水) 10:48:40.60ID:+IMkswc/
そもそも条件満たす非有界列も思いつかんな
どんな話の中で出てきたのかわからないとなんとも言えんな
ただの数オリ的な話?
0490132人目の素数さん
垢版 |
2022/06/22(水) 10:53:30.13ID:HJ9uZz2a
そんなパターンがあったとして
一定の有限巾パターンを
適当な間隔で差し挟んだとしても
条件を満たさないかな
もしそうなら
段々間隔広げてパターン差し挟んだ極限が
この条件満たさないかな(ちょっと望み薄かも)
もし極限がこの条件満たすなら
差し挟むパターンで{Σ[|x|<N] f(x)}/N
つまりパターンの「濃度」を変動させることが出来そうだけど
0491132人目の素数さん
垢版 |
2022/06/22(水) 10:57:08.87ID:HJ9uZz2a
>>489
f(x)が周期関数なら条件満たす
0492132人目の素数さん
垢版 |
2022/06/22(水) 11:03:45.44ID:HJ9uZz2a
でも周期関数なら濃度は収束する
0493132人目の素数さん
垢版 |
2022/06/22(水) 11:11:34.94ID:HJ9uZz2a
fの定義域をZじゃなくてN
つまり数列で考えてみたらどうかな
これだと反例作れるかも?無理かな?
0494132人目の素数さん
垢版 |
2022/06/22(水) 11:20:51.26ID:HJ9uZz2a
あるいは無理数の2進展開はこの条件満たさないこと示せれば(望み薄)
0496132人目の素数さん
垢版 |
2022/06/22(水) 13:40:50.62ID:HJ9uZz2a
>>495
f:Z→{0,1}
0499132人目の素数さん
垢版 |
2022/06/22(水) 14:01:20.72ID:lxyPs7jb
ダメやな
そんな簡単に証明も反例も出そうにない
そもそもちゃんと答えが出る保証がないならこれ以上考えるのは時間の無駄やな
0500132人目の素数さん
垢版 |
2022/06/22(水) 14:26:38.97ID:N+O0UXW0
二進整数環で0101…と表される元をaとするとき、整数nに対してa+nを考えて、初めて1が現れる桁数によってfを定める、とかでできないかな
0501132人目の素数さん
垢版 |
2022/06/22(水) 14:28:50.35ID:zk7J1GTG
てかそもそも答えあるのかどうかすらわからんやん?
これが面白い問題スレとかならヒントもらうとかできるけどここではそれもできんし
時間の無駄だよ
0502132人目の素数さん
垢版 |
2022/06/22(水) 14:46:53.22ID:HJ9uZz2a
時間の無駄だと思う人は考えなければ良いだけ
強制されてるわけではない
0503132人目の素数さん
垢版 |
2022/06/22(水) 14:47:56.55ID:HJ9uZz2a
>>501
>これが面白い問題スレとかなら
ああ確かにそっちのが適当だな
0504132人目の素数さん
垢版 |
2022/06/22(水) 14:59:21.57ID:bjvo/Led
>>503
そう、あっちのスレはちゃんと答えある問題というのが前提で解らなければヒントとかお願いしたりもできる
それでみんなで面白い問題出しあって数学を“楽しむ”場になっててそれなりに便所の落書きながらも文化的な空気もある
数学なんて一見答えが出そうに見えても答えでない問題なんか山のようにあるしそんな思いつきの問題ペタペタ貼ってくるやつなんかアホほどいる
そういうのにどれだけアホな思いさせられたかキリがない
今回のもその類である可能性があるんだから時間の無駄
0505132人目の素数さん
垢版 |
2022/06/22(水) 16:46:19.82ID:HJ9uZz2a
>>504
>あっちのスレはちゃんと答えある問題というのが前提
とするとあっちも不適だな
やっぱこっちか
0506132人目の素数さん
垢版 |
2022/06/22(水) 16:57:43.62ID:HJ9uZz2a
周期関数なら条件は満たすがパターン濃度(勝手に名付けてすまんが{Σ[|x|<N] f(x)}/Nのことね)の極限値が存在する
周期関数で無い例で作らねばならないが
もし周期的で無くてそういうパターンがあったとすると
x>0だけ見れば無理数を定義することになる
(有限小数は条件を満たさないから)
この条件を満たすものが周期関数に限るような気はしないけど
それが証明できてもおかしくはない
おかしくはないが面白みも無い
(0,1)区間の無理数で2進展開がどの部分もある程度の周期っぽい繰り返しを持つものがあると割と面白いかも
(もしかしたらそういうものの全体が非可算で非連続濃度な集合になったりしてと妄想)
0507132人目の素数さん
垢版 |
2022/06/22(水) 17:31:37.48ID:h9wDabWL
出題者が書き逃げしている問題は解かないのが吉
0508132人目の素数さん
垢版 |
2022/06/22(水) 17:41:05.05ID:yE5K70yo
まぁなんかの教科書なりなんなりで「今のオレの知識では解けないけど解き方知ってる人いるかもしれない」で未解決かもしれない問題貼るのは普通だしいいんだけど、ちゃんと答え用意されてる風に書いてきて実は答えもなんもないのが紛れてくるのが厄介
答え持ってなくて解けないかもしれないならその旨書くのが当たり前やろと思う
大学まで数学勉強してきてまだそんな事わからんのかなと思う
0509132人目の素数さん
垢版 |
2022/06/22(水) 18:18:16.99ID:GpuutDrS
これはスレチでしょうか
https://ja.wikiversity.org/wiki/%E6%9C%89%E9%99%90%E7%BE%A4%E3%81%AE%E5%A4%9A%E9%A0%85%E5%BC%8F%E6%99%82%E9%96%93%E4%BA%88%E6%83%B3

この漸近式が知られているらしくて、
https://i.imgur.com/HkIKwQ5.png

実際は、

位数1の群は1個
位数2の群は1個
位数4の群は2個
位数8の群は5個
位数16の群は14個
位数32の群は51個
位数64の群は267個
位数128の群は2,328個
位数256の群は56,092個
位数512の群は10,494,213個
位数1024の群は49,487,365,422個

のようです
0510483
垢版 |
2022/06/23(木) 06:34:09.89ID:+Fz1yPHJ
すいませんレスが遅れて申し訳ありません
>>483は元々は周期関数や適当な準周期関数で確かめて成り立つ性質として確認出来る物なのですが
周期関数や適当な準周期関数では>>506さんのう言う濃度の極限値が存在するという性質も確認出来て
前者の性質から後者を導く事は出来ないかという感じで質問させて頂きました
解決に導くアイデアは本当に自分には無くてここで聞こうと思いました
0511132人目の素数さん
垢版 |
2022/06/23(木) 09:48:28.98ID:q8FpNqr/
N→{0,1}バージョンだけど収束しない例ができたかも。勘違いだったらすまん。

十分早く無限大に発散する整数列c_nを取って、次のように0,1の有限列の列a_n, b_nを定める:
a_1 = 0, b_1 = 1,
a_{n+1} = a_n b_n (a_n)^{c_n},
b_{n+1} = b_n a_n (b_n)^{c_n}.
(積の形に書いてるけど、有限列を並べるという意味)
a_{n+1}がa_nで始まってることに注意して、自然数mに対してa_mのm番目の数をf(m)とする。
0512132人目の素数さん
垢版 |
2022/06/23(木) 10:26:40.64ID:YWNtBQte
>>511
c=1 2 4 8 16 …
a2=0 1 0^1=0 1 0
b2=1 0 1^1=1 0 1
a3=0 1 0 1 0 1 0^2 1^2 0^2=0 1 0 1 0 1 0 1 0 0 1 0
b3=1 0 1 0 1 0 1^2 0^2 1^2=1 0 1 0 1 0 1 0 1 1 0 1
てこと?
前の有限数列を頭に持つからこれら全部の極限みたいなものね
anはbn+1に1ヶ所しか出てきてないけど大丈夫かね?次に出てくるところがbnの巾のcn倍離れたところでしょ?cn→∞ならドンドン離れていかない?
0513132人目の素数さん
垢版 |
2022/06/23(木) 10:27:44.41ID:YWNtBQte
>>512
>a3=0 1 0 1 0 1 0^2 1^2 0^2=0 1 0 1 0 1 0 1 0 0 1 0
>b3=1 0 1 0 1 0 1^2 0^2 1^2=1 0 1 0 1 0 1 0 1 1 0 1
a3=0 1 0 1 0 1 (0 1 0)^2=0 1 0 1 0 1 0 1 0 0 1 0
b3=1 0 1 0 1 0 (1 0 1)^2=1 0 1 0 1 0 1 0 1 1 0 1
0514132人目の素数さん
垢版 |
2022/06/23(木) 12:38:38.52ID:zJMxAtKL
トポロジーの基礎
河澄 響矢 (著)

なぜか「下」だけ先に届きました。

参考文献にJames R. Munkresさんの『Elements of Algebraic Topology』がありました。

James R. Munkresさんの『Elements of Algebraic Topology』は、去年、アマゾンで新品を5000円以下で買いました。
0515132人目の素数さん
垢版 |
2022/06/23(木) 12:44:44.84ID:35Zvm3+4
「下」だけ先に届くなんて、河澄響矢さんは大丈夫な人なのでしょうか?
0516132人目の素数さん
垢版 |
2022/06/23(木) 13:30:08.03ID:q8FpNqr/
>>512
a_nと同じ幅の列だと anan, anbn, bnan, bnbn の一部になっているようなものしか出てこないけど、これらは全部a_{n+2}, b_{n+2}には出てくる。

nが0以下の時も、a_mの長さが-n+1以上になるmを取ってa_mの後ろから-n+1番目をf(n)とする、でいけそう。
0517132人目の素数さん
垢版 |
2022/06/23(木) 13:59:38.60ID:YWNtBQte
>>516
an+1の中でのanの離れ具合はbn巾
bn+1の中でのanの(境界からの)離れ具合はbn巾のcn倍
an+2の中でのanの離れ具合はbn巾のcn倍で
bn+2の中でのanの離れ具合はbn巾の1+cn倍程度
ここから先はずっとbn巾の1+cn倍程度か
なるほど
良さげね
これのパターン濃度が収束しないのは何でかな?
0518132人目の素数さん
垢版 |
2022/06/23(木) 14:07:04.72ID:YWNtBQte
0↔1の入れ替えをtで表すと
a1=1
an+1=an.(an)t.(an)^.cn
かな
帰納的に
bn=(an)t
から
bn+1=bn.an.bn^.n=(an)t.(an)tt.(an)t^.cn=(an.(an)t.((an)^.cn)t=(an+1)t
が示せる
0519132人目の素数さん
垢版 |
2022/06/23(木) 14:12:16.30ID:YWNtBQte
cn=1,2,4,8,16,…のとき
a1=1
a2=101
a3=101010101101
a4=101010101101010101010010101010101101101010101101101010101101101010101101
Σf/N=1, 1/2, 2/3, 1/2, 3/5, 1/2, 4/7, 1/2, 5/9, 3/5, 6/11, 7/12, → 1/2にならない??
0520132人目の素数さん
垢版 |
2022/06/23(木) 14:25:32.01ID:q8FpNqr/
anの幅をdn, anに出てくる1の数をsnとするとき、sn/dn が1/2でない値αに収束するくらいc_nは速く発散するとすると、N=dnでのパターン濃度はαに収束、N=2dnでは1/4 + α/2に収束、となって列全体は収束しないことがわかる。
0521132人目の素数さん
垢版 |
2022/06/23(木) 14:37:22.39ID:dcPOpSXd
>Witold Hurewicz著『Lectures on Ordinary Differential Equations』という本を持っていますが、この本はどうですか?


何この不気味な質問
0522132人目の素数さん
垢版 |
2022/06/23(木) 14:45:38.22ID:YWNtBQte
>>520
>sn/dn が1/2でない値αに収束するくらいc_nは速く発散する
an+1=an.(an)t.(an)^.cn
から
sn+1=sn+(dn-sn)+cnsn=dn+cnsn
dn+1=(2+cn)dn
sn+1/dn+1=(dn+cnsn)/(2+cn)dn=1/(2+cn)+(sn/dn)(cn/(2+cn))=1/(2+cn)+(sn/dn)/(1+2/cn)
1/(2+cn)→0で1+2/cn→1だけど
これ収束はしないのかな
0523132人目の素数さん
垢版 |
2022/06/23(木) 14:50:57.57ID:YWNtBQte
>>520
ああsn/dnが1/2じゃない値αに収束するぐらいにか
その場合
N=2dnでは
an.(an)t
の中に1はdn個だから(そうじゃない?)
dn/2dn=1/2

お見事!
0524132人目の素数さん
垢版 |
2022/06/23(木) 15:21:52.70ID:YWNtBQte
両側にするにはどうするかな
an+1=an.bn.(an)^.cn
の一番右がanだから
これ右にも延ばしていけるよね
うーんでもそこで継げないか?
0525132人目の素数さん
垢版 |
2022/06/23(木) 15:25:32.54ID:YWNtBQte
大丈夫じゃ無いかな
0526132人目の素数さん
垢版 |
2022/06/23(木) 15:26:50.58ID:YWNtBQte
cn>>1だし
0527132人目の素数さん
垢版 |
2022/06/23(木) 15:28:53.62ID:YWNtBQte
>>524
>これ右にも延ばしていけるよね
左(x<0の側)に延ばすでした
0528483
垢版 |
2022/06/23(木) 23:00:43.74ID:+Fz1yPHJ
>>511
反例を作って頂きありがとうございました!
その例はとてもありがたい例になります…
0529132人目の素数さん
垢版 |
2022/06/24(金) 09:59:02.72ID:AgNZocdy
松坂和夫著『集合・位相入門』

「一般に、開集合系(またはそれに付随して定まる閉集合系、近傍系等)にもとづいて
定義される諸概念('位相的概念')については、 (S, O), (S', O') の一方の上で成り立つ
ことは、他方の上でも( f または f^{-1} でうつしかえた対象に対して)そのまま成り
立つのである。」

と書いています。それにもかかわらず、例えば、

f(i(M)) = i(f(M))

が成り立つことなどについて、

「これらのことのくわしい検証は練習問題とする。」

などと書いています。

一般に成り立つならば、それを証明してみせればいいだけの話ではないでしょうか?
0531132人目の素数さん
垢版 |
2022/06/24(金) 12:35:17.39ID:AgNZocdy
>>514

トポロジーの基礎
河澄 響矢 (著)

「上」も先程、届きました。

英語のタイトルが『Basics of Algebraic Topology』ですね。
0532132人目の素数さん
垢版 |
2022/06/24(金) 14:11:52.89ID:AgNZocdy
Algebraic Topologyで、代数学の深い結果を使うことってありますか?
0533132人目の素数さん
垢版 |
2022/06/24(金) 22:55:50.46ID:GTgFojqp
>>532
>代数学の深い結果
とは?
0534132人目の素数さん
垢版 |
2022/06/25(土) 08:51:20.06ID:N/26ftL/
>>533
「代数学の基本定理」かな?でもあれは代数学ではなく位相数学の定理だよな
532は代数学の教科書の後ろの方に書いてある定理、というほどの意味かな

個人的には、スペクトル列などが代数的には深いというか、一見ややこしい話だな
0535132人目の素数さん
垢版 |
2022/06/25(土) 09:11:52.52ID:1otKhIs4
algebraic topologyの括りがでかすぎる
学部の教科書レベルならせいぜいPID上有限生成加群の構造定理くらい?じゃないかな

もちろんそれがalgebraic topologyの全てではないし可換環論や代数幾何を使うこともある
0536132人目の素数さん
垢版 |
2022/06/25(土) 10:15:25.49ID:fIANx7BZ
>>534
>「代数学の基本定理」かな?でもあれは代数学ではなく位相数学の定理だよな
複素関す論の定理じゃ無いの?
0537132人目の素数さん
垢版 |
2022/06/25(土) 10:25:24.46ID:SQSzpSXj
ま、松坂君には関係ないわな
代数学の基本定理もギブアップしてるのに
代数学の基本定理もわからないんじゃ何にも厳密な議論できんわな
0539132人目の素数さん
垢版 |
2022/06/25(土) 13:52:29.12ID:GE9l1rXX
>>537

代数学の基本定理の証明は簡単ですよね。

複素関数論の知識を必要としない証明が齋藤正彦著『線型代数入門』などにあると思います。
0540132人目の素数さん
垢版 |
2022/06/25(土) 14:02:06.01ID:gH6CpD4T
馬鹿ほど「これは簡単」と言い出す
どこに厄介な問題があるかわかっていないから
0541132人目の素数さん
垢版 |
2022/06/25(土) 15:18:46.28ID:GE9l1rXX
>>540

厄介な問題など全くないと思います。
0542132人目の素数さん
垢版 |
2022/06/25(土) 15:20:57.06ID:GE9l1rXX
多少面倒なのは、

f(a) ≠ 0 ならば、

|f(b)| < |f(a)| となるような b ∈ C が存在することの証明の部分ですかね。
0543132人目の素数さん
垢版 |
2022/06/25(土) 15:29:19.35ID:GE9l1rXX
証明のアウトラインは非常にシンプルです。

(1) C ∋ x → |f(x)| ∈ R

という関数は連続関数である。

(2) この関数は |x| を大きくすれば、いくらでも大きな値を取る。

(3) この関数は連続関数だから、閉円板上で最小値をとる。

(4) (2), (3)より、この関数は、C 上で最小値を取る。

(5) f(a) ≠ 0 ならば、 |f(b)| < |f(a)| となるような b ∈ C が存在する。

(6) この関数が x = a で最小値を取るとする。
もしも、 |f(a)| ≠ 0 ならば、(5)により、矛盾がおきる。

(7) よって、この関数の最小値は 0 である。

(8) |f(a)| = 0 だから f(a) = 0 である。
0544132人目の素数さん
垢版 |
2022/06/25(土) 15:58:15.23ID:GE9l1rXX
>>543

訂正します:

(2) lim_{|x|→+∞} |f(x)| = +∞
0545132人目の素数さん
垢版 |
2022/06/25(土) 16:00:12.79ID:NhXKlyKd
メチャクチャwwwwwww
そんな証明が通用するならexp(z)=0も零点持つわwww
代数学の基本定理すら理解できてないで他人の書いた証明はボロクソにいう
アホじゃないのwwwwwwwww
0546132人目の素数さん
垢版 |
2022/06/25(土) 20:04:46.47ID:GE9l1rXX
このアウトラインは齋藤正彦著『線型代数入門』に書かれている証明のアウトラインです。

各ステップの詳細を埋めれば、それで代数学の基本定理の証明になっていることは明らかです。
0547132人目の素数さん
垢版 |
2022/06/25(土) 20:26:33.47ID:ndMGn+yX
その「詳細」にいつも文句を付けてる馬鹿がアスペのお前の日常

自分のレスについては細部を追及されないように逃げを打つのがアスペのお前の日常
0548132人目の素数さん
垢版 |
2022/06/25(土) 20:55:57.24ID:YXcgrrgj
>>546
そんなわけあるかバーカ
じゃあexp(-|z|)は連続じゃないんか?
lim exp(-|z|) = 0 にならんのか?
なるやろ?
そんな証明があってるならexp(-|z|)も零点持ってしまうやろ
アホですか?
結局自分勝手に「つまらない回り道してるな、本質はこれでいい」とか勝手な、しかもデタラメな理解してる
他人にはメチャクチャ厳しく、自分の論理はガダガタ
恥ずかしないの?
0549132人目の素数さん
垢版 |
2022/06/25(土) 21:01:46.69ID:bJzQpuCj
齋藤正彦さんは大丈夫な人なのでしょうか?
0550132人目の素数さん
垢版 |
2022/06/26(日) 09:14:13.25ID:9kFFH/Uk
>>549
死者を鞭打ってる?
0552132人目の素数さん
垢版 |
2022/06/27(月) 07:59:20.06ID:Ril0UddK
松坂和夫著『集合・位相入門』


(A_λ) λ∈Λ を1つの与えられた集合族とするとき、 Λ で定義された写像 a で、
次の条件

(*) Λ のどの元 λ に対しても a(λ) = a_λ ∈ A_λ

を満足するようなもの全体の集合、いいかえれば、条件(*)を満たす族 (a_λ) λ∈Λ 全体の集合を、
集合族 (A_λ) λ∈Λ の直積(または単に積)といい、記号

Π_{λ∈Λ} A_λ

で表わす。


などと書いています。

写像について、終集合が異なれば異なる写像であるとかそんなことを強調していたのに、
ここでは、終集合について何も触れていません。

松坂和夫さんは一体何を考えていたのでしょうか?
0554132人目の素数さん
垢版 |
2022/06/27(月) 10:37:47.27ID:Ril0UddK
a の終集合は ∪ A_λ ということでいいんですか?

ある集合 X の部分集合族の和集合というのは違和感がないのですが、
互いに何の関係もない A_λ の和集合というのは考えてもOKなんですか?
0555132人目の素数さん
垢版 |
2022/06/27(月) 10:49:13.46ID:ugHyeFez
その本で想定されてる読者層なんてその程度なんだよ
そんな事厳密に議論しようとすれば当然集合論の公理持ち出さんと無理、しかしそこまでは無理だから最低数学の教科書読める程度のレベルになるあたりを探って書いておられるんだよバーカ
しかし“写像f”を例えば{<1,2>,<3,4>}などと定義してしまうと{[,2}から{3,4}への写像なのか{1,2}から{3,4,5}への写像なのかわからないからそれではダメなので<{1,2},{3,4,5},{<1,2>,<3,4>}>のようにしないとダメになる
この辺の正確な議論ももちろん集合論の公理までやらなけりゃ出来るわけないから保留さてんだよバーカ
いつまでもいつまでもこんなしょうもないレベルの話しばっかり
お前には無理だって
0556132人目の素数さん
垢版 |
2022/06/27(月) 11:59:51.98ID:Ril0UddK
松坂和夫著『集合・位相入門』

直積位相の定義も抽象的ですね。

世界標準のJames R. Munkresさんの本のほうがずっと分かりやすいですね。
0558132人目の素数さん
垢版 |
2022/06/27(月) 13:31:34.74ID:iHdeoxdC
>>554
それです
0559132人目の素数さん
垢版 |
2022/06/27(月) 13:35:36.10ID:iHdeoxdC
関係は
AとBとA×Bの部分集合の3つ組み
その特別なものが写像
とはいえB⊂Cと拡大したときも
同じと見なすか違うと見なすか
場合場合で適当に考えてるよ
0560132人目の素数さん
垢版 |
2022/06/27(月) 15:53:36.71ID:ALXBfuMb
質問失礼いたします。

ファイバーにレーザー光を当てるとファイバーに沿って少しだけ光が広がるという論文に関するものです。
ファイバーに当てていないレーザー光の画像と
ファイバーに当てているレーザー光の画像
の二枚が実験によって得られているものとします。
この二つを使って、ファイバー方向に光がどのくらい運ばれたか求めるのが目標の論文です。

ファイバー方向にどのくらい光が広がっているかは分散の差の平方項(式①)によって求められるようなのですが、
知識不足により例えば赤丸で示した部分をどのようにして得ればいいのか見当もつきません。
論文にはこうあります
「2次元ガウスフィッティングルーチンを用いて、
励起光スポットとファイバーから発せられる発光プロファイルを定量的に比較した。」
励起光スポットというのはレーザー光の元々の形状の事、
レーザーから発せられた発光プロファイルというのは、レーザー光を照射されて光っているファイバーの事です。

imageJという画像解析ソフトを使っていますが、それ以前に何をめざせばよいのかもわからず悩んでおります。

座標変換したあと長軸方向と短軸方向に白で示されたドットがいくつあるか数えるとかでしょうか?

どなたかアドバイス等いただけますと幸いです。
化学知識というよりは私の実践的な統計の知識が不足していると予想したので、
こちらに書き込みさせていただきました。
よろしくお願いいたします。

引用元
Supporting Information for
Direct Measurement of Energy Migration in Supramolecular Carbocyanine Dye Nanotubes
Katie A. Clark


https://i.imgur.com/zUUB4uG.jpg
0561132人目の素数さん
垢版 |
2022/06/27(月) 19:20:26.47ID:IAhb/zB0
板間違えてますよ
0562560
垢版 |
2022/06/28(火) 12:35:30.11ID:Gr8i2A0V
>>561
そうなんですか、ガウス関数とか統計なので数学板に書き込んでしまいました
理科系のどこかに改めて書き込んでみたいと思います
教えてくださりありがとうございました
0563132人目の素数さん
垢版 |
2022/06/28(火) 15:33:00.80ID:WwTIvkmw
>>562
数学の問題の部分だけ抽出してくれないと
物理の問題のまま提示されても困るてこと
0565132人目の素数さん
垢版 |
2022/06/29(水) 11:39:17.54ID:dAoc2z5/
twitterで、ある整数論が専門の方が「日本人が最初に触れる同値類は有理数」とツイートしておられたのですが、これは間違いでしょうか?
有理数を「日本人が触れる同値類」とするなら、整数のほうが必ず先にくると思うのですが
0566132人目の素数さん
垢版 |
2022/06/29(水) 11:41:43.65ID:MwK3r6lC
>>565
引き算でグロタンディーク構成的な導入もやるので自然数→整数のほうが先だな。
0567132人目の素数さん
垢版 |
2022/06/29(水) 11:46:01.71ID:dAoc2z5/
>>566
自然数を可換モノイドと考えて、同値類全体の集合となるグロタンディーク群は整数そのものなので、やはりそうですよね
ありがとうございます
0568132人目の素数さん
垢版 |
2022/06/29(水) 12:06:36.60ID:IxaIE1jh
そんな奇天烈なことしなくても有理数は分数でも小数でも表せるし約分とかできて色々な表し方できるからってことじゃないんですか?
0569132人目の素数さん
垢版 |
2022/06/29(水) 12:06:51.52ID:MwWGH4Y4
負数に触れるのは中学からなので有理数で合ってる
約分倍分が同値類の考え方
0570132人目の素数さん
垢版 |
2022/06/29(水) 12:32:56.28ID:64Xwjj0f
>>567
2-3=3-4=-1を同値類とは見ないのが普通よな
0571132人目の素数さん
垢版 |
2022/06/29(水) 12:34:23.63ID:64Xwjj0f
それにあまり原理に拘ると自然数だって同値類だとか主張して已まない
0572132人目の素数さん
垢版 |
2022/06/29(水) 14:39:12.77ID:mW3sg+fX
>>558-559

ありがとうございました。
0573132人目の素数さん
垢版 |
2022/06/29(水) 14:41:04.79ID:mW3sg+fX
半額で買った、Sheldon Axler著『Measure, Integration & Real Analysis』が届きました。
0574132人目の素数さん
垢版 |
2022/06/29(水) 15:09:47.58ID:mW3sg+fX
>>559

確かにそうですね。

A_λ := N
B_λ := Z

Π A_λ の終集合は N
Π B_λ の終集合は Z
Π (A_λ ∩ B_λ) の終集合は N

なので、 (Π A_λ) ∩ (Π B_λ) は空集合ですが、 Π (A_λ ∩ B_λ) は任意の λ に 1 を対応させる写像を含むので、空集合ではありません。

よって、以下の問題は成り立ちません。

松坂和夫著『集合・位相入門』 p.51 問題9:

(Π A_λ) ∩ (Π B_λ) = Π (A_λ ∩ B_λ)

を示せ。
0576132人目の素数さん
垢版 |
2022/06/30(木) 12:15:48.09ID:6nvzOuc9
>>569
ああ、負の数より前に分数が登場するから有理数が先ということですね
昔すぎて忘れてました、ありがとうございます
0577132人目の素数さん
垢版 |
2022/06/30(木) 12:22:39.22ID:kmfhv5Z6

自分の非を認めて素直に謝れる人、久しぶりに見ました
いいものを見させていただきました
0578132人目の素数さん
垢版 |
2022/06/30(木) 13:09:26.61ID:6Y3AEyiS
三角形とかの平面図形の合同な図形の集合のほうが有理数よりも早くあらわれる同値類ではないでしょうか?
0579132人目の素数さん
垢版 |
2022/06/30(木) 14:19:54.17ID:8Swi0Rjf
>>577
非?何様?
0580132人目の素数さん
垢版 |
2022/06/30(木) 18:56:37.63ID:Bqb23/3o
>>578
相似は同値関係かな
三角形の三辺の比を1:a:bとして相似による同値関係で割ると基本領域は複雑な形になる
0581132人目の素数さん
垢版 |
2022/06/30(木) 19:05:53.51ID:IGUf2PqM
>>579
大卒理系 陛下 だよ
0582132人目の素数さん
垢版 |
2022/06/30(木) 19:37:31.43ID:z0VixncA
1 ご冗談でしょう?名無しさん 2022/04/01(金) 14:09:54.90 ID:Bj6jftJZ
James R. Munkres著『Analysis on Manifolds』を読破して、電磁気学に備えるスレッド


キチガイな質問をするだけでなくキチガイなスレも立てたりしてるのな
0583132人目の素数さん
垢版 |
2022/07/01(金) 04:49:27.12ID:XZuzU5RQ
逆三角関数atanのテイラー展開なのですが
atan(x)のxの範囲が-1<x<1までしかないのが解せないです
-pi<x<piにはならないものなのでしょうか
0584132人目の素数さん
垢版 |
2022/07/01(金) 06:24:37.79ID:SVfF+p58
>>583
何でπ?
0585132人目の素数さん
垢版 |
2022/07/01(金) 07:17:03.19ID:PurNHiBR
z=±iで非正則だから
0586132人目の素数さん
垢版 |
2022/07/01(金) 08:59:50.30ID:+rjt0MEY
CW複体にならないcell複体の例を考えているのですが
weak topologyを持つけどclosure finiteでない空間ってどのようなものがあるのでしょうか
0587132人目の素数さん
垢版 |
2022/07/01(金) 10:13:08.25ID:NDChOMpW
ちゃんと確かめてないけど、ハワイアンイヤリングに2-セルを貼り付けた空間とかどうでしょう
0588132人目の素数さん
垢版 |
2022/07/01(金) 21:20:07.09ID:bqB2f111
a/bでaを分子と呼びbを分母と呼ぶみたいな感じで、
a mod n
においてaとnって何か名称ありますか?
0589132人目の素数さん
垢版 |
2022/07/01(金) 21:44:59.22ID:SVfF+p58
>>588
0590132人目の素数さん
垢版 |
2022/07/02(土) 07:34:11.03ID:ZdpujcHk
これもスレチかもしれないんですが、

有限集合に、全体集合に対する補集合を表しているか否かのブール値を付与すれば
ANDもORもNOTも演算が可能な一応のなにがしかが得られる、みたいなアイデアを、
もうウン十年くらい前に思いつきました

すなわち、

集合Aを {1, 3, 6} とする
集合Bを {2, 3, 5}以外 とする

AとBの積集合は {1, 6} である
AとBの和集合は {2, 5}以外 である

みたいなアイデアなのですが、これには意味があったりなかったりしますか
0591132人目の素数さん
垢版 |
2022/07/02(土) 07:51:31.29ID:ZdpujcHk
「そんなものは学問ではない。ただの技術だ」と言う人もいて、
実際そうかもしれないんですが
0592132人目の素数さん
垢版 |
2022/07/02(土) 10:25:45.31ID:tZ9wQoKX
無限集合上のブール代数で、有限集合と補有限集合を集めたものが部分代数になってるってこと?
0593132人目の素数さん
垢版 |
2022/07/02(土) 10:55:26.55ID:ZdpujcHk
「部分」ではないかもしれません。この方式で「空集合の補集合」も表せます
0594132人目の素数さん
垢版 |
2022/07/02(土) 11:27:50.86ID:RTa1Ki+1
>>593
>「部分」ではないかもしれません
なんで?全体集合が無限集合なら部分集合とその補集合がどちらも無限集合になることがあるから必ず部分ブール代数でしょ?全体集合が有限集合なら部分集合全体と変わらないけどそのことを言っている?けれどその場合は特に面白みは無いし
>「空集合の補集合」
全体集合では?
0595132人目の素数さん
垢版 |
2022/07/02(土) 11:32:53.04ID:dFtMD75X
正しいとか意味があるとかはともかく、どこらへんがどう新しくてウリなの?
0596132人目の素数さん
垢版 |
2022/07/02(土) 11:35:47.93ID:ZdpujcHk
>>594
あ、今考えててわかりました
確かに「全ての偶数」とか「全ての4の倍数」とかはこの方式では表せないですね
0597132人目の素数さん
垢版 |
2022/07/02(土) 11:40:16.00ID:ZdpujcHk
>>595
ウリはもともと「例えばC言語とかでも実装できること」でした
0598132人目の素数さん
垢版 |
2022/07/02(土) 11:52:22.44ID:ZdpujcHk
曲がりなりにも無限集合を、コンピュータ言語風情が扱えてしまう、というのが新しいと思ったのです
数学的には色々あるだろうから「おかしな先例は作るまい」と思って実装はしませんでした
0599132人目の素数さん
垢版 |
2022/07/02(土) 12:05:21.50ID:RTa1Ki+1
>>598
>>592の書いているように有限集合とその補集合の全体の為す部分ブール代数ってことでよろしいのでは
0600132人目の素数さん
垢版 |
2022/07/02(土) 21:45:01.61ID:1drdpIz0
コンピューターが扱えるのは基本有限のものだけで、かろうじて手が出るのも可算無限集合くらい。
可算無限集合のブール代数は非可算だからお手上げだけど、有限集合とその補集合に限れば可算濃度だから扱ってる気にはなれる、って感じかな。
0601132人目の素数さん
垢版 |
2022/07/02(土) 23:03:12.70ID:ZdpujcHk
エンジニアの体感としては、
「じゃあ、『全ての偶数』が扱えてほしいですか」
と聞かれても
「いえ、あまり。そんなの使わずにやると思いますよ」
って感じですよね
0602132人目の素数さん
垢版 |
2022/07/02(土) 23:59:06.05ID:KGfJgKnJ
正規言語は可算無限集合(全ての可算無限集合ではないが)を扱っていて、
しかも集合演算で閉じている。そして、全ての偶数くらいなら、正規言語で表現可能。

コンピュータで正規言語を扱うときの対応物は(通常)正規表現なので、
正規表現を使えば、ある程度の可算無限集合が実質的に扱えている。

ただし、計算資源はコンピュータごとに有限なので、
そのコンピュータの資源量を上回る情報はそもそも入力すらできない。
それでも、プログラムの意図するところはある種の無限集合にちゃんと対応している。

なので、無限集合がある程度扱えても、別に何の不思議もない。
0603132人目の素数さん
垢版 |
2022/07/03(日) 14:40:39.29ID:/bAPTIHn
例えばaという文字が1億個くらい入力されてきた後、
最終的にaの個数が偶数だったか否かに対応するのには
ごくごく限定的なリソースしか必要としない
変数を1つだけ用意して現在の状態を覚えておくだけ
正規言語(有限オートマトン)のやっていることは基本的にそれと同等
0605132人目の素数さん
垢版 |
2022/07/04(月) 21:10:35.73ID:H/gCJ09w
質問です
0606132人目の素数さん
垢版 |
2022/07/04(月) 21:13:43.96ID:H/gCJ09w
質問です。乗法群としての準同型:一般線形群GL(2,c)→c(指標、一次元表現)
は必ず行列式のべき乗になっている、という有名らしい事実を
分かりやすく証明している文献を教えて下さいm(_ _)m
0609132人目の素数さん
垢版 |
2022/07/05(火) 06:10:47.12ID:Su2MwGaj
例えば
7人競輪って3連複5番人気まで、5点買いで買ってたらポイント還元やキャンペーン分は良い勝負だよな

例えば
1レース3連複10番人気まで10点買いで一番人気さえ来なければ、良い勝負だろ

で、ポイント還元やキャンペーン分プラスになったりするよな

キャンペーン当たりますよね


それか
3連複10番人気までの1つ、3連複1点買い、1点勝負とか、単勝一点買い勝負みたいで熱いよね
0610132人目の素数さん
垢版 |
2022/07/05(火) 08:44:03.21ID:ORT2WHyr
>>608
その貴方の挙げて下さった例も「一般線形群の指標は必ず行列式を経由する
(行列式の要素に分解できる)」は満たしてます。任意の準同型を合成させたら
それも再び一般線形群の指標になるのは自明な話かと思います。
引き続き証明が載っている文献のご教授お待ちしていますm(_ _)m
0611132人目の素数さん
垢版 |
2022/07/05(火) 09:43:51.85ID:Wk5Dstuk
息を吐くように問題を改造してる
0612132人目の素数さん
垢版 |
2022/07/05(火) 09:55:03.26ID:+lxda97r
>>610
偉そうな馬鹿
0613132人目の素数さん
垢版 |
2022/07/05(火) 09:56:10.52ID:ORT2WHyr
>>611
私も(質問させて頂いてる側なので)命題自体ハッキリ分からないので
そこ込みで証明が載っている文献をお聞きしていますが
挙げて下さった例はトリビアルなのが一目瞭然で
何故行列式がこんなにまで強い形でスッキリと関わってくるかの
本来素朴なはずの疑問には変わりありません
0614132人目の素数さん
垢版 |
2022/07/05(火) 10:18:25.51ID:+lxda97r
>>613
対人関係の距離の取り方がおかしいぞ
0615132人目の素数さん
垢版 |
2022/07/05(火) 10:31:31.91ID:erJHsFAM
懇切丁寧に無礼な振る舞いをする人か
0616132人目の素数さん
垢版 |
2022/07/05(火) 10:45:34.30ID:gjNv9qec
Cの乗法群は可換だから任意の準同型 GL(2,C) → C-{0} は可換化 GL(2,C)/[GL(2,C),GL(2,C)] を経由する。
GL(2,C) の交換子部分群は SL(2,C) で可換化は C-{0} に同型、この同型は行列式によって与えられるので、任意の準同型 GL(2,C) → C-{0} は行列式をとる準同型 det を経由する。

代数の教科書の演習問題とか探せば載ってるんじゃない?
0617132人目の素数さん
垢版 |
2022/07/05(火) 10:57:23.76ID:ORT2WHyr
>>616
ゆっくり理解したいので文献を出来れば教えて頂きたいですが
その説明では肝心の「この同型は行列式によって与えられる」の理由が
分かりません(理由が空白のように感じます)し
「GL(2,C) の交換子部分群は SL(2,C)」 も理由もよく分かりませんし
交換子部分群の話自体もあまり知りません
引き続き「証明が載っている文献」のご教授よろしくお願い致しますm(_ _)m
0618132人目の素数さん
垢版 |
2022/07/05(火) 11:29:07.84ID:/5nEDZFi
新作燃料もなかなかいい出来だね
0619132人目の素数さん
垢版 |
2022/07/05(火) 12:25:43.84ID:x8YjcH/V
このスレの回答者は馬鹿ばっかりだな。質問者は文献を教えてくれと言っている。解説してくれとは言っていない。文献を挙げられない奴は黙っているべき。
0620132人目の素数さん
垢版 |
2022/07/05(火) 12:53:33.15ID:+lxda97r
>>619
ブーメラン
0623132人目の素数さん
垢版 |
2022/07/05(火) 13:31:21.38ID:+lxda97r
0626132人目の素数さん
垢版 |
2022/07/05(火) 14:04:38.00ID:ORT2WHyr
>>619
私に寄り添って頂ける本当にコメ有難うございました。とても嬉しいです

>>620
スレッド全体の流れに価値をもたらそうとしている指摘は
ブーメランにならないと思います

>>621
私の質問は
「乗法群としての準同型:一般線形群GL(2,c)→c(指標、一次元表現)
は必ず行列式のべき乗になっている」
の証明が載っている文献をご提示お願いする事なので
交換子について今全く興味ありませんし(証明に必要である事が確定すれば
丁寧にフォローするかもですが)今の所私の質問内容と全く一切関係ないですし
そして証明方法もなるべく一番カラクリが直接的に分かる方法がいいです

>>624
良い文献がもし見つかればtext名を教えて下さい
私は探してもあまり見つけられませんでした。すごく素朴でかつ強い結果のはずなのに
見つからず残念です。(因みに出来れば洋書の方がいいです。)
表現を少しだけ特殊なモノに限定すればチラッとあるかもですが。

引き続き「証明が載っている文献」のご教授
知ってる方いらっしゃいましたらよろしくお願い致しますm(_ _)m
0627132人目の素数さん
垢版 |
2022/07/05(火) 14:18:15.89ID:ORT2WHyr
>>622,>>625
もし交換子の話が万が一仮に私の質問内容と関係あったとしても
非常に余計なモノを迂回した方法になりそうなので別の直接的な証明方法がいい
かもですね。そこに書いてある内容がちょっと非自明ぽいので。

ぶっちゃけゴリゴリ手計算のみによって私の質問内容を一応証明し切る事自体は
可能なのですが、しかしそのゴリゴリした計算の労力がまっとうな証明によって
たとえ減らせても代わりに別の余計なモノの証明に手間を取ってしまうなら
あまり意味がなくなりますので。
私の質問内容は非常に素朴で強力なので「本当のカラクリ」が分かってしまえば
非常にスッキリしたものになるはずではと思っています
0628132人目の素数さん
垢版 |
2022/07/05(火) 14:28:31.79ID:ORT2WHyr
本当に知りたい事は
終結式がx→(ax+b/cx+d)の一次分数変換によって
detAのべき乗を掛けた形にだけ変換するという内容です。
このスッキリする証明をどうやら Gelfand Kapranov「Discriminants, Resultants」
という本の中に見つけたのですがその証明の中に
「乗法群としての準同型:一般線形群GL(2,c)→c(指標、一次元表現)
は必ず行列式のべき乗になっている」らしき事実が使用されており
その説明が載ってないっぽいので、この度質問させて頂いてます
0629132人目の素数さん
垢版 |
2022/07/05(火) 14:38:40.77ID:KXBQbh+2
>>626
>交換子について今全く興味ありませんし
じゃ今この瞬間興味持ちなよ
>今の所私の質問内容と全く一切関係ないですし
貴方が関係を知らないだけじゃね?
>証明方法もなるべく一番カラクリが
>直接的に分かる方法がいいです
群の可換化が
「一番カラクリが直接的に分かる方法」
でないと言い切る理由は?
0630132人目の素数さん
垢版 |
2022/07/05(火) 14:39:20.71ID:+lxda97r
>>627
死ね
0631132人目の素数さん
垢版 |
2022/07/05(火) 14:43:43.83ID:KXBQbh+2
>>627
もしかして群を交換子で割ったら可換群になることが
貴方にとって全く非自明なの?
それって群論が初歩から分かってないってことだけど
だったら真っ先にそこから理解したほうがいいよ
初歩だから
0633132人目の素数さん
垢版 |
2022/07/05(火) 15:00:56.53ID:ORT2WHyr
>>629
>じゃ今この瞬間興味持ちなよ
質問内容と関係ありませんので
数学の膨大な全てに興味を持つ事を強要されたら人生が
いくつあっても足りないから
質問者は自身の質問内容にだけ興味を持ちたいと思います
むしろ質問と無関係な話を振って頂きたくなく思います

>貴方が関係を知らないだけじゃね?
>でないと言い切る理由は?

立証義務が逆だと思います
得体の知れないパンのような物体があった時
それが食べても安全だと立証する義務はお店であり
不安に思う客に「危険だと証明しろ」と迫るのはお門違い

>もしかして群を交換子で割ったら可換群になることが
>貴方にとって全く非自明なの?
GL(2,F_2) の交換子部分群なるものが SL(2,F_3)になる?所
などが自明ではなさそうと言いました
交換子部分群なるものの正確な定義もよく知りませんが今無関係なので
ググりたいとも思いません
0635132人目の素数さん
垢版 |
2022/07/05(火) 15:07:46.79ID:KXBQbh+2
>>633
>交換子部分群なるものの正確な定義もよく知りませんが
大学3年の群論で教わらなかった?
知らない奴はモグリと言われるほど
基本的なことだけど…
0636132人目の素数さん
垢版 |
2022/07/05(火) 15:09:10.79ID:ORT2WHyr
ヤフー知恵袋なら質問者の操作によって決して上位にあがらない劣悪な回答も
5ちゃんねるではどんどんスペースを埋めていく事が往々にして起こりますので
私の質問がちょっと埋もれ気味になってしまってるかもですので
>>606の質問を恐縮ですが念のため再掲示させて頂きます

質問です。乗法群としての準同型:一般線形群GL(2,c)→c(指標、一次元表現)
は必ず行列式のべき乗になっている、という有名らしい事実を
分かりやすく証明している文献を教えて下さいm(_ _)m
0637132人目の素数さん
垢版 |
2022/07/05(火) 15:11:30.70ID:KXBQbh+2
>>633
>今無関係なので
今貴方に関係が見えないことが
関係ないことの証明にはならないが?
0640132人目の素数さん
垢版 |
2022/07/05(火) 16:15:28.63ID:+lxda97r
>>639
616と622のリンク先で十分
0641132人目の素数さん
垢版 |
2022/07/05(火) 16:33:38.53ID:ORT2WHyr
>>640
質問者ですが
大変すみませんが、いい加減なウソを仰っしゃらないで頂けますか
質問が流れてしまいすので。ウソで荒らさないで下さい。
「GL(2,c)の交換子部分群がSL(2,c)」という誰かが挙げて下さった事実と
なぜ「一般線形群の一次元ベクトル空間への指標が行列式のべき乗に
なる」かとは現在の所全く無関係です
この事実からなぜ私の質問が導かれるかは全く意味不明でトンチンカンです

>>616←の方が急にこの事実を急に持ち出して説明をされておられますが
>>617←で既に指摘させて頂いた通り
「この同型は行列式によって与えられるので」の肝心の部分の証明が
全く空白なのでこの説明は全くナンセンスです

全く無関係な数学の話に「この同型は行列式によって与えられる」という文字列を
唐突に機械的に挿入しさえすればいくらでも生成出来てしまうたぐいの
無味乾燥な意味不明な文と同じレベルです
0642132人目の素数さん
垢版 |
2022/07/05(火) 16:53:18.47ID:V13Tb0VD
>この事実からなぜ私の質問が導かれるかは全く意味不明でトンチンカンです


もともとの>>606の疑問は指標が(det)^kの形になることだったろ、これにはすぐ反例>>608が出ている
それに対して行列式の冪乗ではなく経由云々の話にすり替えたのが意味不明でトンチンカンでございます

……もしかして冪乗の意味が分かってなかったパターン?
0643132人目の素数さん
垢版 |
2022/07/05(火) 17:04:10.43ID:ORT2WHyr
>>642

@
私の話がトンチンカンでない理由は
>>610,>>613←で說明済みです

A
A(肝心の「この同型は行列式によって与えられる」の理由が空白の話)
B(私の話)
C(それ以外のあらゆる話)
の3つの話があったとして、そもそもAの話がトンチンカンであるか否かの問題は
いくらB,Cがトンチンカンであろうがなかろうが無関係な話です
そういうのを話題そらしの詭弁というのだと思います
0644132人目の素数さん
垢版 |
2022/07/05(火) 17:39:10.88ID:Wzj9avDx
「この同型は行列式によって与えられる」を能動態に書き換えたら分かる鴨
0645132人目の素数さん
垢版 |
2022/07/05(火) 20:10:38.89ID:AOVCHV0H
>>626
>良い文献がもし見つかればtext名を教えて下さい
教えられないよ
0646132人目の素数さん
垢版 |
2022/07/05(火) 20:16:10.14ID:AOVCHV0H
>>613
>命題自体ハッキリ分からないので

これじゃないの?>>606
>乗法群としての準同型:一般線形群GL(2,c)→c(指標、一次元表現)
>は必ず行列式のべき乗になっている
0647132人目の素数さん
垢版 |
2022/07/05(火) 20:18:52.91ID:AOVCHV0H
>>628
>「乗法群としての準同型:一般線形群GL(2,c)→c(指標、一次元表現)
>は必ず行列式のべき乗になっている」
これが偽なんでしょ
使われている事実は
もう少し別な形なのでは?
0648132人目の素数さん
垢版 |
2022/07/05(火) 21:18:33.46ID:ORT2WHyr
>>647
>これが偽なんでしょ

「乗法群としての準同型:一般線形群GL(2,c)→c(指標、一次元表現)
は必ず行列式のべき乗になっている」の説明が載っている文献はありませんか
という私の質問内容>>606について、以下に貴方の指摘にお答えします
**************

既に610で説明しましたが再度丁寧に説明すると
C→Cの任意の指標λを考えれると、全ての指標σ:GL(2,c)→Cに対して、
σλも指標もまたGL(2,c)→Cの指標になるのは当たり前の話なので
λとして絶対値を取れば、>>608の例は当たり前の話な訳です

そして今「指標」という言葉で私が表現していますが、おそらく
>>628の状況下ではこの指標に何らかの制限が加わって、
C→Cの指標として絶対値が取れない風になってるのだと思います。
例えば指標に「多項式で表される」という制限をつけば指標として絶対値が
取れないのはすぐ分かります。

つまり些末な条件が更に加われば「必ず行列式のべき乗になる」は成立
しますし、そしてその些末な条件がたとえなくても結局「必ず行列式のべき乗を
経由する」となるだけなのでマイナーチェンジにすぎない訳です。
大事なのは「行列式が必ず顔を出す」という所です
>>628の原文を書くと hをGL(2,c)の元として
The coefficient of proportionality e(h) satisfies the property
e(hh') = e(h) e(h'). This implies that e(h) is a power of det(h).
とだけ書いてありました
どんな些末な条件がどう働いてどうなるかの細部は、私は質問を今している側
なのでよく分からないしこのテキストにも説明が載ってるわけでもありません
おそらく上記に書いた多項式に制限された状況である気もします

なので「乗法群としての準同型:一般線形群GL(2,c)→c(指標、一次元表現)
は必ず行列式のべき乗になっている」 という私の質問内容を
分かってらっしゃる人が見れば、それがどの文献に説明されてあって
かつこの準同型にどんなささやかな制限がつけば完全に真になるかも
すぐお分かりになるはずです。
**************
結論
私の質問内容の本質的な部分に
回答者を混乱させるような大きな不備は何もないと思われます。
0649132人目の素数さん
垢版 |
2022/07/05(火) 21:30:22.41ID:ORT2WHyr
>>647
>使われている事実は
>もう少し別な形なのでは?

https://math.stackexchange.com/questions/2714138/one-dimensional-representations-of-gln-k

因みに上の質問サイトでも
any homomorphism σ:GL(n,K)→K∗ factors through the determinant
homomorphism, so has the form σ=μ∘det for
some homomorphism μ:K∗→K∗.
だと誰かが回答されていますので、必ず行列式を経由するのは正しい事のようです
0650132人目の素数さん
垢版 |
2022/07/05(火) 21:55:37.98ID:+lxda97r
頭がおかしい人は推敲が出来ないから長文になる
自分の書いたことは正しいから直す必要がないと考えるのだろう
0651132人目の素数さん
垢版 |
2022/07/05(火) 22:00:33.01ID:UpAd6pr+
それ冪乗じゃないです
0652132人目の素数さん
垢版 |
2022/07/05(火) 22:18:07.35ID:AOVCHV0H
>>648
>λとして絶対値を取れば、>>608の例は当たり前の話な訳です
いくら書いても
「乗法群としての準同型:一般線形群GL(2,c)→c(指標、一次元表現)
は必ず行列式のべき乗になっている」
は偽だよ
君が聞くべきはもう少し別の形なのでは?理解したいと書いている定理?を読み込んでみたらどうかな
0653132人目の素数さん
垢版 |
2022/07/05(火) 22:18:17.96ID:ORT2WHyr
>>650
冒頭と結論だけお読み下さい。そのために小分けしました
プラス649の質問サイトを一瞥すれば十分です
>>651
σ=μ∘det の μをべき乗に取ればいいと思います。

文面からしてIDが異なる同一人物が含まれる気がしますが
完全に無関係な人は特別私の質問に触れて頂かなくても大丈夫です
0654132人目の素数さん
垢版 |
2022/07/05(火) 22:25:31.07ID:ORT2WHyr
>>652
教科書に句読点のミスがあっても教科書の価値は消えません
私の回答内容に触れずに同じ批判を機械的に繰り返されても本来答えようがありませんが
そんなにうるさく仰る貴方だけ向けに質問内容を修正します

any homomorphism σ:GL(n,K)→K∗ factors through the determinant
homomorphism, so has the form σ=μ∘det for
some homomorphism μ:K∗→K∗.
の証明が載っている文献を教えて下さい

これで貴方にもう文句は微塵もないですよね?
これ以降、回答に無関係なレスはもうこれ以上送って頂かなくて大丈夫です
0655132人目の素数さん
垢版 |
2022/07/06(水) 00:03:50.31ID:jrOKtOUb
「aozora 終結式の不変性」で検索せよ
0656132人目の素数さん
垢版 |
2022/07/06(水) 02:42:11.11ID:IKWbvK9o
φがスカラー関数でφ=x^2+y^2ーz, vector(r)=(x, y, z) のときに、
φrというのはなんのことを指しますか?
あまりに基本的?過ぎてネットでは答えが見つかりませんでした
0657132人目の素数さん
垢版 |
2022/07/06(水) 05:51:35.24ID:EWEhPC8J
>>654
>そんなにうるさく仰る貴方だけ向けに質問内容を修正します
修正しなかったから他の人も興味失ってるのかもよ
0658132人目の素数さん
垢版 |
2022/07/06(水) 06:54:04.41ID:6ReqWsl/
結局、聞いてることって群のアーベル化の普遍性の証明ってこと?
0659132人目の素数さん
垢版 |
2022/07/06(水) 07:34:45.29ID:EWEhPC8J
>>658
それおよび[GL,GL]=SLてことかと
0660132人目の素数さん
垢版 |
2022/07/06(水) 08:26:21.38ID:EWEhPC8J
>>656
φrのrが君の書いたvector(r)のことならベクトルのスカラー倍
0661132人目の素数さん
垢版 |
2022/07/06(水) 09:29:09.31ID:OrfsbXyJ
検定統計量とはなんですか?
検定統計量を求められたら、何を書けばいいのでしょうか。有意水準に対応した正規分布の値か、統計量の実測値なのかわからないのですが
0663132人目の素数さん
垢版 |
2022/07/06(水) 17:00:19.41ID:ZPFx8aFT
小林昭七著『続微分積分読本』

陰関数の定理のステートメントが間違っている。
n + k 変数の n 個の関数の連立方程式の k 個の変数が残りの n 個の変数の k 個の関数として書けるなどと書かれています。

これはたちの悪い誤りですよね。
0664132人目の素数さん
垢版 |
2022/07/06(水) 17:12:56.23ID:vVrimzsp
小林昭七さんは大丈夫な人なのでしょうか?
0665132人目の素数さん
垢版 |
2022/07/06(水) 22:40:20.32ID:JNda/fXj
>>663

>>陰関数の定理のステートメントが間違っている。
>>n + k 変数の n 個の関数の連立方程式の k 個の変数が残りの n 個の変数の k >>個の関数として書けるなどと書かれています。

質の悪いのは特にどこ?条件が正確にかけていない?
0666132人目の素数さん
垢版 |
2022/07/06(水) 22:43:09.60ID:z2a93tJR
i^4=1だからi^4は実数って考えるのはダメですか?
0667132人目の素数さん
垢版 |
2022/07/06(水) 23:00:42.36ID:z2a93tJR
i^2=(i^4)^1/2=1
これが間違いである理由は
(i^4)^1/2は複素数の1/2乗だから二価関数になって云々って言われたけど
i^4は複素数なのか?
=1なんだから実数なんじゃないのか?
と思ってしまいました
0668132人目の素数さん
垢版 |
2022/07/07(木) 00:05:33.35ID:SXWnZF2G
実数⊆複素数です
1^(1/2)=1ではなく1^(1/2)=±1です
0669132人目の素数さん
垢版 |
2022/07/07(木) 00:13:05.04ID:IOPmSsg3
んー…
そしたら
1+√1=2,0
になるってことですか?
0670132人目の素数さん
垢版 |
2022/07/07(木) 00:40:17.75ID:HKSEk7cJ
>>667
i^2=-1
i^4=1で、√i^4=√1=1
-1≠1なので、i^2≠√i^4

これで納得出来ないのは何故ですか?
0671132人目の素数さん
垢版 |
2022/07/07(木) 00:53:25.26ID:HKSEk7cJ
|i|=1なので、偏角だけ考えれば十分。極座標を(r, θ)で表すと0≦θ<2πとして
i=(1, π/2)、i^2=(1, π)、i^4=(1, 0)は普通に分かる。

√を二価関数とすると
√i^4=(1, π)、(1, 0)となる。
この世界では√1=±1となります。

中学では√は一価関数でした。
0672132人目の素数さん
垢版 |
2022/07/07(木) 01:00:36.32ID:HKSEk7cJ
要するに間違いである理由は
√○は2個、
□^2は1個、
で納得出来ると思います。

2²=4、√2⁴=±4ということですね。
0673132人目の素数さん
垢版 |
2022/07/07(木) 01:04:09.86ID:4b6vpqXS
>>669
√を複素数上の多価関数として見るならそうだけど、
慣例的には正の実数の√とか有理数乗は正の実数のものを考えるのが普通。

慣例に従うなら正の実数aと整数nに対して(a^n)^(1/n)=aが成り立つけどあくまでも「正の実数」aに対してだし、そこら辺は高校の教科書とかでもよく見ると書いてある。
0674132人目の素数さん
垢版 |
2022/07/07(木) 08:00:39.40ID:bAqVwJKw
>>667
>これが間違いである理由は
a,b,cが実数でも(a^b)^c≠a^(bc)だからでは?
0675132人目の素数さん
垢版 |
2022/07/07(木) 08:02:02.71ID:GS/FqCkE
A...x^(a+3) → x^a * x^3
B... x^(3a) → {x^(a)} ^3

↑ こういう操作が成立するのは
底 x が実数の場合のみ。

xが複素数でこんな操作はない。
i^2 = (i^4)^1/2 ← No. Never. You die
0676132人目の素数さん
垢版 |
2022/07/07(木) 08:05:03.76ID:GS/FqCkE
>>675
たぶん、高校の複素平面あたりの教科書で
ちょろっと説明されていたはず。

1変数1元 と 1変数2元
これらは取り扱いが違うからなぁ?
0677132人目の素数さん
垢版 |
2022/07/07(木) 09:30:42.65ID:b2gYezjC
>>665

このような基本的な定理のステートメントが誤っているというのはたちが悪いとしかいいようがありません。
しかも証明は n = 2, k = 2 の場合について書かれているため、ステートメントの間違いに気づかない可能性
があります。 n ≠ k の場合に証明していれば誤りに気づくはずですが。
0678132人目の素数さん
垢版 |
2022/07/07(木) 09:37:22.40ID:b2gYezjC
小林昭七著『続微分積分読本』

陰関数定理のステートメントにはさらに問題があります。
定理によって存在が保証される陰関数の条件の一つとして、

(ii) |u - u_0| < a かつ |v - v_0| < a のとき、 |φ(u, v) - x_0| < b, |ψ(u, v) - y_0| < b
が成り立つ。

という条件がありますが、全く不要です。
0679132人目の素数さん
垢版 |
2022/07/07(木) 09:47:08.21ID:bZ9jpz9r
>>677
ステートメントwww
雰囲気を説明してるだけの文章を定理のステートメントとして受け取るとか本を読む資格ねぇよ

>>証明は
定理と証明は、だろ
実際の定理は次のページに書いてあって、その後証明してる
定理がn=k=2で書かれてることを隠して証明だけn=k=2でやってるように印象操作か?
0680132人目の素数さん
垢版 |
2022/07/07(木) 09:47:14.92ID:b2gYezjC
例えば、杉浦光夫著『解析入門2』のように、

(u, v, x, y) ∈ (u_0 - a, u_0 + a) × (v_0 - a, v_0 + a) × (x_0 - b, x_0 + b) × (y_0 - b, y_0 + b)

に対して、次の同値が成立つ。

f(u, v, x, y) = 0 ⇔ (x, y) = (φ(u, v), ψ(u, v))

という条件でしたら、 b が登場する理由がわかります。陰関数の一意性が成立つような領域が
存在するということを言っているからです。

小林昭七さんのステートメントでは(ii)は全く不要です。
0681132人目の素数さん
垢版 |
2022/07/07(木) 09:48:06.93ID:bZ9jpz9r
>>678
ステートメントもまともに読めてねぇよ
その条件除いたステートメントキチンと書いて証明してから書き込めよ
0682132人目の素数さん
垢版 |
2022/07/07(木) 09:50:01.20ID:b2gYezjC
>>679

ステートメントは一般の n, k で述べていて、証明は n = k = 2 の場合にしています。
0683132人目の素数さん
垢版 |
2022/07/07(木) 09:52:12.55ID:b2gYezjC
>>681

(ii)は φ, ψ の取りうる範囲を書いていますが、そんなものは余計な情報です。
0685132人目の素数さん
垢版 |
2022/07/07(木) 09:55:06.92ID:b2gYezjC
小林さんのステートメントでは、u_0, v_0 に十分近いところで、 φ, ψ が一意的に存在するという
ことも書いてあるので、 φ, ψ の値の範囲についての記述は全く不要で、余計な記述です。
0688132人目の素数さん
垢版 |
2022/07/07(木) 09:57:23.27ID:b2gYezjC
>>684

第8版 pp.31-32を参照してください。
0689132人目の素数さん
垢版 |
2022/07/07(木) 10:03:19.53ID:b2gYezjC
あ、ステートメントというか陰関数定理の前に、

p.31


陰関数定理は n + k 変数の n 個の関数の連立方程式

f_1(x_1, …, x_n, x_{n+1}, …, x_{n+k}) = 0, …, f_n(x_1, …, x_n, x_{n+1}, …, x_{n+k}) = 0

の場合に拡張される。適当な仮定の下で、その中の k 個の変数、例えば x_{n+1}, …, x_{n+k} が残りの
変数 x_1, …, x_n の k 個の関数として書けるのである。


と書いてあります。

これは最悪の間違いですよね。
0690132人目の素数さん
垢版 |
2022/07/07(木) 10:05:32.15ID:b2gYezjC
ステートメントというか定理の要約ですね。

いずれにしても最悪の間違いです。
0691132人目の素数さん
垢版 |
2022/07/07(木) 10:19:58.55ID:b2gYezjC
シンプルなバージョンの陰関数定理でも同じことですので、それを引用します。
以下の定理1のステートメントの(ii)は全く不要です。

p.22 定理1

点 (x_0, y_0) を含む領域で連続関数 f(x, y) の偏微分 f_x(x, y), f_y(x, y) が存在して、
f_y(x, y) は連続とする。点 (x_0, y_0) で f(x_0, y_0) = 0 そして f_y(x_0, y_0) ≠ 0
ならば、十分小さい実数 a, b > 0 に対し、区間 |x - x_0| < a で定義された関数 y = h(x) で

(i) y_0 = h(x_0)

(ii) |h(x) - y_0| < b, |x - x_0| < a,

(iii) f(x, h(x)) ≡ 0, |x - x_0| < a

となるものがただ一つ存在して連続かつ微分可能で、

f_y(x, h(x)) ≠ 0, |x - x_0| < a

および

h'(x) = -f_x(x, h(x))/f_y(x, h(x)), |x - x_0| < a

が成り立つ。
0692132人目の素数さん
垢版 |
2022/07/07(木) 10:24:06.70ID:b2gYezjC
「十分小さい実数 a, b > 0 に対し、区間 |x - x_0| < a で定義された関数」とかいう話は
定理の証明中で必要な話というだけであり、
h(x) のとる値の範囲が

(ii) |h(x) - y_0| < b, |x - x_0| < a,

となるなどということは全く不要で情報量 0 です。

十分小さい実数 b > 0 などと書いても無意味です。
何が成り立つために十分小さいのかということが述べられていないからです。
0693132人目の素数さん
垢版 |
2022/07/07(木) 10:28:21.60ID:b2gYezjC
h(x) は、ただ一つ存在して連続かつ微分可能なので、 h(x) の値の範囲の話などする必要はありません。
0694132人目の素数さん
垢版 |
2022/07/07(木) 10:33:39.21ID:b2gYezjC
小林昭七さんは推敲など全くしないんでしょうね。
0695132人目の素数さん
垢版 |
2022/07/07(木) 12:20:44.48ID:b2gYezjC
p.32の n = k = 2 の場合の陰関数定理の証明にも問題があることを見つけました。

呆れてしまいます。
0696132人目の素数さん
垢版 |
2022/07/07(木) 12:52:04.85ID:bZ9jpz9r
自分が文章の意味を理解出来てないのを著者の間違いに転嫁しまくってるだけだろ。最低だな、質問にも成ってないし、とっとと消えろ
0697132人目の素数さん
垢版 |
2022/07/07(木) 13:33:07.43ID:b2gYezjC
>>696

それでは

>>691

の(ii)はなぜ必要なのでしょうか?

説明をしてください。
0698132人目の素数さん
垢版 |
2022/07/07(木) 13:45:12.33ID:4b6vpqXS
横からすまん。
本を持ってないから実際にはどうなのか知らないけど、>>691 だと(ii)の部分は不要とかでなく間違いだろ。
原文の意味を保ったまま写せていないか、正しいかどうかの判定ができていない。
0699132人目の素数さん
垢版 |
2022/07/07(木) 14:21:29.23ID:b2gYezjC
>>698

もちろん、原文そのままです。
0700132人目の素数さん
垢版 |
2022/07/07(木) 14:24:06.02ID:b2gYezjC
>>691

さらに、問題を見つけました:

f_x(x, y) も連続でないとだめですよね。
0702132人目の素数さん
垢版 |
2022/07/07(木) 14:37:16.40ID:b2gYezjC
小林昭七さんの弟の小林久志さんはリーマン予想の研究をしているそうですね。
0704132人目の素数さん
垢版 |
2022/07/07(木) 14:42:28.18ID:b2gYezjC
h(x) が微分可能であることを示すのに、 f_x(x, y) が連続であるという仮定が必要です。
0707132人目の素数さん
垢版 |
2022/07/07(木) 15:06:12.82ID:b2gYezjC
(ii) |h(x) - y_0| < b, |x - x_0| < a,

は h(x) が満たすべき性質を述べたものであって、定理が成り立つための条件ではありません。
0708132人目の素数さん
垢版 |
2022/07/07(木) 17:28:22.26ID:iueqHr2z
>>701
「十分小さい実数a,b>0に対し、」
っていうのは
「ある正数があってそれよりも小さな任意の正数a,bに対し、」
って読むのが普通では?
0709132人目の素数さん
垢版 |
2022/07/07(木) 17:30:31.71ID:2+f1jbu7
>>707

> (ii) |h(x) - y_0| < b, |x - x_0| < a,
>
> は h(x) が満たすべき性質を述べたものであって、定理が成り立つための条件ではありません。

そしてその性質を書かなければ一意性が担保できないと書いているのが
>>705,706
0711132人目の素数さん
垢版 |
2022/07/07(木) 17:44:14.44ID:b2gYezjC
以下で、十分なはずです。(ii)は不要です。

点 (x_0, y_0) を含む領域で連続関数 f(x, y) の偏微分 f_x(x, y), f_y(x, y) が存在して、
f_y(x, y) は連続とする。点 (x_0, y_0) で f(x_0, y_0) = 0 そして f_y(x_0, y_0) ≠ 0
ならば、十分小さい実数 a, b > 0 に対し、区間 |x - x_0| < a で定義された関数 y = h(x) で

(i) y_0 = h(x_0)

(iii) f(x, h(x)) ≡ 0, |x - x_0| < a

となるものがただ一つ存在して連続かつ微分可能で、

f_y(x, h(x)) ≠ 0, |x - x_0| < a

および

h'(x) = -f_x(x, h(x))/f_y(x, h(x)), |x - x_0| < a

が成り立つ。
0712132人目の素数さん
垢版 |
2022/07/07(木) 17:44:44.71ID:iueqHr2z
>>710
もちろんここを正しく解釈するなら
「ある正数a,bがあって」
だとは思うし、そう解釈した上で>>705の主張が正しいと思うけど、一般的な数学の記述としては>>708のように読まない?
0713132人目の素数さん
垢版 |
2022/07/07(木) 17:45:00.47ID:b2gYezjC
訂正します。

以下で、十分なはずです。(ii)は不要です。

点 (x_0, y_0) を含む領域で連続関数 f(x, y) の偏微分 f_x(x, y), f_y(x, y) が存在して、
f_y(x, y) は連続とする。点 (x_0, y_0) で f(x_0, y_0) = 0 そして f_y(x_0, y_0) ≠ 0
ならば、十分小さい実数 a > 0 に対し、区間 |x - x_0| < a で定義された関数 y = h(x) で

(i) y_0 = h(x_0)

(iii) f(x, h(x)) ≡ 0, |x - x_0| < a

となるものがただ一つ存在して連続かつ微分可能で、

f_y(x, h(x)) ≠ 0, |x - x_0| < a

および

h'(x) = -f_x(x, h(x))/f_y(x, h(x)), |x - x_0| < a

が成り立つ。
0714132人目の素数さん
垢版 |
2022/07/07(木) 17:46:00.64ID:b2gYezjC
訂正します。

以下で、十分なはずです。(ii)は不要です。

点 (x_0, y_0) を含む領域で連続関数 f(x, y) の偏微分 f_x(x, y), f_y(x, y) が存在して、
f_x(x, y), f_y(x, y) は連続とする。点 (x_0, y_0) で f(x_0, y_0) = 0 そして f_y(x_0, y_0) ≠ 0
ならば、十分小さい実数 a > 0 に対し、区間 |x - x_0| < a で定義された関数 y = h(x) で

(i) y_0 = h(x_0)

(iii) f(x, h(x)) ≡ 0, |x - x_0| < a

となるものがただ一つ存在して連続かつ微分可能で、

f_y(x, h(x)) ≠ 0, |x - x_0| < a

および

h'(x) = -f_x(x, h(x))/f_y(x, h(x)), |x - x_0| < a

が成り立つ。
0715132人目の素数さん
垢版 |
2022/07/07(木) 18:00:30.22ID:IlLilGbF
定理の主要部分は(i)(ii)(iii)を満たす関数hがただ一つ存在する
までで連続であるとか微分可能であるとか導関数がこうなるとかはその定理の系みたいなもんじゃないの?
定理の主要部分を親切に証明しやすい形で提示してそこが証明できればあとは簡単という感じで
0716132人目の素数さん
垢版 |
2022/07/07(木) 20:10:20.90ID:iueqHr2z
>>714
x_0=y_0=0, f(x,y) = sin y とすると、
h(x) ≡ 0 も
h(0) = 0, x ≠ 0 に対し h(x) = π も
(i), (iii)を満たし、一意でない。
0717132人目の素数さん
垢版 |
2022/07/07(木) 20:32:17.92ID:b2gYezjC
>>716

ありがとうございます。

よく見たら、

「となるものがただ一つ存在して」

の部分で一つの主張になっていたんですね。

「となる連続かつ微分可能なものがただ一つ存在して」

かと勘違いしていました。
0718132人目の素数さん
垢版 |
2022/07/07(木) 20:35:53.28ID:b2gYezjC
>>715

ありがとうございました。
0719132人目の素数さん
垢版 |
2022/07/07(木) 20:50:49.51ID:bAqVwJKw
>>717
どっちでも同じじゃないの?
0722132人目の素数さん
垢版 |
2022/07/08(金) 06:30:21.67ID:JQJC0KGM
>>704
こっちの主張は撤回しないの?
0723132人目の素数さん
垢版 |
2022/07/08(金) 07:40:18.60ID:nedRpoQC
C*を複素数体の乗法群として、
Hom(C*,C*)=Z(整数環)
という記述があるのですが、どうやれば示せるでしょうか?
https://arxiv.org/abs/1211.0763
この論文の4ページ目です。
0725132人目の素数さん
垢版 |
2022/07/08(金) 09:43:35.32ID:w1/lZ7lo
f(x, y) = f(r*cos(θ), r*sin(θ)) =: g(r, θ)

∂g/∂θ の意味で ∂f/∂θ という記号を使うことがありますが、

(∂/∂θ) g(r, θ) のように関数の引数も明示する場合、 ∂f/∂θ を使って表わすにはどうするのでしょうか?
0726132人目の素数さん
垢版 |
2022/07/08(金) 09:44:42.09ID:w1/lZ7lo
(∂/∂θ) g(r, θ) = (∂/∂θ) f(r, θ)

と書くのでしょうか?

非常に違和感があります。
0727132人目の素数さん
垢版 |
2022/07/08(金) 10:38:34.23ID:GzmORaVV
>>724
そのページの初めにHomは複素リー群の準同型って書いてある。

被覆C → C*をとって、2πi の行き先が 2πiZに入っていないといけないから、って感じかな。
0728132人目の素数さん
垢版 |
2022/07/08(金) 10:40:32.36ID:YspWWlMa
なんだ、位相込みか
0729132人目の素数さん
垢版 |
2022/07/08(金) 11:08:20.09ID:I0CaNP7S
論文見てないけど代数群のカテゴリでの話じゃないか
Langlanda dualとか言ってるし

ならば多項式写像ゆえ z -> z^n (n \in Z)のみ
0730132人目の素数さん
垢版 |
2022/07/08(金) 19:11:15.17ID:w1/lZ7lo
小林昭七著『続微分積分読本』

2次対称行列についてですが、その固有値が実数であること、互いに異なる固有値 λ1, λ2 を持つときに、
λ1 に対する固有ベクトルと λ2 に対する固有ベクトルは直交することが書いてあります。

そして、

「λ1 ≠ λ2 の場合には、逆に v1 も v2 も実数倍を除けば一意に定まる。」

と書いてあります。その理由として、

「なぜなら、 v1 と v2 を1つずつ固定すれば、 λ1 に対する固有ベクトルはすべて v2 に直交するから
v1 の実数倍である。全く同様に、 λ2 に対する固有ベクトルはすべて v1 に直交するから v2 の実数倍である。」

と書いてあります。

これは無理がありますよね?

v2 に直交するベクトルは v1 の実数倍とは限らないですよね。明らかに。

小林昭七さんは幾何学者だったそうですが、信じられません。
0731132人目の素数さん
垢版 |
2022/07/08(金) 19:21:37.03ID:QJ7qtGfP
>>730
2次対称行列だから2次元だけ考えたらいいだろ
2次元なら直交したら実数倍でいいんじゃないの?
0732132人目の素数さん
垢版 |
2022/07/08(金) 19:39:31.54ID:w1/lZ7lo
>>731

あ、3次元でイメージしていました。
ありがとうございました。
0735132人目の素数さん
垢版 |
2022/07/09(土) 01:26:39.99ID:V9DvWBnU
自分のミスへの指摘は「チッ、うっせーなー」とか思ってそう
0736132人目の素数さん
垢版 |
2022/07/10(日) 23:55:02.37ID:O6UFZuIG
ベルヌーイ型の微分方程式が解けません

y'+ylogx=x^3*y^7
という問題なのですが
u=y^(-6)とおいて変形し一般解を求めると
u=c*exp6(xlogx-x) (cは実数)
となりました
このcをxの関数にして、1階線形微分方程式のほうに代入すると
c'=(-6x^3)/exp6(xlogx-x)
となりここで詰まってしまいました
この先の解き方を聞きたいです
0738132人目の素数さん
垢版 |
2022/07/11(月) 02:11:56.44ID:kI2HTsSz
>u=c*exp6(xlogx-x) (cは実数)

そのままu=y^(-6)に戻して終わりじゃダメなん?
0739132人目の素数さん
垢版 |
2022/07/11(月) 06:40:18.26ID:67twRYnr
>>738
それは
y’+ylogx=0の答え
0740132人目の素数さん
垢版 |
2022/07/11(月) 08:40:01.64ID:GK3V8uy1
>>737
定数です書き間違えました
0741132人目の素数さん
垢版 |
2022/07/12(火) 15:06:00.65ID:TSOFUNop
0より大きい正の実数全体は交換則を満たさないんですかね?
教科書にアーベル群ではなく単に群と書いてあるので気になるのですが
0743132人目の素数さん
垢版 |
2022/07/12(火) 15:45:48.12ID:TSOFUNop
アーベル群⇒群ですが群⇒アーベル群とはならないので、教科書に「群」としか書かれていない場合「アーベル群」とは言えないのかなと思いました

あと演算は通常の乗法です
0745132人目の素数さん
垢版 |
2022/07/12(火) 18:41:50.28ID:+HRji7aF
>>743
その疑問の持ち方がおかしい、普通にアーベル群です

「1は整数」「整数は実数だが逆は言えない」からと言って「1は実数ではないのか?」なんてアホな疑問は持たないでそ
0746132人目の素数さん
垢版 |
2022/07/12(火) 19:14:29.45ID:PS4VFggu
白馬非馬論の逆か
0747132人目の素数さん
垢版 |
2022/07/12(火) 20:17:26.07ID:3u3ORUUK
>>745
なんとなくわかったようなわからないような……
教科書はアーベル群の場合は「アーベル群」、交換則を満たさない群は「群」と表記されていたので
「群」としか書かれていない場合は交換則を満たさないのか?と思っていましたが
単にアーベル群かどうかは関係ないから「群」と書いただけで、実際はアーベル群ってことですかね
0748132人目の素数さん
垢版 |
2022/07/12(火) 20:32:09.26ID:Jk7+/xPN
複素解析の質問です。
aを1より大きい実数とした時に、
∫[0,π]dθ/(a+cosθ) = π/√(a^2-1)
を留数計算で示しました。
領域C - {1,-1}の上で、左辺も右辺もaに関して正則であるから、一致の定理によりa∈C - {1,-1}に対してこの式が成り立つ、という議論をしたいのですが、左辺が正則であることはどのように示すのでしょうか?
また、これを示そうとすると、一般に次のような定理が成り立つことを期待してしまうのですが、これは正しいですか?
Dを領域とする。2変数関数f(x,y)は、各y∈Dに対し、xに関し正則であるとする。このとき、D上の積分∫[D]f(x,y)dyはxについて再び正則となる。
0749132人目の素数さん
垢版 |
2022/07/12(火) 20:33:45.23ID:2B88ElX1
>>748
訂正
C - {-1,1}上で左辺は正則にならなさそうなので、C - [-1,1]に訂正させてください。
[-1,1]は-1以上1以下の実数の集合とします。
0751132人目の素数さん
垢版 |
2022/07/12(火) 21:14:13.88ID:3u3ORUUK
>>750
何か交換則をみたさない反例があるんじゃないかと疑ってしまいましたが、証明読んだらアーベル群なのは当たり前でしたね……
当たり前過ぎて教科書も書かなかったのかもしれないです
ソースまで載せて頂きありがとうございました!助かりました
0752132人目の素数さん
垢版 |
2022/07/12(火) 22:25:33.78ID:qh9H5dC+
>>748
左辺を偏微分してコーシーリーマンの関係式を確かめればいい
微分と積分の順序交換ができないようなヤバい関数でなければ下に書いてあるようなこと(積分する範囲はDでない気がするが)も成り立つ

ちなみに右辺もC-{1,-1}だと正則に定義することはできないので注意
0753132人目の素数さん
垢版 |
2022/07/13(水) 18:33:33.65ID:PNq567Hc
杉浦光夫著『解析入門I』

「lim_{x → a} f(x) = f(a) が存在するとき」などと書いていますが、日本語として意味が通じません。
「lim_{x → a} f(x) が存在して、その値が f(a) に等しいとき」などとすべきです。
0754132人目の素数さん
垢版 |
2022/07/13(水) 18:36:42.08ID:om320upW
通じてるやんけ
0758132人目の素数さん
垢版 |
2022/07/13(水) 19:30:48.18ID:PNq567Hc
lim_{x → a} f(x) が存在して、その値が f(a) に等しいときに、

「lim_{x → a} f(x) = f(a)」と書きます。

「lim_{x → a} f(x) = f(a) が存在して」などというのはおかしな日本語です。

「lim_{x → a} f(x) = f(a) が成り立って」などと書くのなら分かります。
0759132人目の素数さん
垢版 |
2022/07/13(水) 20:28:08.55ID:lFKZStIn
全順序集合 (A, ≦) と (B, ≦) が順序同型であることの定義として
「全ての A の元 x, y に対して
x ≦ y ⇔ f(x) ≦ f(y)
となるような全単射 f : A→B が存在すること」
とあったのですが、AもBも全順序集合でかつその間に全単射が存在し、しかし順序同型とはならない事はあり得るのでしょうか
0760132人目の素数さん
垢版 |
2022/07/13(水) 20:30:09.69ID:ur/BdnsW
>>758
「意味が通じません」ということは全くないのは納得したか?

正しく意味が通じた上での話なんだよな。つまりお前は嘘をついているということ。
0762132人目の素数さん
垢版 |
2022/07/13(水) 20:39:58.49ID:eYYCPnn+
>>759
自然数と整数
0764132人目の素数さん
垢版 |
2022/07/13(水) 21:02:17.85ID:eYYCPnn+
自然数と有理数も順序同型ではない
任意の有理数A,Bに対しA<BならA<C<Bとなる有理数Cが存在するが、こんなことは自然数や整数では起こらない
0768132人目の素数さん
垢版 |
2022/07/13(水) 23:21:18.72ID:oH75wZW2
1つ目、どうでもいい参院の時期っていう不自然さ。
第二次政権の安倍総理の全盛期でもなく、
辞任をして警備がゆるくなる時期でもなく、衆院選の前でもなく。

2つ目、政治上の安倍総理の言動についての
怒りや批判が無いことの不自然さ。
まるで安倍さんが政治家であることを
知らないフリをしているかのような供述。

疑惑は深まったな ( '‘ω‘)
0769132人目の素数さん
垢版 |
2022/07/14(木) 00:12:43.21ID:4bleQ6w1
3人以上の候補者がいる選挙の当確を検定したい場合、カイ2乗検定であってますか?
0770132人目の素数さん
垢版 |
2022/07/14(木) 00:15:28.93ID:kgE2wpsL
>>766
順序はどうでもいいので、存在するでしょう。
たとえば、0 は 0 に、偶数 2n は n に、奇数 2n+1 は -n  に  とすれば自然数から整数への全単射   
0771132人目の素数さん
垢版 |
2022/07/14(木) 06:04:51.77ID:/emQQzhQ
>>765-767
条件を満たさない全単射の例を聞いかれたから答えてあげたのに「条件を満たす全単射はないが」とか言うんですね
0772132人目の素数さん
垢版 |
2022/07/14(木) 08:20:07.91ID:vNnX9kv8
杉浦光夫著『解析入門I』

「例えば a ∈ A のとき B = {x ∈ A | x ≠ a} ならば lim_{x → a, x ∈ B} f(x) を lim_{x → a, x ≠ a} f(x) と記す。」

と書かれています。

a が A の孤立点のときには、 a ∈ B の閉包ではありません。

ですので、この場合、極限は定義できません。

不注意ですね。
0774132人目の素数さん
垢版 |
2022/07/14(木) 10:06:39.43ID:Gf9rMW1J
>>772

「a ∈ B の閉包ではありません。」などと書いていますが、日本語として意味が通じません。
「a は B の閉包には含まれません。」などとすべきです。
0775132人目の素数さん
垢版 |
2022/07/14(木) 10:56:23.61ID:LNcnROBh
「a ∈ B の閉包ではありません。」と書いたら
読み手が自然に「a∈(Bの閉包)ではありません」って読んでもらえると思ってるあるいは判読性を疑いもしないあたり数学出来ない系アスペ感凄い
0776132人目の素数さん
垢版 |
2022/07/14(木) 17:23:48.91ID:udJsmUoK
>>775
ヘクトルの積でa・b×cと書かれてたらa・(b×c)と解釈するようなもの
数学ではこういう言い回しは多用するからギリ許容範囲かな
0777132人目の素数さん
垢版 |
2022/07/14(木) 20:28:04.74ID:vNnX9kv8
小林昭七著『続微分積分読本』

p.59の定理1(広義の極値に関する定理)のステートメントがおかしいですね。
0778132人目の素数さん
垢版 |
2022/07/14(木) 20:33:16.73ID:vNnX9kv8
↓これはおかしいですよね。

(a, b) において f(x, y) が極大値をとるならば、 f_x(a, b) = f_y(a, b) = 0 で

f_{xx}(a, b) ≦ 0
f_{yy}(a, b) ≦ 0
f_{xx}(a, b) * f_{yy}(a, b) - f_{xy}(a, b)^2 ≧ 0

となる。

---------------------------------------------------------------------------------------
↓こう書くべきでした。

(a, b) において f(x, y) が極大値をとるならば、 f_x(a, b) = f_y(a, b) = 0 で

f_{xx}(a, b) ≦ 0
f_{yy}(a, b) ≦ 0
f_{xx}(a, b) * f_{yy}(a, b) - f_{xy}(a, b)^2 = 0

となる。
0779132人目の素数さん
垢版 |
2022/07/14(木) 20:34:48.88ID:vNnX9kv8
あ、間違っていませんでした。
0780132人目の素数さん
垢版 |
2022/07/14(木) 20:38:12.47ID:buNER6W9
そもそも間違えていたとして質問スレに書くのがおかしいの分かってんのかな
0783132人目の素数さん
垢版 |
2022/07/14(木) 21:55:33.09ID:BfOKPEeG
無駄無駄、どうせこいつは自分のことを歴史上の大天才レベル(より上)の人間だと思ってるから
だから本見てちょっとでも理解に詰まったら「ここは間違ってる、著者はおかしい」と攻撃する

いや大天才どころか全知全能の神様だと思い込んでる節すらあるな……まあとにかく馬鹿は死ななきゃ治らんわ
0785132人目の素数さん
垢版 |
2022/07/15(金) 03:23:22.19ID:ETgxF3jY
Juergen Jost著『Postmodern Analysis Third Edition』

exp(x) < exp(y) for x < y

が成り立つ理由として、

(1 + x/n)^n = 1 + x + other positive terms that are increasing in x, and consequently
(1 + y/n)^n - (1 + x/n)^n ≧ y - x for x < y.

などと書いてあります。

Binomial(n, 2) * (x/n)^2 は x の単調増加関数ではありません。

f(x) := (1 + x/2)^2 = 1 + x + x^2/4

とおきます。

f(-4) = 1
f(-2) = 0

-4 < -2 ですが、 f(-4) ≦ f(-2) は成り立ちません。

Juergen Jostさんは大丈夫な人なのでしょうか?
0786132人目の素数さん
垢版 |
2022/07/15(金) 04:10:02.46ID:ETgxF3jY
小谷元子訳では、

>>785

が修正されていますね。
0787132人目の素数さん
垢版 |
2022/07/15(金) 07:48:52.36ID:QsRH1o6V
大丈夫な人というのが最低限の意思伝達能力を持っていることを指すならあなたは大丈夫じゃないです
大丈夫な人というのが1つのミスも犯さない人という意味ならあなたは大丈夫じゃないです(>>779)
0789132人目の素数さん
垢版 |
2022/07/15(金) 10:35:08.95ID:s4PSqeI1
そういうことか笑
誤植情報を拾って来てドヤ顔するという

原書を読んでおかしいと思って翻訳書に目を通したら修正されてたみたいにミスリードしてるが実態はそういうことなのか
0790132人目の素数さん
垢版 |
2022/07/15(金) 14:54:27.56ID:fXGKawOF
Z係数ホモロジー群がQと同型になるような空間の例ってあるのでしょうか
かなり大きくて病的な空間になりそうな気がしますが具体的な例があったら教えて下さい
0791132人目の素数さん
垢版 |
2022/07/15(金) 15:44:31.99ID:6kL98BuR
正の有理数でパラメトライズされたループを一点で繋げて正の整数nでb = naとなる全てのa,bについて円盤を周がaループをn周、bループを一周するように貼り付けていけばいいのではなかろか
0792132人目の素数さん
垢版 |
2022/07/15(金) 16:39:11.24ID:jemf3Cey
>>790
Q=colim(Z→Z→Z→……)だから
X=mappingcylinder(C→C→C→……)で良くない?
→は2倍3倍4倍……Cは円周
0793132人目の素数さん
垢版 |
2022/07/15(金) 17:41:24.51ID:fXGKawOF
おお確かにどちらもなってますね
極限使って形式的に構成できるってのは盲点でした
お二人ともありがとうございます
0794132人目の素数さん
垢版 |
2022/07/16(土) 02:09:08.00ID:7+cl3AN1
人間の証明をしてください。
人間の定義があいまいなのに、その証明を麦わら帽子でやろうなんて
無謀ですよね。僕にはあれが空飛ぶ円盤にしか見えないです。
だから、数学的に厳密な証明を希望いたします。あくまで数学的に。
0795132人目の素数さん
垢版 |
2022/07/16(土) 05:06:48.31ID:zdLLCXp/
メルカリのキャンペーンで、出品すると特典が抽選であたるというのをやっています。

1等が50%ポイント還元クーポンです。

「お買い物が実質半額」などと宣伝しています。

クーポンは1回しか使えないと思います。

1000円のものを購入したとします。500ポイント還元されます。
ですので、1500円のものを1000円で購入できることになります。

結局、(1 - 1000/1500) * 100 = 33 %オフでしか買い物できません。

このような嘘を書くのはありですか?
0796132人目の素数さん
垢版 |
2022/07/16(土) 07:25:39.59ID:9uA8AfdZ
>>795
500ポイントでたとえばふだん必ず使う消耗品買えば1000円のお買い物が500円で買えたことになる
今後永久にお金を使うことないなら1500円分の買い物を1000円でしたことになる
0797132人目の素数さん
垢版 |
2022/07/16(土) 07:28:02.59ID:zdLLCXp/
>>796

「今後永久にお金を使うことないなら」

↑これは何ですか?
0798132人目の素数さん
垢版 |
2022/07/16(土) 08:36:45.77ID:7+cl3AN1
人間の証明をしてください。
そもそも人間の定義があいまいなのに、その証明を麦わら帽子でやろうなんて
無謀ですよね。僕には自転しながら谷に落ちていく麦わら帽子が空飛
ぶ円盤にしか見えないです。哲学板で聞いたけど余所でやれの書き込み禁止
くらいました。

僕の証明:僕は僕を思う僕である。さらにその僕は僕を思うことができるというか、
想像できる。同様にしていけば永遠の連鎖となるので疲れる僕がいる。
それを永遠のゼロと呼ぶこととする。
僕が存在しないのに疲れることはない。よって僕が存在する。しかし、それでは
中身がなく空虚すなわちゼロであるから、僕は形式的にのみ存在する。

多分、森村誠一による人間の定義と証明:人間は男と女の総体である。
母さん、あの帽子どこ行ったんでしょうね、
あの帽子ですよと問う。母は何も応えない。母さんはレイプで生まれたこの子
が邪魔になった。
そのため、母を訪ねて三千里の息子の気持ちを冷酷に無視して殺害。
息子は致命傷を負い、どうしようもない悲しみの中、母をかばい、去るように言う。
母に疑いがかからないように自らの体を引きずってビルを出るがそこで絶命。
その努力も空しく母親に容疑がかかり追い詰められた、この女は自らの罪を告白し自殺。
親子の情と保身を天秤にかけると女はずるいから後者を選ぶ。
ある程度の教養をもつと、後悔と自責の念にさいなまれるが、このケースだと被害者の
情を援用し、やはり自己正当化で後者を選ぶ。追い詰められても同情を受けようとして
死によって逃走する。一方、男は情のために死ぬ。特攻隊精神。
0799132人目の素数さん
垢版 |
2022/07/16(土) 10:51:14.48ID:vwqs+SEq
nを正の整数とし、Sはn個の元からなる集合とする。このとき、Sに定まる演算は何個あるか。

という問題なのですが、答えがn^(n^2)個になる理由がわかりません…
どなたか解説して頂けないでしょうか
宜しくお願いします
0800132人目の素数さん
垢版 |
2022/07/16(土) 10:59:00.48ID:RNxCBTOU
演算=二項演算=単なる写像S×S→S?ならその個数は#Map(S×S,S)=(#S)^#(S×S)=n^(n×n)=n^(n^2)
0802132人目の素数さん
垢版 |
2022/07/16(土) 12:24:42.75ID:vwqs+SEq
二項演算です
解答にはn^(n^2)と書いてあるんですよね…
自分もn^3なのでは?と思ったのですが
0803132人目の素数さん
垢版 |
2022/07/16(土) 12:27:41.31ID:vwqs+SEq
例えばn=2とすると、S×Sは4通り、Sは2通りになるので、演算は4*2=8個になる気がします
しかしn^(n^2)が答えだとするとn=2のときには16個も存在することになります
どうすれば16個もの演算を考えられるのか…?と疑問に思った次第です
0804132人目の素数さん
垢版 |
2022/07/16(土) 12:37:12.85ID:mj9hLDkO
演算を*で表すことにしてS = {0,1}とすると、
0*0 = 0 or 1, 0*1 = 0 or 1, 1*0 = 0 or 1, 1*1 = 0 or 1
で、それぞれ2通りだから、2^4 = 16通りある
0805132人目の素数さん
垢版 |
2022/07/16(土) 12:42:29.39ID:hSoFIeCr
>>803
4つのもの一つ一つに対し
二通りの割り当て方があるとしたときに
全部で16通りの割り当て方があるというのは
不思議でしょうか
0806132人目の素数さん
垢版 |
2022/07/16(土) 12:46:03.35ID:t68xhH3Q
S={a,b}のときS×S→Sは

1.(a,a)→a,(a,b)→a,(b,a)→a,(b,b)→a
2.(a,a)→a,(a,b)→a,(b,a)→a,(b,b)→b
3.(a,a)→a,(a,b)→a,(b,a)→b,(b,b)→a
4.(a,a)→a,(a,b)→b,(b,a)→a,(b,b)→a
5.(a,a)→b,(a,b)→a,(b,a)→a,(b,b)→a
6.(a,a)→a,(a,b)→a,(b,a)→b,(b,b)→b
7.(a,a)→a,(a,b)→b,(b,a)→a,(b,b)→b
8.(a,a)→b,(a,b)→a,(b,a)→a,(b,b)→b
9.(a,a)→a,(a,b)→b,(b,a)→b,(b,b)→a
10.(a,a)→b,(a,b)→a,(b,a)→b,(b,b)→a
11.(a,a)→b,(a,b)→b,(b,a)→a,(b,b)→a
12.(a,a)→a,(a,b)→b,(b,a)→b,(b,b)→b
13.(a,a)→b,(a,b)→a,(b,a)→b,(b,b)→b
14.(a,a)→b,(a,b)→b,(b,a)→a,(b,b)→b
15.(a,a)→b,(a,b)→b(b,a)→b,(b,b)→a
16.(a,a)→b,(a,b)→b,(b,a)→b,(b,b)→b
0807132人目の素数さん
垢版 |
2022/07/16(土) 12:48:17.19ID:Z2vZ2kBR
便宜上S={1, 2, ... , n}とかおく
(S×S)の元の数はn^2
(S×S)の写像1つ1つを考えると(1, 1)からSへの写像はn通り、(1, 2)からでもn通り、……(n, n)からでもn通り
n×n×…×nって(n^2)回掛け合わせるので
n^(n^2)通りの2項演算が考えられることになる
0808132人目の素数さん
垢版 |
2022/07/16(土) 12:52:01.50ID:t68xhH3Q
>>800
ここで恐らく#Map(X,Y)=(#Y)^(#X)ではなく#Map(X,Y)=(#X)^(#Y)だと勘違いしてるんだろう、割とよくあるミス

X={a},Y={b,c}のときXからYへの写像の個数は(#X)^(#Y)=1^2=1でしょうか、それとも(#Y)^(#X)=2^1=2でしょうか……?
0809132人目の素数さん
垢版 |
2022/07/16(土) 12:52:02.95ID:Z2vZ2kBR
(S×S)とSの各元の組を(1,2,3)みたいに並べたらその組み合わせはn^3になっちゃうな
演算"の"数だから>>805みたいな数え方もとい考え方が正
0810132人目の素数さん
垢版 |
2022/07/16(土) 14:03:02.80ID:vwqs+SEq
>>809
まさにその組として考えてしまっています…
>>805に関しても4*2=8として考えてしまうのですがなぜ16通りになるのでしょうか…?

S={a,b}とした時のS×S→Sの写像は
(a,a)→a or b
(a,b)→a or b
(b,a)→a or b
(b,b)→a or b
で8通りなんじゃ…?
0811132人目の素数さん
垢版 |
2022/07/16(土) 14:11:04.72ID:vwqs+SEq
なんとなくわかったかもしれません!
S×S内の4点がS内の2点に対応する時の全体の組を考えなければならないってことですよね?
(a,a),(a,b),(b,a),(b,b)の全てがaに対応することもあれば、その中の1つだけがbに対応したり2つがbに対応することもある、と
そうなると確かに16通り存在しますね…!
0812132人目の素数さん
垢版 |
2022/07/16(土) 15:24:11.16ID:ET2A5Xfv
>>810
> S={a,b}とした時のS×S→Sの写像は
> (a,a)→a or b
> (a,b)→a or b
> (b,a)→a or b
> (b,b)→a or b
> で8通りなんじゃ…?
なんでそれわかってて8と考えるのかがむしろわからん
2×2×2×2=16
なのに
まさか
2+2+2+2=8
となると思った?なんでそうなると思うのかがわからんが…
0813132人目の素数さん
垢版 |
2022/07/16(土) 16:43:34.91ID:vwqs+SEq
>>812
2+2+2+2=8だと思ってました…
(a,a)→aで演算1個だと思ってたんです
(a,a),(a,b),(b,a),(b,b)→aとか(a,a),(a,b)→a∧(b,a),(b,b)→bで演算1個ですもんね
0814132人目の素数さん
垢版 |
2022/07/16(土) 17:10:59.83ID:hSoFIeCr
演算の定義が分かっていなかったという風には
認識していない?
0815132人目の素数さん
垢版 |
2022/07/16(土) 17:59:20.92ID:vwqs+SEq
>>814
演算の定義以前に写像を理解できてなかったです…
0816132人目の素数さん
垢版 |
2022/07/16(土) 18:59:36.62ID:hSoFIeCr
では
写像が理解できているかどうかをチェックしてみよう。
空集合から空集合への写像は
集合としては
何でしょう。
0817kubi-nagayao
垢版 |
2022/07/16(土) 19:36:35.35ID:PWIxHuW3
>>785
(1 + x/n)^n = 1 + x + other positive terms that are increasing in x,
     =1+x+m(x) here is m(x) is other positive terms that are increasing in x

and
(1 + y/n)^n - (1 + x/n)^n =y - x+ m(y)-m(x)
because m(z) is strictly increasing in z,so m(y)-m(x)>0 .
and consequently

(1 + y/n)^n - (1 + x/n)^n > y - x
0818132人目の素数さん
垢版 |
2022/07/17(日) 00:02:51.76ID:mWwK63KX
g({y_i},t) = Π_i δ(y_i - x_i(t))
のとき、gを時間微分すると、
∂g/∂t= -Σ_i ∂/∂y_i ∂x_i/∂t g
になるようなのですが、計算が出来ません。
わかる方教えていただきたいです。
0819132人目の素数さん
垢版 |
2022/07/17(日) 01:19:13.26ID:8V0ggkZa
Juergen Jost は正しい。>>785
0820132人目の素数さん
垢版 |
2022/07/17(日) 02:41:22.15ID:0eh/Rr0N
nが偶数のとき
xⁿ/n! + xⁿ⁺¹/(n+1)!
は単調増大である
(I) n=0のとき
xⁿ/n! + xⁿ⁺¹/(n+1)! = 1 + x
により正しい
n=2のとき
xⁿ/n! + xⁿ⁺¹/(n+1)! = x²/2+x³/6
により正しい
(Ii) n=k≧2で正しいとしてn=k+2とする
f(x) = xᵏ⁺²(k+2)! + xᵏ⁺³/(k+3)!
とおいて帰納法の仮定からf''(x)は単調増加かつf''(0)=0により
f''(x)>0 (∀x>0)、f''(x)<0 (∀x>0)、
さらにf'(0)=0であるからf'(x)>0 ( unless x=0)
∴f(x) = xᵏ⁺²(k+2)! + xᵏ⁺³/(k+3)! は単調増加である
0822132人目の素数さん
垢版 |
2022/07/17(日) 18:27:57.62ID:O+96FzAs
>>818
>∂g/∂t= -Σ_i ∂/∂y_i ∂x_i/∂t g
∂x_i/∂tって?
0823132人目の素数さん
垢版 |
2022/07/17(日) 18:32:35.06ID:GEw8M/v/
>>822
ブラウン運動からフォッカープランク方程式を導出する過程で熱力学的量x_iに対するランジュバン方程式を立てるところからスタートしてます。
以下のpdfの第二章p30の(2.4)式に対応してるのですが、この導出がわかりません…
なんでマイナスつくんだ…
http://www7b.biglobe.ne.jp/~ftanaka/member/ftanaka/webct/brown.pdf
0825132人目の素数さん
垢版 |
2022/07/17(日) 21:45:17.50ID:O+96FzAs
>>823
うーん聞きたいのは君の式ではx_i(t)と1変数関数賭して書かれているのに∂x_i/∂tと偏微分になってることだったけどここはdx_i/dt?
答え聞く前に次の質問も先にして良いかな?
∂/∂y_i ∂x_i/∂t gの∂/∂y_iってどこに掛かるの?
0827132人目の素数さん
垢版 |
2022/07/17(日) 22:22:54.93ID:O+96FzAs
>>826
キモ
0828132人目の素数さん
垢版 |
2022/07/17(日) 22:30:35.41ID:mWwK63KX
>>825
そういうことでしたか、偏微分はミスでした。
∂/∂yはそのまま後ろにかかる演算です(多分)
なぜそうなるのか分からないので質問しました
0829132人目の素数さん
垢版 |
2022/07/17(日) 23:08:06.63ID:O+96FzAs
>>828
後って?dx_i/dt?
0831132人目の素数さん
垢版 |
2022/07/18(月) 01:51:47.96ID:dXA+eO+l
>>823
∂/∂vδ(v-u)=-∂/∂uδ(v-u)
と変形してるだけ
u(t)の代わりにvと書きました
0832132人目の素数さん
垢版 |
2022/07/18(月) 02:39:20.72ID:ipG3C1dJ
X×Yの直積空間の閉集合はどう表せるのでしょうか?
F×G (ただしFはXの閉集合、GはYの閉集合)と表せるということで合っていますか?
0833132人目の素数さん
垢版 |
2022/07/18(月) 03:57:46.36ID:hENwXt36
>>832
F×Gの形の集合は閉集合だし、そうした形の集合を有限個持ってくるとそれらの和集合も閉集合。
一般にはそのように有限和の形で表される集合たち任意個の共通部分として表される。
0834132人目の素数さん
垢版 |
2022/07/18(月) 04:25:53.14ID:6YxrF13H
一般にヤバい話っていうのはプチエンジェル事件や
尼崎の角田美代子のようにすーっと消えていく。

ところが、今回は 安倍総理暗殺 っていう
「アメリカ政府が無視できない特殊な事例」だからな、
さすがにもみ消せない。
という事で、 「原因は宗教」って事にして統一教会たたきで
日本の政府もTVも足並みを揃える事にした。
0835132人目の素数さん
垢版 |
2022/07/18(月) 06:59:16.91ID:hF7fHcbX
陰謀論
0836132人目の素数さん
垢版 |
2022/07/18(月) 08:59:23.11ID:qLlNRYN/
斎藤毅著『微積分』


2次元の閉区間K  K = [a, b] × [c, d] の面積(area) (b - a) * (d - c) を m(K) で表わす。


と長方形の面積を定義しています。

次に、長方形の合併の面積を包除原理で計算しています。

その際、 m(空集合) の値が一般に必要になりますが、その値を定義していません。

また、複数の長方形の共通部分がまた長方形になることについて、言及も証明もしていません。

結構、雑ですね。
0837132人目の素数さん
垢版 |
2022/07/18(月) 09:05:21.57ID:6I/umC6N
雑なのはお前の思考回路や
0839132人目の素数さん
垢版 |
2022/07/18(月) 10:16:09.59ID:qLlNRYN/
斎藤毅著『微積分』

長方形の合併の面積を包除原理で計算していますが、組合せ論的議論についての説明は
一切なしですね。

式変形だけです。

説明が難しいところは証明なし(式変形の正当性の証明なし)で済ませますね。
0840132人目の素数さん
垢版 |
2022/07/18(月) 10:19:44.38ID:qLlNRYN/
包除原理を使った組合せ論的な計算による証明みたいに説明が難しい命題の証明を
厳密に書くのってどうやるんですか?

正しいのは分かりますが、形式的にどうやって証明するんですか?
0841132人目の素数さん
垢版 |
2022/07/18(月) 12:26:33.12ID:3RgdQwSY
>>833
R×Rの閉集合{(x,x)|x∈R}をどう表すんだよ
閉集合の基底はX×Y\U×Vだぞ
0843132人目の素数さん
垢版 |
2022/07/18(月) 12:44:11.07ID:M/Vf9x5d
>>841
X×Y - U×V = (X - U) × Y \cup X × (Y - V) だから>>832の形の集合全体も閉集合の準基としては採用できる
ただし基底にはなってないので有限和も考えないと例えば対角集合みたいなのは表せない
0844132人目の素数さん
垢版 |
2022/07/18(月) 14:10:28.76ID:6YxrF13H
TVで報道しているのは
山上家が崩壊した過程と統一教会の悪質さ。

「安倍総理暗殺」 の理由を
何も説明していない。
0845132人目の素数さん
垢版 |
2022/07/18(月) 16:31:55.23ID:24qHT5+Y
無理やろ
集合族
F = { A×B | A,Bは共にℝの閉集合 }
はそもそも無限のintersectionで閉じてる
すなわちAₖ×Bₖ∈Sなら
∩[k]Aₖ×Bₖ = (∩[k]Aₖ)×(∩[k]Bₖ) ∈ S
だから
しかしSの要素の有限和で全ての閉集合がかけるわけではない
実際Δ={(x,x) | x∈ℝ }はもちろん閉集合
Δ = A₁×B₁∪...∪Aₙ×Bₙ
とすると各Aₖ×Bₖは全部Δに含まれないといけないがそのためにはAₖ=Bₖかつ一元集合になる必要がある
よってAₖ×Bₖは全部一元集合なので矛盾
0846132人目の素数さん
垢版 |
2022/07/18(月) 16:41:22.84ID:lrb2Q5F2
>>845
だから共通集合でやることになるね
0847132人目の素数さん
垢版 |
2022/07/18(月) 17:15:46.57ID:2Q5zlvqr
>>846
イヤ、だから元々Sは閉集合を無限個とる操作で閉じてるからその操作には意味がない
一般に集合族Sが与えられたとしてSを閉集合とする位相を作るには大概
Sの無限共通部分で出来るS₁
S₁の有限和で出来るT₁
T₁の無限共通部分で出来るS₂
S₂の有限和で出来るT₂
....
の作業するを無限回繰り返さないとダメな場合が普通
今回のも簡単な操作2、3回で済ませるのは無理やろ
0849132人目の素数さん
垢版 |
2022/07/18(月) 17:40:43.70ID:qLlNRYN/
小林昭七著『続微分積分読本』


定理3

集合 R ⊂ I の面積が存在し、 f(x, y) が R で定義された有界な連続関数ならば積分

∫_{R} f(x, y) dx dy

が存在する。


という定理の証明中で

「グループ(iii)に属する I_{jk} では、 f(x, y) の下限 m_{jk} = 0 だから、」

などと書いていますが、間違っています。

マイナスになる可能性があります。

昭七だけに小学校を7年通ったということはないでしょうか?
0850132人目の素数さん
垢版 |
2022/07/18(月) 17:44:31.56ID:qLlNRYN/
「グループ(iii)に属する I_{jk}」とは一部分が R に含まれ、一部分が R の外にあるような
部分区間 I_{jk} のことです。

R の外では、 f(x, y) = 0 ですが、 R の点においては、 f(x, y) の値が正であるか 0 であるか負であるかは
全く不明です。
0851132人目の素数さん
垢版 |
2022/07/18(月) 17:49:06.37ID:qLlNRYN/
ちなみに小林昭七さんは昭和七年生まれです。
0852132人目の素数さん
垢版 |
2022/07/18(月) 18:16:18.37ID:hF7fHcbX
>>844
「安倍総理暗殺」の理由が報道されない理由は
安倍が総理ではないから。
0853132人目の素数さん
垢版 |
2022/07/18(月) 21:51:37.65ID:M/Vf9x5d
>>848
今回の場合に限らず (∩_i A_i) ∪ (∩_j B_j) = ∩_{i,j}(A_i ∪ B_j) だから有限和→無限共通部分で十分じゃない?
0854132人目の素数さん
垢版 |
2022/07/18(月) 22:15:43.66ID:6S8jz6PT
>>853
ダメな反例知らないけどいける証明も見たことないからダメなんやろと思ってる
Sが無限intersection閉じてるとして、それの有限和で書けるクラス作ったらもはやそれは無限intersectionで閉じてない
0855132人目の素数さん
垢版 |
2022/07/18(月) 22:26:28.36ID:jLhM2MWw
開集合の場合は開集合の直積U×Vの任意有限個の和の有限共通部分で書けるもの全体なんだし、これから補集合とればいいだけじゃないん?
0856132人目の素数さん
垢版 |
2022/07/18(月) 22:26:56.89ID:jLhM2MWw
任意有限個→任意個に訂正
0857132人目の素数さん
垢版 |
2022/07/18(月) 22:28:35.28ID:M/Vf9x5d
先に有限和を取ってから共通部分を考えないとダメだよ。
言い換えると、有限和で閉じてるクラスを考えてその無限共通部分で表されるクラスを作ると、>>853より新しい方は有限和でも閉じてる。
0858132人目の素数さん
垢版 |
2022/07/18(月) 22:31:08.94ID:jLhM2MWw
順番逆だったわ、有限個の共通部分の任意和だな
酔ってるとだめだわすまん
0860132人目の素数さん
垢版 |
2022/07/18(月) 22:37:53.44ID:M/Vf9x5d
>>858
開集合で考えるなら有限個の共通部分を取ってもU×Vの形にしかならないから任意和取るだけで十分だったりする。
一般にある集合族を準開基とする位相を考えるとかならその手順でやらないとダメだと思うけど。
0861132人目の素数さん
垢版 |
2022/07/18(月) 23:32:49.84ID:XAXIcH1s
>>854
すぐやん
0862132人目の素数さん
垢版 |
2022/07/18(月) 23:40:52.88ID:5R1VI83w
距離空間において、
x∈f^-1(A) ⇔ f(x)∈A
という変形をよく使うけど、これってfが連続写像もしくはxで連続であるっていう仮定が無いと使えないよね?
何の説明もなしに使ってる解答があって混乱してる
0863132人目の素数さん
垢版 |
2022/07/18(月) 23:51:23.49ID:2FC9qUmA
ごめんわけわからんこと言った
連続とはなんの関係もないか
頭ゴチャゴチャになってるので寝ます
0864132人目の素数さん
垢版 |
2022/07/19(火) 00:07:57.79ID:oG/BNXr/
距離も関係ない単なる逆像の定義だね
0865132人目の素数さん
垢版 |
2022/07/19(火) 15:22:35.59ID:23JZ+dL5
デルタ関数δ(x-c) を積分区間(-∞,c)まで積分したらどうなるか導けますかね
1/2になる気がするんですが
cは定数です
0866132人目の素数さん
垢版 |
2022/07/19(火) 16:50:45.73ID:bwuL2W/g
>>865
本義には無理
δ関数は汎関数積分の∫[〜]f(x)∂(x)dx
のように“テスト関数”f(x)と一緒に使う
f(x)は急減小関数とかコンパクトサポート関数とかの積分性がめちゃくちゃいいものをとる
定数関数1なんてもちろん入ってない
なので本義的には定義不能
0868132人目の素数さん
垢版 |
2022/07/19(火) 20:39:19.87ID:sALsYZsJ
本義的には定義不能が正しそうですね
ただ、デルタ関数の積分でヘヴィサイド階段関数θ(x)が出てきて、定義にはx=0含まないけどθ(0)=1/2とかθ(0)=0とか利便性に応じて使うみたいです(物理では)
超関数むずいっすね
0869132人目の素数さん
垢版 |
2022/07/19(火) 21:06:28.19ID:Xqs9+LUu
コンパクトリー群の随伴表現の固有値が非縮退になる理由が分からないんだけどだれか分かる?
0871132人目の素数さん
垢版 |
2022/07/19(火) 23:33:11.50ID:Xqs9+LUu
固有値というかルートか
一つのルートに対するルートベクトルの張る空間が1次元になることの示し方が分からない
0872132人目の素数さん
垢版 |
2022/07/20(水) 13:19:03.50ID:tPi43Zb0
コンパクトリー群なら左右不変なリーマン計量からリー環(の複素化)のG不変で非退化な内積が作れるからあとは半単純の場合と同じやり方でいけるんじゃないか?
0873132人目の素数さん
垢版 |
2022/07/20(水) 13:31:57.98ID:hfIz94Hn
>>872
非退化な内積が作れることはわかる
でもその「半単純と同じやり方」がわからないんだ...
0874132人目の素数さん
垢版 |
2022/07/20(水) 14:29:09.60ID:tPi43Zb0
とりあえず概略だけ書くで。

Gをコンパクトリー群、gをそのリー環の複素化、hをカルタン部分環、( , )をG不変で非退化な内積とする。
ルートαとg_αの0でない元Xに対し、g_{-α}の元Yで(X,Y)が0でないものがとれる。
H_0:=[X,Y]はhの元だが、hの元Hに対して([X,Y],H)を考えることにより、内積のG不変性からH_0が0でないことがわかる。
さらにh上でも内積が非退化であることに注意してX,Yを取り替えて同じようなことをすると[g_α,g_{-α}]=CH_0がわかる。
最後にgの部分空間V:=CY + CH_0 + g_α + g_{2α} +…について、Vがad X, ad Yで閉じていることからV上でad H_0のトレースは0.
これによりg_α=CXがわかる。
0875132人目の素数さん
垢版 |
2022/07/20(水) 14:45:12.05ID:hfIz94Hn
>>874
ありがとう
これでちょっと頑張ってみるわ
ちなみに途中から入ってくるCって何を表してる?
0876132人目の素数さん
垢版 |
2022/07/20(水) 17:31:47.11ID:cn6ZaibS
a_i < b_i とする。

Q := [a_1, b_1] × … × [a_n, b_n]

Q の測度は 0 でないことを証明せよ。

この証明ですが、簡単そうに見えますが、それほど簡単ではない証明が本に載っています。

簡単な証明はないですか?
0877132人目の素数さん
垢版 |
2022/07/20(水) 18:45:17.84ID:tPi43Zb0
>>875
だいぶ端折ってるからわからんとこあったら聞いてくれ
Cはmathbbの出し方がわからんかっただけで複素数体のCです
0878132人目の素数さん
垢版 |
2022/07/21(木) 00:23:25.86ID:PtPzQBp+
x^n+y^n=z^n ± 1 (nは3以上の整数)を満たす自然数x,y,zは任意のnに対して無限個存在するのでしょうか?
それとも有限個?
0880132人目の素数さん
垢版 |
2022/07/21(木) 07:32:11.79ID:nhifmKm6
>>876
μ(Q)=μ([a_1, b_1]) × … × μ([a_n, b_n])=(b_1-a_1) × … × (b_n-a_n)≠0
0881132人目の素数さん
垢版 |
2022/07/21(木) 12:19:40.37ID:dpegJJB+
G={f(X)=pX+q | p,q∈R, p≠0}が写像の合成に関して群になるとき、単位元と逆元はどのようになりますか?
0883132人目の素数さん
垢版 |
2022/07/21(木) 13:01:18.51ID:dpegJJB+
>>882
わかりました
ありがとうございます!

このpx+qのときに{{p,q},{0,1}}を考えるというのは何故なのでしょうか?
0884132人目の素数さん
垢版 |
2022/07/21(木) 13:46:09.68ID:n1AVA04Y
>>883
タテ・ベクトル

x
1

に左から作用させるとベクトル

px+q
1

になりアフィン変換の行列表示を得る
(変換の合成と行列の積が対応するね)
0885132人目の素数さん
垢版 |
2022/07/21(木) 16:31:14.69ID:n1AVA04Y
ごめん、ちょっと大袈裟だったか
元の問題だったら、
Y=pX+q
の逆は
X=p^{-1}Y- p^{-1}q
と中学生の計算だな

行列表示しておくとn次元ユークリッド空間でアフィン変換は
pがn次正方行列、qがn次ベクトルとかけるし、合同変換は
pを直行行列O(n)などに取れば良い、
と言うような話を記述しやすい
0886132人目の素数さん
垢版 |
2022/07/21(木) 17:25:00.88ID:6aTG/AQE
ばかばかしいね
簡単なもんだいをなかみのないままこねくりまわしている。
0887132人目の素数さん
垢版 |
2022/07/21(木) 23:37:42.65ID:dpegJJB+
>>885
あ、たしかに高校までの考え方で簡単にできる問題でしたね
自分も気付きませんでした
アフィン変換の行列表示?はまだ習っていないと思うのでこれから習うのが楽しみです
ありがとうございました!
0888132人目の素数さん
垢版 |
2022/07/22(金) 10:19:17.19ID:Y1ggWoKi
単射の定義について質問です
集合X, Yについて f:X→Y を写像とします
単射の定義は「任意のa, b∈X に対し a≠b ⇒ f(a)≠f(b)」だとします
Yをf(X)に制限して「任意のα, β∈f(X)⊂Y に対し α≠β ⇒ αの原像≠βの原像」が成り立つfを単射としても
もとの単射の定義と同値になりますか?
0889132人目の素数さん
垢版 |
2022/07/22(金) 10:52:55.82ID:1XI1PxJf
原像が1点と暗黙のうちに仮定している時点で単射じゃん。
「α, β∈f(X)⊂Y に対し α≠βならば αの原像∩βの原像=空集合」はfが単射でなくても、原像の定義から自明に成り立つ。
0890132人目の素数さん
垢版 |
2022/07/22(金) 10:53:48.21ID:ZnYOCTTw
>>888
ならない
というか、「任意のα, β∈f(X)⊂Y に対し α≠β ⇒ αの原像≠βの原像」はどんなfに対しても成り立つ
0892132人目の素数さん
垢版 |
2022/07/22(金) 20:20:34.33ID:gdAucGD2
Q を R^n における閉長方形とする.
f : Q → R とする.
f は Q で積分可能とする.

f(x) > 0 for any x ∈ Q ⇒ ∫_Q f > 0

が成り立つことを証明せよ.

この問題ですが,簡単そうに見えますが,教科書に書いてあるそんなに簡単でない定理を
使うとあっさりと解けます.

一見,積分の定義から簡単に成り立つことが言えそうに見えますが,そのような解答はありますか?
0893132人目の素数さん
垢版 |
2022/07/22(金) 20:40:27.19ID:gdAucGD2
おそらく,著者が期待している解答を以下に書きます:

定理:
Q を R^n における閉長方形とする.
f : Q → R とする.
f は Q で積分可能とする.
f が非負で, ∫_Q f = 0 ならば, f の値は,測度ゼロである Q の部分集合 D 以外の Q の点で 0 である.

f(x) > 0 for all x ∈ Q だから, f は非負である.
よって,積分の定義から簡単に分かるように, ∫_Q f ≧ 0 である.
今,仮に, ∫_Q f = 0 であると仮定してみる.
すると,上の定理により, f の値は,測度ゼロである Q の部分集合 D 以外の Q の点で 0 である.
よって, f が 0 以外の値を取るのは, D の部分集合においてである.
測度ゼロの集合の部分集合はまた測度ゼロであるから, f が 0 以外の値を取るのは測度ゼロの
集合においてである.
Q は測度ゼロではなく, f(x) ≠ 0 for all x ∈ Q であるからこれは矛盾である.
0894132人目の素数さん
垢版 |
2022/07/22(金) 22:18:10.00ID:J02IRPVB
コレは?
Aₙ = { x | f(x) ≧ 1/n }
とおく
あるnでμ(Aₙ) = m > 0 なら∫f(x)dx ≧ m/nなのでよい
μ(Aₙ) = 0 ∀n ならμ(∪Aₙ) = 0だが仮定より∪Aₙ=全体なので矛盾
0895132人目の素数さん
垢版 |
2022/07/23(土) 09:08:39.75ID:tjv7zC++
>>894

μ(A) ってかならず存在するんですか?
0896132人目の素数さん
垢版 |
2022/07/23(土) 11:32:56.20ID:Y2RMmY14
>>895
f(x)が可測関数でなければそもそも∫f(x)dxが定義できない
f(x)が可測⇔任意のsに対して{ x; f(x) > a } が可測集合
0897132人目の素数さん
垢版 |
2022/07/23(土) 14:46:06.62ID:PeHIUTl0
0<f<∞なので、非負な項の和aからなる単関数gの上限でfの積分を表せる(定義)ことと
gについてほとんどいたるところでg>0となる事が言えればOK?
0898132人目の素数さん
垢版 |
2022/07/24(日) 12:40:44.23ID:0kKeH2DF
Q.気象庁が「明日の東京の降水確率は20%です」という予報を出した時、何に対する何の比率が20%なのだろうか?
確率の定義に即して、分子・分母がはっきり分かるように説明せよ
ただし「確率」という単語を説明に使ってはならない



この問題誰か解いてくれませんか?
日本語での説明問題なのですが全く意味不明です
0899132人目の素数さん
垢版 |
2022/07/24(日) 13:08:18.06ID:tNVZF0gd
>>896

ありがとうございました.
0900132人目の素数さん
垢版 |
2022/07/24(日) 13:15:05.48ID:tNVZF0gd
Q = [0, 1] × [0, 1] とする.
Q の部分集合で以下の条件を満たす集合 S の例をあげよ.

(1) closure(S) = Q
(2) #({(x0, y) ∈ Q | y ∈ [0, 1]} ∩ S) ≦ 1 for any x0 ∈ [0, 1]
(3) #({(x, y0) ∈ Q | x ∈ [0, 1]} ∩ S) ≦ 1 for any y0 ∈ [0, 1]
0901132人目の素数さん
垢版 |
2022/07/24(日) 14:27:49.85ID:/yE3haLm
>>900
f(x) = x - [x]([x] は x を超えない最大の整数)とするとき
S := {(f(√2 a), f(√3 a)) | a は有理数} とすればいい
a の動く範囲は整数でも十分だけど(1)を示すのが少し面倒になる
0902132人目の素数さん
垢版 |
2022/07/24(日) 18:10:49.99ID:JskeI+8g
>>901
見事ですねえ、思いつきませんでした。
0903132人目の素数さん
垢版 |
2022/07/24(日) 19:19:51.93ID:xapNNQNo
9.5 (2)(a)を解ける賢い方、教えてください。
大学の微分積分です。。。
https://i.imgur.com/DlBtiUs.jpg
0905132人目の素数さん
垢版 |
2022/07/24(日) 21:31:47.80ID:bep9qfXF
>>898
誰かこの問題教えてくれませんか?
お願いします🙇‍♂
0907132人目の素数さん
垢版 |
2022/07/24(日) 21:40:24.24ID:JskeI+8g
>>898
分母は「明日の東京の降水確率は20%です」という予報が出た日の数
分子はその予報が出た日に実際に雨が降った日の数
0908132人目の素数さん
垢版 |
2022/07/24(日) 21:44:38.31ID:0Hjr5VjH
>>906
誰がどこに答えてくれてるんですか?
0910132人目の素数さん
垢版 |
2022/07/24(日) 21:48:03.98ID:0Hjr5VjH
>>909
だから投稿したスレのどこに答えてくれてくれてる人がいるんだ?
0911132人目の素数さん
垢版 |
2022/07/24(日) 21:51:15.71ID:oT73/c/i
なんで非推奨のマルチポストしてる奴が偉そうなんだ?
そんな大量に投下したのかよ
せめて自分で探せよ
0912132人目の素数さん
垢版 |
2022/07/24(日) 21:53:22.63ID:0Hjr5VjH
>>911
何が偉そうなんだよw
答えてくれてるスレがどこにあるのか言ってみろや
分からないなら黙ってろよニート
0913132人目の素数さん
垢版 |
2022/07/24(日) 21:54:37.62ID:Wt8S/Vrr
気象予報士のサイト見ろとか全然答えになってなくね
0915132人目の素数さん
垢版 |
2022/07/24(日) 21:56:59.50ID:0Hjr5VjH
>>914
論破って何に対して?
お前みたいなアホには聞いてねえから黙ってれば?w
0916132人目の素数さん
垢版 |
2022/07/24(日) 21:59:09.64ID:oT73/c/i
>>915
答えになってないっていう意見に対してだよ
俺に言ってなくてもおかしなこと書いてあるのは分かるから指摘してやってんだよ
0917132人目の素数さん
垢版 |
2022/07/24(日) 21:59:56.40ID:0Hjr5VjH
>>916
答えになってないなんて一言も言ってないぞ
もしかして>>913と勘違いしてるのか?
0919132人目の素数さん
垢版 |
2022/07/24(日) 22:01:58.76ID:28Dqt4ko
>>918
無理やりそういうことにしてて草
勘違いしちゃったねぇw
0921132人目の素数さん
垢版 |
2022/07/24(日) 22:04:17.60ID:28Dqt4ko
>>920
イライラで草
図星で悔しいねえw
0922907
垢版 |
2022/07/24(日) 22:15:16.28ID:JskeI+8g
>>917
俺は答えた
0923132人目の素数さん
垢版 |
2022/07/24(日) 22:16:36.74ID:WYuvbd17
>>922
あなたは確かに答えてくださいました
素晴らしい
ありがとうございます
0924132人目の素数さん
垢版 |
2022/07/24(日) 22:24:23.94ID:xapNNQNo
>>904
ありがとうございます!
0925132人目の素数さん
垢版 |
2022/07/24(日) 22:37:50.89ID:JskeI+8g
>>923
自分の思う通りに動いた人間を誉め、そうでない人間を罵倒することによって人を操ろうとする卑劣な精神を感じる。
0926132人目の素数さん
垢版 |
2022/07/24(日) 22:52:32.03ID:BUWSY4GD
>>925
ほえー用済みじゃさっさと失せろ
0927132人目の素数さん
垢版 |
2022/07/25(月) 11:41:18.14ID:Ltj4OJHE
このスレのお客様にでもなったつもりなんじゃね?お客様は神様ですなんだろ型迷惑客。
用済み失せろとか言い出す時点でお客様気分確定

労働者を労わず消費者の理不尽にかしづく国、日本
こういったお客様は店の事務所にご案内だ
0928132人目の素数さん
垢版 |
2022/07/25(月) 11:59:35.36ID:77cFTX67
不良品である割合に差があるかどうかの仮説限定って何を検定統計量として用いれば良いのでしょうか。
0930132人目の素数さん
垢版 |
2022/07/26(火) 13:36:33.82ID:rjYQacky
>>901

ありがとうございました.

S ⊂ R^n を有界とする.

S の孤立点の集合は可算集合であるか?
0931132人目の素数さん
垢版 |
2022/07/26(火) 14:48:00.61ID:rjYQacky
あ,わかりました.

S ⊂ R^n を有界とする.
I を S の孤立点の集合とする.
x ∈ I とする.
{|y - x| | y ∈ I - {x}} の下限は正の実数である.
各 x ∈ I に r_x := inf {|y - x| | y ∈ I - {x}} > 0 を対応させる.
任意の x ∈ I に対して, {y | |y - x| < r_x} ∩ I = {x} である.
任意の x ∈ I に対して,有理数の集合の稠密性により, {y | |y - x| < r_x} は有理点を含む.
そのような有理点のどれか1つを x に対応させれば, I から Q^n への単射ができる.
よって, I は高々可算な集合である.
0932132人目の素数さん
垢版 |
2022/07/26(火) 14:49:07.51ID:rjYQacky
あ, S は有界じゃなくてもOKですね.
0933132人目の素数さん
垢版 |
2022/07/26(火) 14:50:21.45ID:An7phzKx
>>930
R^n が第二可算であることから可算集合。
Sは有界でなくてもよく、有界であったとしても孤立点が有限個とは限らない。
0934132人目の素数さん
垢版 |
2022/07/26(火) 14:57:40.30ID:An7phzKx
自分で分かってたみたいでよかったです。
証明もほぼそれでOKだけど、r_x は inf {…}/2 としないとIからQ^nへの対応が単射とは限らないので注意。
0935132人目の素数さん
垢版 |
2022/07/26(火) 14:59:44.07ID:rjYQacky
>>933-934

ありがとうございました.

訂正します:

I を S の孤立点の集合とする.
x ∈ I とする.
{|y - x| | y ∈ I - {x}} の下限は正の実数である.
各 x ∈ I に r_x := (1/2) * inf {|y - x| | y ∈ I - {x}} > 0 を対応させる.
任意の x, x' ∈ I かつ x ≠ x' に対して, {y | |y - x| < r_x} ∩ {y | |y - x'| < r_x'} は空集合である.
任意の x ∈ I に対して,有理数の集合の稠密性により, {y | |y - x| < r_x} は有理点を含む.
そのような有理点のどれか1つを x に対応させれば, I から Q^n への単射ができる.
よって, I は高々可算な集合である.
0937132人目の素数さん
垢版 |
2022/07/28(木) 18:00:34.63ID:XYlOZovv
以下の条件を満たす集合 A が存在することを証明せよ.

(1) A は1次元の開区間の可算個の和集合である.
(2) A の境界の測度はゼロでない.
0938132人目の素数さん
垢版 |
2022/07/28(木) 19:22:50.18ID:Vt9Ae2Es
ここは出題スレじゃないよ
0941132人目の素数さん
垢版 |
2022/07/29(金) 19:58:54.51ID:8fr3lszP
Hi(X)を特異ホモロジーとしてh^i=Hom(Hi(X),Z)と双対を取ったときに
h^iがコホモロジーの公理を満たさない事を示せという問題が分かりません
トージョンが消える事が問題かとおもって組(RP^2,RP^1)のコホモロジー完全列を調べて
成り立たない事が言えないかと考えたのですがうまくいきませんでした
分かる人いたら教えて下さい
0942132人目の素数さん
垢版 |
2022/07/29(金) 21:29:21.76ID:x7cRi59b
>>941
Hom(-,Z)が完全じゃないからでしょ
0943132人目の素数さん
垢版 |
2022/07/29(金) 21:33:24.55ID:x7cRi59b
0→Z→Z→Z/2→0:完全
0←Hom(Z,Z)=Z←Hom(Z,Z)=Z←Hom(Z/2,Z)=0←0:全然非完全
0944132人目の素数さん
垢版 |
2022/07/29(金) 21:56:49.72ID:8fr3lszP
>>942
はい
それを元に成り立たない例を構成しようと思ったんですが
連結準同型の部分がホモロジーの連結準同型の双対である保証がないので
単純に双対が完全性を保たないってだけだと別の連結準同型取ってきて
完全列になる場合が排除できないところで困ってましたが
メビウスの帯とその境界使って包含写像のinduced mapのところが2倍写像となる
完全列考えれば良さそうですね
ありがとうございます
0945132人目の素数さん
垢版 |
2022/07/29(金) 22:10:06.10ID:nApN5BOK
>>944
問題文の原文を見せて欲しい
「一般には成り立たない」なのか「どんな場合にも成り立たない」なのかハッキリしていない
0946132人目の素数さん
垢版 |
2022/07/29(金) 22:53:15.95ID:8fr3lszP
>>945
Hatcherの代数トポロジーのp.205(pdfではp.214)
http://pi.math.cornell.edu/~hatcher/AT/AT+.pdf
の問題7です

コホモロジーの公理はp.202にあり
連結準同型以外の部分の写像は関手hで誘導したhomと指定されてる状況です
0947132人目の素数さん
垢版 |
2022/07/29(金) 22:55:40.11ID:8fr3lszP
>>945
あと示すのは一般には成り立たないの方かと思います
ねじれのない空間の場合にはそのまま特異コホモロジーの完全列になるので
0948132人目の素数さん
垢版 |
2022/07/30(土) 00:01:41.80ID:uNuHzJ9q
なら探してみるのはホモロジー群がねじれ群でHom(-,ℤ)作用させて消えてしまう項がある話でしょ
例えばℝℙ²をメビウスバンドMと円盤D²に分ける
ただしM∩D = S¹は円周
Mayer–Vietoris列
→ H₁(S¹) → H₁(M)⊕H₁(D²) → H₁(ℝℙ²)
→ H₀(S¹) → H₀(M)⊕H₀(D²) → H₀(ℝℙ²)
は長完全列
→ 0→ 0 ⊕ 0 → 0
→ ℤ → ℤ ⊕ 0 → ℤ/2ℤ
→ ℤ → ℤ ⊕ ℤ → ℤ
誘導するけどコレにHom(-,ℤ)を作用させると
← 0← 0 ⊕ 0 ← 0
← ℤ ← ℤ ⊕ 0 ← 0
← ℤ ←ℤ ⊕ ℤ ←ℤ
というchainができるけど2段目のℤ ← ℤ ⊕ 0は2倍写像で全射では無い、しかし上の段は全部0なので完全列になってない
0949132人目の素数さん
垢版 |
2022/08/01(月) 06:12:22.16ID:dHWXTxuI
複素関数e^zは一価関数なのに
a≠eのa^zは多価関数ってなぜ?
たとえば1^i=e^(2nπii)=e^(-2nπ)
eもloge=1+2nπiなのだから
e^i=e^(-2nπ+i)=e^(-2nπ)(cos1+isin1)なのでは?
0952132人目の素数さん
垢版 |
2022/08/01(月) 12:00:18.56ID:0c3xP5Im
e^zの定義はe^{zloge}ではなかったと思う
0953132人目の素数さん
垢版 |
2022/08/01(月) 16:32:43.05ID:Gz24ryfT
多変数の広義積分のところを読んでいるのですが,おそらく n=1 の場合を考えると,1変数の
微分積分でのよくある広義積分の定義と一致すると思います.

多変数の広義積分のやり方のほうが分かりやすいと思います.

1変数の場合も多変数に通用するやり方で広義積分を論じている本はありますか?
0954132人目の素数さん
垢版 |
2022/08/01(月) 16:37:24.46ID:ni+squ1I
>>953
ちょっと質問の意味が分かりにくい
0955132人目の素数さん
垢版 |
2022/08/01(月) 18:40:08.60ID:Gz24ryfT
今,読んでいる本で,非負連続関数に対する広義積分の定義が以下です:

A を R^n の開集合とする.
f を A 上の非負連続関数とする.
∫_A f を sup {∫_D f | D ⊂ A はコンパクトな体積確定集合} と定義する.

D に有界という条件を課すのは有界な集合上でしか普通の積分は定義していないので,当たり前です.
D に閉集合という条件を課すのはなぜでしょうか?
0956132人目の素数さん
垢版 |
2022/08/01(月) 18:49:19.99ID:XaA77f47
>>955
具体例で考えてみよう
特にn=1のときを
0957132人目の素数さん
垢版 |
2022/08/01(月) 18:55:38.08ID:hzbZ4qDF
>>953
一致しません。
sin x/xとか高次元の定義で広義積分可能かどうか判定してみれば?
0958132人目の素数さん
垢版 |
2022/08/01(月) 23:13:44.13ID:dHWXTxuI
>>952
a^zの定義と違うんならe^zと書くべきでないと思う
exp(z)が妥当でないかしら
0959132人目の素数さん
垢版 |
2022/08/02(火) 01:43:59.32ID:RH9zMX6s
なかやまきんに君が吉本退社バブル″数秩@
ユーチューブ収入は7000万円

きんに君は「世界でも活躍できるようになりたい。その夢をかなえたくて
退所した」と説明。「アメリカの筋肉情報って、めちゃめちゃ面白い。
スーパーのプロテインのゾーンもめちゃめちゃある。壁一面ドーン!と。
めちゃめちゃ面白くないですか?」と目を輝かせ、今後は筋肉知識を広げ、
新たな情報を配信していきたいと語った。
すでに「結構、メールとかも頂いて。CM依頼とか、筋肉系なので
食品とか」と、仕事依頼は来ていることも明かした。
退所発表後の反響については「驚くことに大変反響いただきまして、
CM依頼とかイメージキャラクターの依頼が7〜8件来てるんです。
この2、3日で」とバブル到来≠におわせた。さらに年収は?と
聞かれると「めちゃくちゃ貯金、あります!貯金の金は筋肉の筋!」
と鍛えた体があると訴え、スタッフを笑わせていた。
0960132人目の素数さん
垢版 |
2022/08/02(火) 08:45:53.27ID:XdN9uWcE
>>958
eとaは意味が違うのでOK
0961132人目の素数さん
垢版 |
2022/08/02(火) 20:21:12.60ID:9uFfoMVa
>>960
は?
同じ複素数ですが?
0962132人目の素数さん
垢版 |
2022/08/02(火) 21:10:36.68ID:XdN9uWcE
でも意味が違う
0963132人目の素数さん
垢版 |
2022/08/02(火) 21:12:57.58ID:9uFfoMVa
>>962
説明して
0964132人目の素数さん
垢版 |
2022/08/02(火) 21:13:25.31ID:9uFfoMVa
なぜa=eだけ特別扱いする理由があるのか
0965132人目の素数さん
垢版 |
2022/08/02(火) 21:18:20.38ID:XdN9uWcE
e^zはeの一つの定義を特殊値とする
べき級数として定義する。
例えばRudinの本はこのやり方を採用している。
本来はWeierstrass流であり、円周率もe^{iz}+1の
最小の正の零点として定義する。
0967132人目の素数さん
垢版 |
2022/08/03(水) 00:52:20.36ID:xa6kyHcD
まぁe^zを多価関数とみなしたい場合が出てきたらその旨明記して使えばいいんじゃやいの?
単にそうすべき場面がほとんどないから使われないだけでしょ?
0968132人目の素数さん
垢版 |
2022/08/03(水) 03:26:30.86ID:miPVTGIx
>>966
なら1^z=1とすればピッタリね
0969132人目の素数さん
垢版 |
2022/08/03(水) 16:35:15.31ID:AGcJG1sk
>>単にぴったり一致するからじゃないの?

>>なら1^z=1とすればピッタリね

「ぴったり」と「ピッタリ」は
かなり意味が違うようだね
0970132人目の素数さん
垢版 |
2022/08/03(水) 20:05:51.19ID:miPVTGIx
>>969
同じだケド?
0971132人目の素数さん
垢版 |
2022/08/03(水) 20:48:46.25ID:22Dgj5ca
>>970
そう断言できるのは
966と968が自演であることの証拠と考えてよいか?
0973132人目の素数さん
垢版 |
2022/08/03(水) 21:48:50.80ID:miPVTGIx
>>971
はぁ
自演の必要ってあるのかw
0974132人目の素数さん
垢版 |
2022/08/03(水) 21:52:56.22ID:miPVTGIx
>>969,971
e^zが一般の複素数aについてのa^zの定義と異なるのは
「ピッタリ一致するから」と>>966が書いていたから
それは実数関数と「ピッタリ一致する」という意味だと解釈
つまり1^z=1が実数関数1^xの解析接続として「ピッタリ一致」しているわけ
0975132人目の素数さん
垢版 |
2022/08/03(水) 21:56:18.06ID:22Dgj5ca
>>974
そう解釈しているなら全く問題ない。
0976132人目の素数さん
垢版 |
2022/08/03(水) 22:28:59.17ID:miPVTGIx
>>975
しかし複素函数の1^zは1ではなくて多価関数
1^z=1とすると間違いとされる
e^zとは書くべきでなくΣz^n/n!=exp(z)と書くべき
そしてe^zも多価関数を表すとするべき
工学でも物理でもe^xもあまり使わない
exp(x)が妥当だということが数学以外でのコンセンサス
0977132人目の素数さん
垢版 |
2022/08/03(水) 22:57:46.10ID:22Dgj5ca
>>976
工学や物理学の都合もあるだろうが
数学者たちの感情を満足させるためには
EulerやWeierstrassらの記号法を残しておかねばならない
0978132人目の素数さん
垢版 |
2022/08/04(木) 01:39:47.77ID:JNkykMa9
数学での複素表記 a+bi i:虚数単位
電気電子工学での複素表記 a+jb j:虚数単位 i:交流電流
0979132人目の素数さん
垢版 |
2022/08/04(木) 02:13:18.03ID:2zMvcFob
>>976
どこから来てる自信か知らんが数学の話で工学や物理の人間が数学畑の人間に“こうすべき”などという言葉がはけるのがアンポンタン
0981132人目の素数さん
垢版 |
2022/08/04(木) 08:08:51.12ID:iJDYaUtB
>>955,957

ありがとうございました.
0982132人目の素数さん
垢版 |
2022/08/04(木) 09:13:47.41ID:cn+b/DZH
電気電子工学ではΩをオームと読む
0983132人目の素数さん
垢版 |
2022/08/04(木) 10:08:54.00ID:6TVllEjA
そりゃ人名のOhmが元だし工学に限らず抵抗の単位をオメガなんて読むやつなんておらん
記号Ωを使った理由は知らんが
0984132人目の素数さん
垢版 |
2022/08/04(木) 12:33:09.92ID:iJDYaUtB
松坂和夫著『解析入門下』

A を有界集合とする.
K_A を A の定義関数とする.
A ⊂ I なる区間 I をとるとき, K_A の I における上積分および下積分の値は, I によらず A によって一意的に定まる.

という内容の補題があります.

その証明のはじめの部分ですが,


I, J をともに A を含む区間とする.はじめに I ⊂ J である場合を考える.
その場合,上の右の図のように J を網状分割すれば, I 以外の長方形の部分では K_A = 0 であるから,
上積分,下積分はともに 0 に等しい.


などと書いています.

I の境界と A の共通部分が空集合でない場合には,松坂さんの論法は成り立ちません.

デリケートな議論をしているのに,なぜこんなにも不注意なのか全く理解できません.
0985132人目の素数さん
垢版 |
2022/08/04(木) 12:47:12.51ID:iJDYaUtB
その後,

I, J をともに含む区間 K を作り,↑の結果を適用して,
K_A の I 上の上下積分と J 上の上下積分は等しいことを証明しています.

K はいくらでも大きな区間をとってもいいので,

I をその内部に含む J を考え, I を少し大きくした I' ⊂ J を考えて,

K_A の I 上の上下積分と I' 上の上下積分が等しいことを示せばいいですね.
I' を限りなく小さくすれば, K_A の I 上の上積分と I' 上の上積分の差はいくらでも小さく
なるのでこれらは等しいことが分かります.
0986132人目の素数さん
垢版 |
2022/08/04(木) 12:48:46.06ID:iJDYaUtB
訂正します:

その後,

I, J をともに含む区間 K を作り,↑の結果を適用して,
K_A の I 上の上下積分と J 上の上下積分は等しいことを証明しています.

K はいくらでも大きな区間をとってもいいので,

I をその内部に含む J を考え, I を少し大きくした I' ⊂ J を考えて,

K_A の I 上の上下積分と I' 上の上下積分が等しいことを示せばいいですね.
I' を I に限りなく近づければ, K_A の I 上の上積分と I' 上の上積分の差はいくらでも小さく
なるのでこれらは等しいことが分かります.
0988132人目の素数さん
垢版 |
2022/08/04(木) 13:14:51.48ID:iJDYaUtB
松坂和夫さんの『解析入門』シリーズは,全体としての統一感がないですよね.

一人の著者が書いた本とはとても思えません.

Walter Rudinの本と酷似している部分は既に確認済みです.
0989132人目の素数さん
垢版 |
2022/08/04(木) 21:33:57.86ID:d/omH9Ei
>>979
いや
数学だけが過去に固執している
exp(z)を使うのがベスト
同様にして
1+2+3+…=ζ(-1)=-1/12
とか部外者を煙に巻くのもいかがなものかね
0990132人目の素数さん
垢版 |
2022/08/04(木) 21:44:45.24ID:RodH6OOt
>>989
そういうアンポンタンな通俗の読み物ばっかり読んでキッチリした数学書読んでないからそんなアホなことがほざけるんだよバーカ
0991132人目の素数さん
垢版 |
2022/08/04(木) 22:02:16.08ID:d/omH9Ei
>>990
おやおや
ζ関数はs>0であるときのζ(s)=1+2^(-s)+3^(-s)+…を解析接続することでζ(-1)の値を考えることができるようになるわけだが
それは1+2+3+…であるわけではない
あくまでs>0でのζ(s)を解析接続した複素函数のs=-1のときの値ζ(-1)にすぎない
0994132人目の素数さん
垢版 |
2022/08/04(木) 22:39:45.38ID:d/omH9Ei
>>993
>素人相手の通俗本
だからそれを止めろっていっているのがワカラン金
1+2+3+…=-1/12
がまさにそれだ
止めるべき
0995132人目の素数さん
垢版 |
2022/08/04(木) 22:41:20.73ID:T2kTsJyE
>それは1+2+3+…であるわけではない
>あくまでs>0でのζ(s)を解析接続した複素函数のs=-1のときの値ζ(-1)にすぎない

物理出身なら、アーベル総和法とかボレル総和法くらい知ってそうなもんだけど、
この人は総和法の概念を1つも知らないのかな。

まあ、アーベル総和法だと AΣ[n=1〜∞] n =+∞ で、
ボレル総和法だと BΣ[n=1〜∞] n =+∞ で、
結局は −1/12 なんて出てこないので例としては不適切だが、
ζ関数の解析接続で発散しない値が割り当て可能なら「ζ総和法」とでも名付けて
新しい総和法にすればいいだけ。それを例えば ζΣ[n=1〜∞] a_n と表記すれば、

ζΣ[n=1〜∞] n =−1/12

が厳密に成り立つわけで、このことにインチキな要素は1つもない。

Σ[n=1〜∞] n = −1/12

と書いちゃったらインチキだけどね。
0997132人目の素数さん
垢版 |
2022/08/04(木) 22:53:03.82ID:RodH6OOt
アホか
書いてるレベルでせいぜい学部でもがいてるレベルのアンポンタンやとすぐわかるわ
そしてその程度のカスみたいな力しかないくせにその認識すらなく数学界に一言苦言申し上げるアフォ
高木とかセタとかとギロチ50歩100歩
10011001
垢版 |
Over 1000Thread
このスレッドは1000を超えました。
新しいスレッドを立ててください。
life time: 98日 1時間 44分 37秒
10021002
垢版 |
Over 1000Thread
5ちゃんねるの運営はプレミアム会員の皆さまに支えられています。
運営にご協力お願いいたします。


───────────────────
《プレミアム会員の主な特典》
★ 5ちゃんねる専用ブラウザからの広告除去
★ 5ちゃんねるの過去ログを取得
★ 書き込み規制の緩和
───────────────────

会員登録には個人情報は一切必要ありません。
月300円から匿名でご購入いただけます。

▼ プレミアム会員登録はこちら ▼
https://premium.5ch.net/

▼ 浪人ログインはこちら ▼
https://login.5ch.net/login.php
レス数が1000を超えています。これ以上書き込みはできません。

ニューススポーツなんでも実況